You are on page 1of 82

Practice

Test Explanations
Step 1 Practice Test # 18 – Block 1
Question Total: 50


Step 1 Practice Test #18 Explanations


Block 1

1. 65-year-old man comes with vague right-sided flank pain and 10-lb weight loss over past 8 weeks. No recent trauma or
back strain. Urine is darker than usual over 1.5 weeks. Renal ultrasound shows 1.76-cm solid mass in upper portion of
right kidney. Urine shows 47 erythrocytes, 5 leukocytes, no bacteria. Dx?
- Renal cell carcinoma


Why it’s right: This is a patient with recent weight loss, flank pain, and hematuria, which at his age, is 100% renal cell carcinoma
(RCC) until proven otherwise. The typical triad is the latter two symptoms (flank pain, hematuria) plus palpable mass. But the triad
doesn’t always need to be complete. This is how the medical board examiners separate the men from the boys (or rather, to satisfy
all our future feministic physicians, the women from the girls). They’re not always going to give you the complete because they want
to make an easy question a more difficult one. This is how medicine is in real life too, so they’re not doing it just to trick you –
medicine is not a triad or a pentad, and when it does fit into that cookie-cutter presentation, it’s a real miracle, and that’s not even
the cure.

Nevertheless, they still give you the palpable mass; they provide the ultrasound results – a solid mass. So there you go, a freaking
miracle. Also, please note, a cystic mass is more favorable and signifies a benign process, so here the word “solid” directs the test-
taker towards malignancy.

Why the others are wrong: Metastasis from another cancer would present with other signs/symptoms of another malignancy
elsewhere in the body – but again, not always. An abscess is less likely to be a solid mass – similarly, a hematoma would also not be a
solid mass, but a combination of cysts, fluid, etc. An abscess would also present with fever and have an acute onset with bacteria
and WBCs (more than just 6, which is borderline normal) in the urine. Finally, transitional cell carcinoma is referring to cancer of the
ureter or pelvis, located either in the ureter or in the inner portion of the kidney (the pelvis). The cancer described is not only typical
of RCC but also is located on the upper portion of the kidney – the cortex – and therefore is derived from the renal cortex
parenchyma itself.

Take home point: Patient >50 YO with weight loss, hematuria, and flank pain must have RCC ruled out is therefore is the top Dx.

2. 31-year-old man who is a professional cyclist undergoes extensive physiologic testing as part of his training regimen.
His resting pulse is 32/min, and blood pressure is 112/60 mm Hg. Echocardiography shows dilated ventricles with normal
function and a left ventricular ejection fraction of 76%. Which of the following best describes the findings in this patient?
- Eccentric hypertrophy


Why it’s the right answer: “Dilated ventricles” is the key phrase in the question stem. A dilated heart is built from eccentric
hypertrophy, or in-line (or in-series) enlargement of muscle cells/fibers. In contrast, a thick heart of normal size (not dilated) is built
from concentric hypertrophy, or parallel building and enlargement of muscle cells. This is a key concept and is repeatedly tested on
these standardized tests. Eccentric hypertrophy is also an overworked heart, which can happen in an athlete and lead to a
pathologically high EF (normal EF is 55-65%).

3 ©Test Pirates, LLC. All Rights Reserved.


Step 1 Practice Test #18 Explanations


Why the others are wrong: Congestive cardiomyopathy would have a decreased EF. Diastolic dysfunction is incorrect because
although in diastolic dysfunction there is preserved EF with impaired relaxation, the ventricles are not dilated but usually
hypertrophied and thicker; the same reason why also hypertrophic cardiomyopathy and increased myocardial stiffness is incorrect –
both may have a normal to increased EF, but the ventricles would be thick and not necessarily dilated.

Take home point: Eccentric hypertrophy results in dilation; concentric hypertrophy results in thickening.

3. During a clinical study of calcium and phosphorus metabolism, a 62-year-old man undergoes series of lab studies. His
serum Ca, PO4 and PTH are normal. He is given infusion 1.5 g Calcium chloride over a couple hours. His serum Ca
concentration now is 11.9mg/dl. Compared with pre infusion levels, the serum concentration of which of the following
substances is likely to be increased at this time?
- 24,25-Dihydroxycholecalciferol


Why it’s the right answer: First off, it is important to know the sequence of vitamin D metabolism for the test because this concept
incorporates not only physiology, biochemistry, and nutrition, but also high-yield diseases like kidney disease (problem with alpha-1-
hydroxylase), Crohn’s colitis (issue with vitD absorption), and sarcoidosis (elevated Ca++). So let’s take a gander:

1) So we start with Vitamin D3 (AKA cholecalciferol, from diet like fortified dairy products and fish oils; or synthesized from 7-
dehydrocholesterol in the skin when exposed to UV rays) à (via the liver) à
2) 25-hydroxyvitamin D3 à(via the kidney) à
3)
(a) 1,25-dihydroxyvitamin D3 (by enzyme 1-alpha-hydroxylase located in the kidney and gets positive feedback from PTH)

(b) 24,25-dihydroxyvitamin D3 (by enzyme 24-alpha-hydroxylase also in the kidney)

Now, the question is asking about negative feedback. When there is a high level of calcium, there is negative feedback on the
amount of PTH released (because PTH leads to increased calcium). PTH increases the enzymatic activity of 1-alpha-hydroxylase, an
enzyme that converts 25-Vitamin D3 to 1,25-Dihydroxycholecalciferol. Conversely, low PTH does not activate 1-alpha-hydroxylase
when calcium levels are high (such as in the clinical scenario above). So 25-Vitamin D3 is shunted to the other pathway and
converted to 24,25-Dihydroxylase (by another enzyme that is not important for purposes of the boards). The other choices are ruled
out based on the above explanation. This is one of the most difficult questions of the test because it requires that you not only know
the metabolism of vitamin D like the back of your hand, but it also requires that you know the concept of enzyme kinetics, positive
and negative feedback, and specifically where Vitamin D3 goes if it’s not being converted to 1,25-Vitamin D and shunted to the other
pathway.

Take home point: PTH gets positive feedback from low Ca++ and negative feedback from high Ca+. PTH increases Ca++, and does so
in part by increasing the activity of enzyme 1-alpha-hydroxylase, which converts vitamin D3 to 1,25-Vitamin D, the active form of
vitamin D that increases Ca++ absorption from the gut. When vitamin D3 isn’t being converted to 1,25-Vitamin D, it’s being shunted
to its other pathway that forms 24,25-Vitamin D.

For more information, and a great schematic on what I described above regarding vitamin D metabolism, follow this
link: http://www.ncbi.nlm.nih.gov/pmc/articles/PMC2879391/

4. 69-year-old man comes with skin blistering for 5 days. No oral lesions. Physical shows tense bullae in joint folds of
upper and lower extremities. Biopsy shows subepidermal blister formation. Immunofluorescence microscopy shows
antibodies against proteins at the dermal-epidermal junction. Target by antibodies?
- Hemidesmosome

4 ©Test Pirates, LLC. All Rights Reserved.


Step 1 Practice Test #18 Explanations


Why it’s the right answer: This is a pretty basic question on bullous pemphigoid, which is the dx presented in the vignette. In this
autoimmune disease, antibodies are directed against the hemidesmosomes, which link the dermis to the overlying epidermis.

Take home point: Bullous pemphigoid is a disease of the hemidesmosomes and characterized by a linear immunofluorescent
pattern that highlights the dermal and epidermal junction. Conversely, pemphigus vulgaris, the other high-yield autoimmune skin
disease that’s tested on the boards, is a disease of autoantibodies targeting desmosomes, creating a circular immunofluorescence
pattern INTRAdermally.

Bullous pemphigoid: Layer - deep, age - elderly, blisters - tense/firm, oral lesions - rare, Nikolsky's sign - negative,
Immunofluorescence - Basement membrane, target antigen - hemidesmosome, blister content – hemorrhagic (not always)

Pemphigus vulgaris: Layer - superficial, age - middle-age (not always), blisters - flaccid, easily ruptured, oral lesions - common,
Nikolsky's sign - positive, Immunofluorescence - intraepidermal, circular, target antigen - desmosome, blister content - fluid-filled

5. 71-year-old man with abnormal blood pressure. 5 months ago, bp was 120/75. Today bp 180/97. Bruit heard over left
renal artery. CT angiography shows left arterial stenosis. Labs?
- Total peripheral resistance increased, plasma renin activity increased, serum aldosterone concentration increased


Why it’s the right answer: The board examiners love these up and down arrow questions, and they are not something to be
intimidated by because they are usually pretty straight forward as long as you’re able to break down the clinical vignette. Here,
there’s a guy that had good, well-controlled blood pressure 6 months ago, but now has very high blood pressure that is a result of
gradual onset renal artery stenosis. When there is a blockage of one or two renal arteries, the kidney(s) are starved by blood. In
response, the kidneys think blood pressure is low, so they release factors to increase the pressure in the form of the RAAS cascade
(silly little kidneys). One factor/hormone they release is renin (from the juxtaglomerular cells). Renin then causes a cascade of other
hormones to be released, including aldosterone, which increases blood pressure so the kidneys are happy and no longer starved. In
other words, total peripheral resistance (TPR) is increased. The other important thing to remember is the columns do not go in order
necessarily– they should really in fact be reversed to show first increases renin, which increases aldosterone, which increases TPR, so
don’t also be fooled by that.

Take home point: Low kidney perfusion (from things like renal artery stenosis or fibromuscular dysplasia) leads to activation of the
RAAS cascade and an overall increase in BP/TPR.

6. 39-year-old woman with 4-month progressive shortness of breath with exertion. RR 28. Physical shows jvd and
prominent a wave. Lungs clear. Cardiac exam shows loud pulmonic component of S2 and right-sided S4 gallop. Increased
pulmonary expression of what?
- Endothelin-1


Why it’s the right answer: Dx is pulmonary hypertension secondary to pulmonary fibrosis. An important point I want to bring up
here is that the boards don’t always follow the demographic rules we know and love – i.e. a middle-aged woman is the classic
demographic category for idiopathic pulmonary fibrosis. However, the board examiners know that everyone is going to get the
answers right if they put that there, so they try to make the question harder (sometimes) by changing it up and maybe giving you a
40-50-year-old man with pulmonary fibrosis. That’s how, again, they separate the doctors from the medical students. This clinical
vignette is still pretty obvious sans the demographics of the patient based on physical exam alone. And one of the factors that is
upregulated in pulmonary fibrosis is endothelin-1. Another KEY point is that the lungs are the ONLY organ system, that when starved
for blood, vasoconstricts instead of vasodilating. Vasodilation is a normal response to decreased blood flow à to let more blood
flow in!! However, the vessels in the lungs CONSTRICT and mainly do so through endothelin-1. And the reason they constrict?
Answer: To not promote further deoxygenation through a V/Q or shunt defect. The more blood you have coming into the lung
circulation, with less oxygen coming into the lungs themselves, results in a shunt defect, and an overall DECREASE in oxygenation.
5 ©Test Pirates, LLC. All Rights Reserved.
Step 1 Practice Test #18 Explanations

Finally, know the drug to treat this condition, which decreases pulmonary hypertension by antagonizing the endothelin receptor:
Bosentan. It could be on your test…

Take home point: Pulmonary artery hypertension (PAH) results from increase in release of endothelin-1. PAH is a consequent of
pulmonary fibrosis. Bosentan is a medication frequently used to treat PAH through antagonization of the endothelin receptor.

7. 42-year-old woman with 1.5-years of numbness, blanching, and bluish color to ears, fingers and toes after emotional
upset or cold exposure. Vitals and PE normal. Avoid taking which drug?
- Phenylephrine


Why it’s the right answer: Dx is Raynaud’s phenomenon, which is vasospasm of arteries and arterioles, impairing circulation and
resulting in color changes in extremities, commonly the hands. The mechanism of vasoconstriction is mediated by alpha agonists, so
you would absolutely avoid drugs that act as agonists on the alpha receptors. The only drug listed that is an alpha agonist is
phenylephrine, specifically a selective alpha1-adrenergic receptor agonist.

Take home point: Avoid alpha-agonists in Raynaud’s phenomenon because it precipitates vasoconstriction and spasm.

8. 40-year-old woman with 3-day history of blistering lesions on sun-exposed face, arms and hands. Recurrent episodes of
skin lesions over several years. Taking oral contraceptives for 12y. PE shows fluid-filled vesicles and bullae. Labs: AST
increased, ALT increased, total porphyrin increased, urine uroporphyrin III increased. Precursor to uroporphyrin?
- Succinyl CoA


Why it’s the right answer: Dx: porphyria cutanea tarda. This condition is exacerbated by alcohol consumption and oral contraceptive
use, like in this question, and is the most common porphyria. This is a straight you-either-know-it-or-you-don’t question. Remember
that beautiful chain reaction that starts with glycine + succinyl CoA and ends with heme? This should be memorized, including all the
seemingly insignificant intermediates and their respective acting enzymes. This question was cake – easy. They can make it much
harder and as about where in the cell uroporphyrinogen III à coproporphyrinogen III and what enzyme is at play here.

The last point to make is that the liver enzymes are increased. I didn’t know this was necessarily true before answering the question,
and an infamous book doesn’t say it, so this is the point in time I take away something from every question. Don’t ignore things you
don’t know even though you know the answer. Practice questions are gifts, and the gifts should be fully appreciated. That’s how I
scored >260 – by reading all the questions, learning from questions I even thought were easy, and reading all the answer
explanations.

Take home point: Know glycine + succinyl CoA ----> HEME; the MC porphyria is porphyria cutanea tarda, but not always the most
commonly tested.

9. 59-year-old with sudden onset generalized tonic-clonic seizures. Personality change last 4 months; used to be mild
mannered and now verbally abusive. CT shows single mass enhances with contrast in right frontal lobe and crosses to left
hemisphere through corpus callosum. Dx?
- High-grade fibrillary astrocytoma


Why it’s the right answer: Frontal lobe = personality changes, and in adults, the deadliest brain cancer, astrocytoma, tends to go to
the frontal lobe. It also tends to bridge and infiltrate both brain hemispheres. The question stem pretty much says the answer in this
case.
6 ©Test Pirates, LLC. All Rights Reserved.
Step 1 Practice Test #18 Explanations


Take home point: High-grade astrocytoma in adults can present as personality changes when present in the frontal lobe.

10. 2-week-old male newborn with recurrent vomiting after feeding since birth, and eager to feed after vomiting.
Abdominal exam shows firm, mobile mass in epigastrum to right of midline. Dx description?
- Single primary development defect

Why it’s the right answer: Dx: pyloric stenosis. Pyloric stenosis typically presents is the 2 to 6 week period. Compare to duodenal
atresia, which can present within the first 1-2 days of life. Also, pyloric stenosis is typically nonbillous vomiting because the
obstruction is before the ampulla of Vater, where bile enters (vs. duodenal atresia, which is billous vomiting because the obstruction
is after the ampulla of Vater. Patients with pyloric stenosis are typically male and the first born male. Again, it’s quite possible that
the test will give you a 4th-born female in order to throw you off on the real exam – so you must be confident in the other findings.

Finally, the cause is believed to be the result of a congenital malformation in utero that results in anatomic narrowing of the pyloris
and/or incomplete formation of a space that allows gastric content transit from the stomach to the duodenum.

Take home point: Pyloric stenosis is due to a primary congenital malformation. It is a block between the stomach and duodenum
that leads to NONbillous vomiting between 2-6 weeks of age.

11. 42-year-old man with 2-week of red spots on shins, joint pain and fatigue. PE shows purpura over lower extremities.
Liver palpated 5 cm below costal margin. Labs: WBC 10,000, AST 152, ALT 165, hepatitis C virus RNA positive, anti-
hepatitis C virus antibody positive, cryoglobulins positive, complement-4 100 (N=350-600), urine protein 3+, urine RBC
numerous. Hypersensitivity reaction?
- Type III (immune complex-mediated)


Why it’s the right answer: There’s a lot of information in this question and lot to learn. First, know that that skin reaction is called
erythema nodosum and all the diseases that can present with it – i.e. IBD, sarcoid, TB, cat-scratch disease, etc. Basically, this is
hepatitis C infection leading to liver abnormalities and cryoglobulinemia resulting in a nephrotic syndrome (the 4+ protein is grounds
to push this from the category of nephritic syndrome into a nephrotic syndrome). The specific nephrotic syndrome is
membranoproliferative glomerulonephritis, which is a nephritic syndrome but can copresent as a nephrotic syndrome. The
cryoglobuminemia is the result of complement and immunoglobulins directing acting on an antigen (i.e. the kidney), and leading to a
type III immune reaction. This question is great because it combines several organ systems and immunology, and the board
examiners love getting more bang for their buck – or your buck rather!

Take home point: Cryoglobuminemia is a type III hypersensitivity reaction and causes membranoproliferative glomerulonephritis.

12. 51-year-old man with hypertension not compliant with medications. bp 155/105. Cardiac exam shows apical impulse
displaced laterally, loud S2 and S4 gallop. Echo shows thickening of left ventricular wall. Mechanism of change in cardiac
muscle?
- Transcription factor c-Jun: increased, beta-myosin heavy chain: increased, endothelin: increased


Why it’s the right answer: If you don’t know what c-Jun is, first break the question down to determine what is going on and the
diagnosis. This patient has uncontrolled hypertension and, by that statement alone, has hypertrophic cardiomyopathy until proven
otherwise. So what happens in hypertrophic cardiomyopathy? Cardiac muscle cells enlarge, and cell duplication and enlargement
alone relies on transcription factors to increase to aid in DNA replication and translation. In the question, they tell you the c-June is a
transcription factor, so based on the basic reasoning alone we can deduce that c-Jun must be increased. Beta-myosin heavy chain is

7 ©Test Pirates, LLC. All Rights Reserved.


Step 1 Practice Test #18 Explanations

also a building block of muscle cells, and would also be expected to increase in cardiac hypertrophy. Finally, endothelin is a peptide
hormone and a marker of cardiac stress. Hypertrophic cardiomyopathy would surely be the result of this cascade: c-Jun à
upregulates endothelin à increases the production of beta-myosin heavy chain à cardiac hypertrophy

Take home point: In uncontrolled HTN: transcription factor c-Jun à upregulates endothelin à increases the production of beta-
myosin heavy chain à cardiac hypertrophy

13. 65-year-old man with early morning awakening, decreased energy, difficulty concentrating, anhedonia, psychomotor
retardation, depressed mood for 2 months. He had myocardial infarction and nonsustained ventricular tachycardia 3
years ago. Tx?
- Paroxetine


Why it’s the right answer: SIGECAPS! He’s got 5 of the criteria (sleep increase or decrease, decreased energy, change in
concentration, decreased interest in activities (anhedonia), psychomotor retardation) plus depressed mood for at least 2 weeks. The
nuance here is that aside from the 5 criteria, a clinically depressed patient must have either depressed mood or anhedonia. Another
way they’re trying to trick you is throwing in the MI and v-tachy, which for an elderly man of 70-yo may be secondary to stress and
depression and since the v-tach was nonsustained, it was likely just that.

Take home point: Know SIGECAPS, at least 5, for at least 2 weeks, and must include either depressed mood or anhedonia. It often
masquerades as somatic complaints in the elderly.

14. 7-year-old boy with motion sickness. Planning vacation to Australia, and wants diphenhydramine for motion sickness.
Mechanism of action for motion sickness?
- Antagonist at muscarinic-3 (M3) receptors


Why it’s the right answer: They love diphenhydramine on these tests. Reason? It has many different mechanisms of action and can
be used to treat many things. It acts on H1-R as an inverse agonist (probably the most well-known mechanism of action), and for this
effect it’s used to treat allergies and also to sedate (taking it makes you drowsy). It also acts on the muscarinic Ach receptors as an
antagonist. Therefore, it is used to treat nausea, and also can have anti-parkinsonism effects in its action as an anticholinergic.
Finally, it can act as a sodium channel blocker, and it that regard can be used as a local anesthetic.

So diphenhydramine is an antihistamine, but its anti-nausea properties come from its effect on the muscarinic cholinergic receptors,
no on its effect on the histamine receptors.

Take home point: Know the three main mechanisms of action of diphenhydramine – H1-R inverse agonist, antagonist of muscarinic
Ach receptors, and sodium channel blocker.

15. Newborn is found to have a cervical rib. Transformation of seventh cervical segment to thoracic identity. Which is
true of HOX gene alteration?
- Expression of a HOX gene normally expressed only caudal to C7


Why it’s the right answer: Hox or homeobox genes are involved in segmental organization in the craniocaudal direction, and a
mutation in a Hox gene results in segmental disorganization. For example, If there is a mutation in the Hox gene that is responsible
for a leg being the position of a leg, and the mutation now changes it to be in the position of an arm, the leg will be in the position of
the arm. Another example is syn- or polydacyly – these conditions are also the result of a mutation in a Hox gene. For this question,

8 ©Test Pirates, LLC. All Rights Reserved.


Step 1 Practice Test #18 Explanations

there is a cervical rib that was originally of thoracic identity. Therefore, the mutation was to a new location (cervical) that is normally
only further down, or caudal, in the thoracic region. Compare this to other important genes of embryogenesis:
- Wnt-7 gene: controls development in the dorsal-ventral axis (or front to back)
- Sonic hedgehog gene: controls development in the anterior-posterior axis
- FGF gene: controls the lengthening of limbs

Take home point: Hox or Homeobox genes control organization in the craniocaudal direction, and a mutation in these during
embryogenesis results in malpositioning of structures – i.e. ribs, limbs, fingers.

16. 32-year-old man with ulcerative colitis, history of partial colectomy, and improved rectal bleeding and diarrhea since
then. Meds include mesalamine, hydrocodone and acetaminophen. He says, "I've used all my opiate prescription and
stole some from my job." Next step?
- Discuss treatment options and a referral for detoxification program


Why it’s the right answer: On the actual test, the ethical questions often have very long question stems like this one, with lots of
medical terminology, and then you get to the question and answer choices and it’s an ethics question – so sometimes for these very
long question stems, it does help and doesn’t slow you down to quickly glance at the answers before diving into a hefty question
stem like this one. Another important point that this question brings up, is that the general rule for ethics questions is that the
physician should NEVER refer out – the physician should first attempt to address the issue him/herself. But here, this is the
exception because not only does the correct answer also have the physician discuss treatment options with the patient, but also
there is clearly pathological pain addiction that needs referral to a professional detox program.

Take home point: When pathological addiction to pain medications is suspected (especially in patients with high levels of pain like
colitis, sickle cell), the most appropriate course of action is to discuss alternative treatment plans and refer the patient to a
detoxification program.

17. 68-year-old man dies 6 days after myocardial infarction. Gross of heart shown (perforated interventricular wall).
Histology?
- Erythrocytes, cellular debris, macrophages, and early granulation tissue


Why it’s the right answer: This is a very popularly tested topic – the histopathologic evolution of an MI. And in my personal opinion,
the pathology between days 3-7ish is the most commonly tested time point. At around day 3 post-MI, macrophages begin infiltrate
there are of infarction to do their duty – clean up the RBCs and cellular debris. At this point, because the macrophages are chewing
away from old, infarcted, necrotic debris, the heart is at highest risk for perforation – think of the mac’s at chewing away at the
weak point and the heart wall getting thinner. And during the healing process, granulation tissue also starts to fill in the empty space
where new tissue will attempt to regenerate. Another improtatn point to remember is that the perforation can also occur in the
ventricular wall (wherever the dead tissue is located), and this perforation can lead to blood spilling out and filling the pericardial
space, leading to cardiac tamponade. So this question could have also been presented as signs and symptoms of cardiac tamponade
about 3-10 days after the MI.

Take home point: Know the time points of the histopathology of the post-MI period. Day 3-10 or 14 is when the heart is at highest
risk of perforation due to macrophages chewing and cleaning up cellular debris.

18. 15-year-old boy who frequently thinks about sex, daydreams about girls before going to sleep, and masturbates one
to two times daily. After counseling about safe sex, best next step?
- Schedule next routine examination

9 ©Test Pirates, LLC. All Rights Reserved.
Step 1 Practice Test #18 Explanations


Why it’s the right answer: This is a pretty basic question that presents signs that may seem pathological, especially to a female if she
doesn’t know the male experience, and the answer is typically, continue routine medical care. Boys at this age think frequently
about sex and masturbate, almost to a seemingly pathological level, but this is all normal. The question could have also left out the
safe sex part, and that would have been the answer. First talk about safe sex so your patient doesn’t come back with STIs the next
visit, and then proceed with routine care.

Take home point: Males ages 13-19 will frequently think about sex and masturbate, and for these reasons require counseling about
safe sex. This is not an uncommonly testing topic on the USMLE.

19. 21-year-old man comes with cracked lips and peeling sunburned skin. Works as lifeguard. PE shows desquamation of
sunburned skin. Lips are dry and cracked. Petrolatum to lips may reduce lip symptoms by which of following effects of the
compound?
- Barrier

Why it’s the right answer: Dermatology is often tested with pictures – especially the skin cancers. All physicians should know the
appearance of big three – BCC, SCC, and melanoma. Other topics they like for derm are skin protection and management. In this
question, they are asking about the mechanism of action of petrolatum jelly in protecting the skin from the sun – it acts as a barrier.
Other sun blocks limit UVA/B rays, and do not act as barriers. Barrier sun blocks are the only true way to block out all sun rays, and
for that reason are the most effect sun blocks. Another barrier skin cream to be aware of is zinc oxide.

Take home point: Barrier sunscreens include petrolatum and zinc oxide. Another treatment to protect skin from the sun is
sunscreen that blocks UVA/B rays, but are not true barrier creams.

20. 24-year-old woman comes to the physician because of irregular menstrual periods since menarche at the age of 11.
She is 5.2Ft and weighs 90kgs. BMI 36. She is evaluated and a diagnosis of PCOS is made. After explaining the diagnosis,
the physician discusses behavioral changes, including dietary modification and exercise as part of her treatment. Which
of the following will ensure adherence?
- Provide follow ups to monitor progress in attaining her goal


Why it’s the right answer: This question pertains to primary follow-up in someone with a new diagnosis, for which the management
includes dietary and weight changes, and making sure that the patient is meeting goals for weight loss. PCOS is treated with diet
modifications and weight loss, since part of the pathophysiology relates to obesity. In the patient presented here, she is considered
obese based on her BMI. Other treatments for PCOS include low dose OCPs or medroxyprogesterone, spironolactone, and
clomiphene (for female patients who want to get pregnant)

Take home point: important management for patient with PCOS (but not in all) include diet changes and weight loss, especially in
patients who are overweight, obese, or morbidly obese.

21. 27-year-old man in bicycle collision and hits right shoulder forcefully. Unable to flex right elbow with decreased
sensation to pinprick over right lateral forearm. Brachial plexus lesion?
- E (musculocutaneous nerve)


Why it’s the right answer: A very popular anatomy question on USMLE tests is on the brachial plexus, and it is my guess, that it
frequently is accompanied by a diagram that requires the test-taker to identify the lesion. So, in short, know the brachial plexus, the
anatomy in diagram and gross form, like the back of your hand. Knowing this makes these questions easy, and very straightforward.

10 ©Test Pirates, LLC. All Rights Reserved.


Step 1 Practice Test #18 Explanations

Here, this is a musculocutaneous nerve lesion since its motor function controls the biceps, brachialis, coracobrachialis, and is
responsible for flexion of the arm at the elbow. It is also responsible for the sensation of the lateral forearm.

Take home point: Know all the nerves of the brachial plexus and how to identify them in diagram form and gross form for the test.
The musculocutaneous nerve has both motor and sensory function. The motor is control of flexion of the arm at the elbow by
innervating the biceps, brachialis and coracobrachialis; while the sensory portion controls sensation of the lateral forearm.

22. 69-year-old man with 2-year progressive difficulty writing and walking. Pt is stooped and talks slowly. PE shows bland
facial expression, fine resting tremor in both hands, no tremor when moves, walks with difficulty starting and stopping,
cogwheel rigidity. Brain tissue histology shown as well as gross cross sections of midbrain both normal and diseased.
Substance referred to by arrow?
- Alpha-synuclein


Why it’s the right answer: The diagnosis here is Parkinson’s disease, and the symptomatology of PD is remembered by the
mnemonic TRAP = tremor (at rest tremor, slow and “pill rolling”), rigidity (cogwheel rigidity), akinesia (or bradykinesia; otherwise
known as no movement or slow movement), and postural instability. We know the diagnosis, but that’s not the question. The
question is about the histopathology, for which they show you a picture of a Lewy body and for reference, the gross section of the
midbrain with depigmentation of the substantia nigra. But the answer choices don’t include “Lewy body,” which oftentimes the
USMLE does not use acronyms, requiring you to know what a Lewy body exactly is – alpha-synuclein inclusion body that is
INTRACELLULAR.

It’s easy to remember TRAP, but difficult to know what each of these findings is in real life, and the USMLE expects you to take it that
one step further. To be thorough in lieu of efficient, and since Parkinson’s is so frequently tested, let’s go through each
symptom/finding in the question stem and fit it into the TRAP mnemonic: difficulty writing and walking relates to akinesia and
bradykinesia. Difficulty walking may also be from postural instability. A fine, resting tremor at rest is spelled out in the question
stem. Don’t be surprised if on the actual exam the test makers take the opportunity to use some mixed media and show a video clip
of the resting tremor, or another type of tremor for that matter (i.e. intention tremor, essential tremor). Walking with difficulty
starting and stopping relates to akinesia/bradykinesia and postural instability. And finally, they tell you cogwheel rigidity, but again
this could be presented as a video clip, and simply asking what the potential dx is, so know what that looks like – and for purposes of
Step 2 CS, what it feels like!

Take home point: Parkinson’s disease can be diagnosed with help the TRAP mnemonic. Gross anatomy shows depigmentation of the
substantia nigra in the midbrain. Histopathology shows Lewy body’s which are intracellular inclusion bodies of alpha-synuclein.

23. 28-year-old woman with polycystic kidneys and 3-month history of weakness, fatigue, headaches, hypertension, loss
of appetite and itching. Cr 3.8. Labs?
- bicarb (HCO3-) decreased, inorganic phosphorous (PO4) increased, parathyroid hormone increased


Why it’s the right answer: The diagnosis here is autosomal dominant polycystic kidney disease, and the patient is in kidney failure.
Although these patients have normal kidney function during the first few years of life (unlike in autosomal recessive polycystic
kidney disease), their kidney start to fail and as a consequence can first present with very high blood pressure and headaches as a
result. Moreover, the patient here is itchy because of the buildup of phosphate that deposits in the skin and causes irritation. This
question pertains to the labs expected in end stage renal disease (due to ADPKD or some other cause in general). With failed kidneys
(and the reason for the need for dialysis), the labs show decreased bicarb (since the kidney is responsible for reabsorbing bicarb, so
without the reabsorption, a metabolic acidosis results), increased potassium, increased phosphorous, and increased PTH. The
increased PTH relates to something called “renal osteodystrophy,” which is the dysregulation of the PTH/VitD/Calcium/Phos during
renal failure, resulting in bone breakdown.

11 ©Test Pirates, LLC. All Rights Reserved.


Step 1 Practice Test #18 Explanations

Take home point: autosomal dominant polycystic kidney disease results in renal failure, and can present as high blood pressure,
headaches and puruitis (related to hyperphosphatemia). The labs in renal failure are elevated phosphorous, potassium and PTH, and
decreased bicarbonate.

24. 35-year-old woman brought in after 14-foot fall from ladder. PE suggests severe hemorrhage and shock. Xray shows
fracture of left ninth and tenth ribs. Organ injured?
- Spleen


Why it’s the right answer: Blunt trauma often nicks the spleen as a result of its proximity to the lower ribs (and the liver as well for
that matter). This is an anatomy question, asking about the location of the spleen. Another way to ask the question would be to
show a CT scan and to identify the spleen (it’s further back than you think!). Another way to ask about the spleen and its
vulnerability in blunt trauma is a patient who had a motor vehicle accident in the past and now has sepsis due to pneumococcus.
This is a great question because you have to infer that the spleen was lacerated, hemorrhaged, had to be removed in an emergent
splenectomy, and years later, they are vulnerable to all the encapsulated microorganisms, including strep pneumo, and present with
sepsis.

th th
Take home point: Know the anatomy of the abdomen including the location of the spleen under the left 9 and 10 ribs, and the
spleen’s vulnerability to blunt trauma and its propensity to bleed when injured.

25. 31-year-old man comes in 4 hours after injuring arm skiing. Sensation to pinprick absent over lateral aspect of
shoulder. Xray of right shoulder shown (fracture of surgical neck of humerus). Nerve damaged?
- Axillary


Why it’s the right answer: The axillary nerve (C5-6) has motor and sensory function. The motor function is abduction of the arm at
the shoulder by innervation of the deltoid muscle. The sensory function is sensation over the lateral shoulder (over the deltoid
muscle). It is typically damanged when the neck of the humerus is fractured/damanged because the nerve travels directly over the
neck of the humerus.

Take home point: The axillary nerve innervates the deltoid muscle and is responsible for abduction of the arm at the shoulder and
sensation over the lateral aspect of the shoulder.

26. 50-year-old woman with COPD comes with 3 months of progressive shortness of breath. Physical shows JVD, loud
pulmonary component of S2. Pulmonary function tests show FEV1:FVC ratio of 20% and decreased diffusing capacity for
carbon monoxide. Which is decreased in pulmonary vascular smooth muscle?
- Endothelial nitric oxide synthase production


Why it’s the right answer: This patient has advanced COPD with an FEV1:FVC ratio of 20% (in obstructive lung disease like asthma,
COPD, emphysema, the ratio is characteristically low, <80%). With advanced COPD comes advanced airway damage and vessel
damage from airway collapse. With each inhalation and slow exhalation, the airways pathologically collapse and the blood vessels
are damaged, causing pulmonary hypertension. Pulmonary hypertension in this patient is evidenced by JVD and a loud S2
component, the S2 sound from the closure of the pulmonic valve. In terms of nitric oxide (NO), NO has been shown to act as a
vasodilator in several body systems including the pulmonary vasculature. Due to remodeling in lung disease, NO production is
decreased from damage to the endothelium, but also from vasoconstriction in the pulmonary vasculature to shunt blood away from
damaged airways and prevent a V/Q defect. **The lungs are the only organ system that vasoconstrict in response to hypoxemia.**
In patients with pulmonary hypertension secondary to lung disease, NO is actually a treatment because it counters the response of
decreased NO synthase production, and improves pulmonary hypertension by vasodilating the vessels in the lungs. Endothelin
12 ©Test Pirates, LLC. All Rights Reserved.
Step 1 Practice Test #18 Explanations

receptor antagonists are also used to treat pulmonary hypertension – i.e. bosentan – since endothelin produced by pulmonary
endothelial cells causes vasocontraction.

Take home point: In pulmonary hypertension secondary to chronic lung disease (i.e. in COPD), endothelial nitric oxide synthase (and
therefore NO) is decreased. NO administration is used to treat pulmonary hypertension.

27. An experimental animal is created that has a defect in an innate gastrointestinal defense mechanism. Organism is
found to have decreased HCl prod. After 3 months on biopsy gastric fundus and body show decreased mucosal thickness
and hyperplasia of enterochromaffin like cells. This closely resembles?
- Chronic gastritis


Why it’s the right answer: Chronic gastritis is an autoimmune disorder that destroys the parietal cells in the stomach. The parietal
cell produces HCl, therefore, an animal model that has decreased HCl production would closely represent the findings in chronic
gastritis since the pathophysiology is the same. In autoimmune, chronic gastritis, the parietal cells affects are located in the fundus
and body of the stomach (versus chronic gastritis due to H. pyloris, which affects the parietal cells of the antrum). With decreased
production of HCl, the parietal cells atrophy and the thickness of the mucosa decreases. Moreover, the parietal cells receive
stimulation to secrete HCl from histamine secreted by the enterochromaffin-like cells (ECL cells). Therefore, the ECL cells undergo
hyperplasia to attempt to increasingly stimulate the parietal cells. Another point not included in the question is pernicious anemia,
which is also seen in chronic gastritis due to the lack of intrinsic factor secretion from the atrophied parietal cell.

Take home point: Chronic gastritis involves loss of parietal cells in the fundus and stomach body, leading to mucosal atrophy in
these regions of the stomach, and also ECL cell hyperplasia due to the lack of HCl secretion from the parietal cell.

28. 56-year-old woman with 1-week history of daily episodes of severe, lancinating, left-sided facial pain. Pain lasts 15 to
45 seconds and shoots down ear along jawline. Precipitated by chewing. Dx?
- Trigeminal neuralgia

Why it’s the right answer: This is a typical presentation of trigeminal neuralgia and a straightforward, diagnosis question. In
trigeminal neuralgia, the pain is one-sided and usually lasts less than 1 minute. It is characterized by debilitating pain that is
described as “sharp” and “shooting.” It can be precipitated by use of the trigeminal nerve, such as when chewing or brushing teeth
with jaw movement. It is distinguished from cluster headaches by the time course, with cluster headaches lasting more than 15
minutes typically. First-line treatment is typically carbamazepine.

Take home point: Trigeminal neuralgia is unilateral sharp pain that lasts less than 1 minute and can be precipitated by use of the
trigeminal nerve such as in brushing teeth or chewing. This should be distinguished from cluster headaches, which are also one-sided
and can cause face pain, but last longer, usually >15 minutes.

29. 36-year-old homeless man found unconscious. Breath smells of alcohol. Vitals stable. PE shows bronzed skin and
spider angiomata on chest. Labs: hemoglobin 9.8, hematocrit 27%, MCV 115, WBC 8000, platelets 150,000, ferritin 212,
b12 490, folate 22. Blood smear shows hypersegmented neutrophils and 4+ oval macrocytes. Labs?
- Methylmalonic acid: normal, Homocysteine: increased


Why it’s the right answer: The diagnosis here is macrocytic anemia due to folate deficiency. Folate deficiency is distinguished from
B12 deficiency based on labs. Folate deficiency has increased levels of homocysteine and normal levels of methylmalonic acid; while
B12 deficiency has increased levels of both homocysteine and methylmalonic acid. Another common finding in folate deficiency is
hypersegmented PMNs and macro-ovalocytes on blood smear. The cause of folate deficiency is malnutrition (which is common in

13 ©Test Pirates, LLC. All Rights Reserved.


Step 1 Practice Test #18 Explanations

alcoholics, and is the case in the question stem), malabsorption, medications (i.e. MTX), and increased folate use (i.e. during
pregnancy, hemolytic anemia).

Take home point: Macrocytic anemia due to folate deficiency results in elevated levels of homocysteine and normal levels of
methylmalonic acid, while B12 deficiency results in elevated levels of both homocysteine and methylmalonic acid. Both vitamin
deficiencies result in macrocytic (or megaloblastic) anemia.

30. 72-year-old man with creatinine 2.8 due to chronically increased hydrostatic pressure in Bowman space. Cause?
- Benign prostatic hyperplasia


Why it’s the right answer: this patient has kidney injury evidenced by the elevated creatinine. Because we are not given the BUN,
we can’t necessarily tell if its pre-renal, renal or post-renal. But the stem does tell us that that is increased pressure in Bowman’s
space, or the space at the beginning of the renal tubules, implying that something likely post-renal is creating a block and buildup of
pressure and urine. Of the choices, BPH is the most likely cause of post-renal kidney damage.

Take home point: Benign prostatic hyperplasia results in post-renal kidney damage via increased hydrostatic pressure in Bowman’s
space.

31. 68-year-old woman comes to the physician because of increasingly severe pain in the right knee over the past 2
months. She has fallen repeatedly. She has a 15-year history of symptomatic osteitis deformans. She has bowed tibia and
tenderness of the proximal right tibia. an X ray shows a fracture of the proximal tibia with elevated periosteum and
sunburst pattern. an x-ray of lungs shows pulmonary nodules of various sizes. which of the following findings is most
likely on biopsy.
- Pleomorphic neoplastic cells producing new woven bone


Why it’s the right answer: The diagnosis here is osteosarcoma. This is based on her age, our knowledge of osteosarcoma being the
nd
2 most common primary bone malignancy, the appearance on x-ray which is typical of osteosarcoma (elevated periosteum with
sunburst pattern), and repetition of fractures based on the bowing of the tibia and osteitis deformans. They are asking in this
question about the histopathology, so even if you know the diagnosis, you must also know that osteosarcoma on histology is
neoplastic cells producing new women (and unstable) bone. And finally, the sarcoma has already metastasized to its most common
location for mets – the lungs, as evidenced by multiple pulmonary nodules.

Take home point: Osteosarcoma is the second most common primary malignant bone cancer. It often presents with a fracture that
is due to minimal trauma. On X-ray its appearance is elevated periosteum and a sunburst pattern, while histologically it is neoplastic
cells in the background of new woven and weak bone.

32. 33-year-old woman with a long-standing history of pelvic inflammatory disease has surgical resection of a scarred
segment of a fallopian tube. Which of the following inflammatory cells is most likely to be found in the resected
specimen?
- Macrophages


Why it’s the right answer: Chronic inflammation is often characterized by macrophages versus acute inflammation is associated
with a higher proportion of neutrophils, and then later macrophages start to infiltrate. Macrophages also digest cell debris and
scavenge damaged tissue, so in the case of a scarred section of fallopian tube, macrophages will be there to clean up the dead/dying
tissue. Lymphocytes are also found in chronic inflammation.

14 ©Test Pirates, LLC. All Rights Reserved.


Step 1 Practice Test #18 Explanations


Take home point: Macrophages are like scavengers, and are often found in tissue that is chronically inflamed along with
lymphocytes (vs. PMNs in acute inflammation).

33. 72-year-old woman hx of atrial fibrillation with sudden onset severe abdominal pain. Ex-lap shows embolus in
superior mesenteric artery with complete occlusion of middle colic artery. Ischemic changes where?
- Small intestine, ascending colon, and part of the transverse colon


Why it’s the right answer: This is an anatomy question testing your knowledge of the colonic blood supply. The patient has a-fib,
and often in a-fib a clot can be thrown to different organs and cause ischemia (i.e. in the brain, it would cause a stroke). She
happened to throw a clot down the SMA and into one of the SMA’s branches, the middle colic artery. The SMA supplies blood from
the distal duodenum up until the proximal 2/3 of the transverse colon. From there, the IMA takes over down to the upper portion of
the rectum. Knowing that the middle colic is a branch of the SMA, and that the SMA supplies the small intestine and more than the
first half of the large intestine, the answer that fits this anatomy is ischemia to the small intestine, ascending colon and transverse
colon.

Take home point: The SMA supplies blood from the distal duodenum to the proximal 2/3 of the transverse colon. The middle colic is
a branch of the SMA. Ischemia to parts of the SMA will affect these parts of the GI tract.

34. 53-year-old man with alcoholism comes for fever, chills and productive cough of purulent sputum for 2 days. Blood
cultures positive. Gram stain: gram-positive, lancet-shaped diplococci. Vaccine is against which bacterial component?
- Capsular polysaccharide


Why it’s the right answer: The diagnosis here is Streptococcus pneumoniae, presenting in an alcoholic man with fever, chills and
productive cough, who is also now septic with positive blood cultures. S. pneumoniae is specifically a gram positive, lancet-shaped
diplococci, exactly as described in the question. The vaccine against S. pneumoniae targets its capsule made of sugar changes – or a
polysaccharide capsule.

Take home point: The vaccine for Streptococcus Pneumonia is against its polysaccharide capsule.

35. Patient making sexual advances towards physician. Appropriate measure?


- Have a chaperone join them


Why it’s the right answer: This is a common ethics question, even before the MeToo movement. If sexual advancements are made,
the first thing to do is have a chaperone. The same goes for a belligerent patient. The next and last thing to do is end the care of the
patient if the harassment persists and take appropriate action in terms of having law enforcement authorities involved.

Take home point: Sexual advancement or belligerent behavior warrants having a chaperone present as a “first line” step to taking
control of the situation.

36. Cholera toxin catalyzes transfer of ADP-ribose to an arginine residue in stimulatory G protein resulting in inhibition of
GTPase activity. Which increases in cells as a result?
- Concentration of cAMP

15 ©Test Pirates, LLC. All Rights Reserved.


Step 1 Practice Test #18 Explanations


Why it’s the right answer: Vibrio cholera produces a toxin that permanently activates Gs, which leads to an increase in cAMP. The
toxin does this by inhibiting GTPase, and GTPase stops Gs. this question requires a bit of knowledge of enzyme kinetics. Also
remember that anything that ends in –ase is an enzyme that breaks down that substance (i.e. lipase breaks down lipid). GTPase
breaks down GTP so that it cannot activate Gs. Without GTP being broken down, Gs is constantly activated by GTP à which leads to
increase production of cAMP.

Take home point: GTP activates Gs, which increases the production of cAMP. GTPase breaks down GTP so Gs activation stops. The
toxin of vibrio cholera targets GTPase and prevents it from breaking down GTP. Therefore, Gs continues to produce cAMP, leading to
rice-water diarrhea characteristic of Cholera.

37. 9-year-old boy with pain on back and head since he fell off swing. Tender 3.0cm swollen mass over right occiput. CT
shows osteolytic and soft-tissue mass in skull with inward displacement of dura. Biopsy shows sheet-like infiltrate of pale
eosinophilic cells with bean-shaped nuclei. Cells positive for CD1a. Electron microscopy of cells shows Birbeck granules.
Abnormal cell type?
- Langerhans cells


Why it’s the right answer: The diagnosis is Langerhans cell histiocytosis (formerly known as histiocytosis X), a disorder that is part of
a group of disorders called histiocytoses. Histiocyte describes activated dendritic cells and macrophages. Langerhans cells are
dendritic cells that act as antigen-presenting cells in the skin. In a normal state (and pathological state), Langerhans cells contain
Birbeck granules (look like tennis rackets on electron microscopy), while in the immature state, they express CD1a and S-100
markers. Clinically, this disease is an abnormal proliferation of Langerhans cells and can be multisystem or confined to bone lesions.
The patient presented in the question stem has unifocal Langerhans cell histiocytosis since only the bones are affected and there is
an eosinophilic granuloma of the skull. Additionally, it is common for patients to develop a scaly, erythematous rash pronounced in
the intertriginous zones. A more serious form of the disease, also known as Letterer-Siwe disease, typically presents in children <2
years old. It is multifocal and involves multiple systems, presenting as a rapid proliferation of Langerhans cells in multiple tissues.

Take home point: Langerhans cell histiocytosis represents a group of disorders characterized by abnormal proliferation of
Langerhans cells. Patients can present with a wide range of symptoms including lytic bone lesions, eosinophilic granulomas, and a
scaly skin rash. Langerhans cells have Birbeck granules and express CD1a and S-100 cell markers, all of which aid in the diagnosis of a
LC histiocytosis.

38. 55-year-old man 2 day of fever, chills, confusion and memory loss. Returned from Gulf coast where he walked
barefoot. Hx of severe cirrhosis and portal hypertension. T 39.5C (102.9 F), RR 24, bp 92/49. Physical shows early blister
formation on right lower extremity. Blood culture: gram-negative, lactose-fermenting organism. Bug?
- Vibrio vulnificus


Why it’s the right answer: This patient has sepsis as a result of a gram negative bacteria (GNB) that he likely contracted during his
recent trip to the Gulf Coast. We know he is septic because his blood cultures are positive, but other symptoms consistent with
sepsis include fever, confusion, low BP and elevated RR. The fact that the patient presents with a wound infection and was walking
around barefoot in water that could be contaminated points towards only one organism as the cause of sepsis – Vibrio vulnificus.
You may know that food poisoning from eating seafood is most commonly caused by the organism vibrio vulnificus and also vibrio
parahaemolyticus. This question also tests your knowledge of vibrio vulnificus as a cause of wound infections when a patient is
exposed to contaminated water exposure. Due to the patient’s predisposition to infection, his wound infection led to severe sepsis.

Take home point: Vibrio vulnificus can lead to both food poisoning when ingesting shellfish or contaminated water and wound
infections via superficial contact – i.e. walking barefoot in contaminated water.

16 ©Test Pirates, LLC. All Rights Reserved.


Step 1 Practice Test #18 Explanations

39. 30-year-old woman at 28 weeks' gestation is admitted to the hospital because of a 12-hour history of intense uterine
contractions occurring every 6 minutes. Her membranes ruptured 38 hours ago. Her temperature is 39.2°C(102.6°F), and
pulse is 125/min. Physical examination shows tenderness of the uterus. Pelvic examination shows a closed cervix that is
not effaced. The fetal heart rate is 220/min. Which of the following is the primary stimulus for her uterine muscle
contractions?
- Inflamed maternal decidua release of prostaglandin


Why it’s the right answer: We know you haven’t had your OB/GYN rotation, yet this question is asking about the physiology of
uterine contractions and the role of prostaglandins. Prostaglandins cause uterine contractions. During normal menses,
prostaglandins are released to cause shedding of the uterine lining. In medical abortions, prostaglandins are given along with
mifepristone or methotrexate to cause contractions and release the fetus. During induction of labor, misoprostol (a prostaglandin
analogue) is given with oxytocin to enhance or start uterine contractions so that the infant is delivered.

In the case of the patient described above, prostaglandins are being naturally released from inflamed tissue due to an infection. This
patient, by definition, has prolonged premature rupture of the membranes (also known as PPROM). PPROM occurs when a female
has had premature rupture of her membranes for more than 24 hours prior to labor onset. It places the mother and fetus at high
risk for infection. Our patient now has an infection from PPROM evidenced by her fever and the elevated heart rate of both the
mother and fetus. The body’s natural response is to release prostaglandins and deliver the baby to prevent infection spread. Again,
there is a lot of clinical OB/GYN involved in this question, so this shows how the first step of the examination series asks you to use
your basic science knowledge of prostaglandins to extrapolate their clinical use.

Take home point: Prostaglandins both occur naturally and are used as medical treatment to cause uterine contractions. In
prolonged premature rupture of the membranes (PPROM), infection results in tissue inflammation, prostaglandin release, and
premature labor.

40. Child with XXY karyotype, genetic studies showed he received the extra "x" from his father. An error of chromosome
segregation occurred during anaphase at which of the following stages of spermatogenesis in the patient's father?
- Primary spermatocyte


Why it’s the right answer: The order of spermatogenesis is: spermatogonium à primary spermatocyte à secondary spermatocyte
à sperm. This child with Klinefelter’s has to receive an XY from his father, and an X from his mother. During the first meitotic
division (Meiosis I), the cell undergoes primary spermatocyte à secondary spermatocyte division. During the second meitotic
division (Meiosis II), the cell undergoes secondary spermatocyte à sperm division, and each sperm has either an X or a Y. However,
during this case of spermatogenesis in the father, one of the sperm receives XY. The only way this could occur is during Meiosis I. At
the start of normal Meiosis I, as a primary spermatocyte, the cell has two copies of X and two copies of Y, and the resulting
secondary spermatocytes have either two copies of X or two copies of Y. Then during Meiosis II, the final split gives each sperm
either one copy of X or one copy of Y. The only way both an X and Y can be in the same cell is a malfunction occurring during meiosis
I, because afterwards, there’s only one cell with two copies of X or two copies of Y. In this case, the resulting cell would have an XY,
XXY, or XYY.

Take home point: In Klinefelter’s or XXY karyotype, the error in spermatogenesis occurs during Meiosis I, during the transformation
from a primary spermatocyte to a secondary spermatocyte. Malfunction of Meiosis II occurs during the transformation of a
secondary spermatocyte to a sperm.

41. 9-year-old boy with 3 days nosebleeds and easy bruising. Two weeks ago, upper respiratory tract infection. PE
ecchymoses on upper and lower extremities. Labs shows hemoglobin and leukocyte count normal, and decreased platelet

17 ©Test Pirates, LLC. All Rights Reserved.


Step 1 Practice Test #18 Explanations

count. Bone marrow smear shows increased megakaryocytes. Mechanism of dz?


- Antibodies against glycoprotein (Gp) IIb/IIIa complex


Why it’s the right answer: The diagnosis is idiopathic thrombocytopenic purpura (or ITP). ITP often occurs in the setting of a prior
URI or in pregnant women. In the case of URI, the infection caused the patient’s body to form autoantibodies along with the
antibodies against the virus, and these autoantibodies happen to be targeting platelets, which get removed by the immune system
once tagged by the autoantibody. In ITP, the only abnormal lab value is platelets – which is low. And a BM smear will show increased
megakaryocytes since the megakaryocytes proliferate to try to produce more platelets in response to the low platelet count. All of
this in our minds, the question wants to know the mechanism of ITP, which has been shown to be autoantibodies specifically
targeting the glycoprotein IIb/IIIa complex on the platelet.

Take home point: ITP occurs (but not always) in the setting a recent URI or pregnancy, results in low platelets in the blood, increased
megakaryocytes in the bone marrow, and is specifically the production of autoantibodies against the Gp IIb/IIIa complex on the
platelets.

42. 42-year-old man with a 2-month history of a progressive rash on his feet. Rash is not itchy or painful. A photograph of
feet is shown. HIV+. In addition to HAART, which of the following pharmacotherapy is most appropriate?
- Antineoplastic


Why it’s the right answer: The diagnosis is Kaposi’s sarcoma. Knowing what Kaposi’s looks like is very high yield. Do not mistaken its
appearance for bacillary angiomatosis (due to infection by Bartonella hensle). Treatment with HAART alone will not sufficiently treat
Kaposi’s since it is a neoplastic process of the endothelial cells of the skin, mouth, GI tract, and respiratory tract. This question could
have also asked about its causal virus, which is HHV-8.

Take home point: Kaposi’s sarcoma is a painless, non-pruritic rash found in patients with HIV and HHV-8, and is treated with
antineoplastic medication in addition to HAART. Its appearance is high-yield for the purposes of the test and should not be confused
with bacillary angiomatosis due to infection by Bartonella hensle.

43. 51-year-old man with elevated LDL cholesterol is prescribed lovastatin. This treatment would result in which adaptive
responses at the cellular level?
- Increased transcription of HMG-CoA reductase


Why it’s the right answer: This question requires the test-taker to know the mechanism of action of statins. Statins specifically
inhibit the enzyme HMG-CoA reductase to prevent the conversion of HMG-CoA to mevalonate, which is then converted to
cholesterol, in the hepatocyte. The rules of enzyme kinetics and cellular physiology show us that in response to the inhibition of an
enzyme, the cell will respond by producing more HMG-CoA reductase – an adaptive response at the level of transcription.

Statins are so prevalent in medicine today, that it’s important to know their use, their side effects and their mechanism of action. It’s
also high-yield to know the mechanisms of action of all the other lipid-lowering agents, and their specific effect on LDL, HDL and TGs,
because some of the agents affect, for instance, TGs more than others (i.e. fibrates).

Take home point: Statins inhibit the enzyme HMG-CoA reductase, which leads to increased transcription of HMG-CoA reductase and
an increase in the enzyme’s levels.

44. A new antiplatelet agent is developed for the prevention of recurrence of stroke. In a large randomized clinical trial
with equal numbers of men and women, the rates of stroke are lower in patients receiving the new agent than in patients
18 ©Test Pirates, LLC. All Rights Reserved.
Step 1 Practice Test #18 Explanations

receiving the standard treatment. Results are shown:


Recurrent Stroke Rates per 1000 Person-Years
Standard Treatment vs. New Antiplatelet Drug
Women .12 .04
Men .24 .08
Overall .18 .06
Based on these results, which of the following is the relative risk reduction in women?
- 67%


Why it’s the right answer: Not all the stats questions are going to fit into the perfect mold of all the equations you memorized.
Sometimes, if you just logically think through them, you will be able to solve the question without the use of a specific equation.
Here we are looking at the relative risk reduction of the new drug in women – so we can forget about the numbers for the men in
this case. Next, we need to think about the term relative, which implies a ratio, versus absolute, which implies a difference. So in the
case of relative, we perform a division, versus absolute for which we perform a subtraction. Here, the risk reduction is not .04, since
.04 is the rate of stroke compared to the rate of stroke with the standard treatment, .12. So we will not take .04 and divide it by .12,
a common mistake. If the new risk rate of stroke is .04 down from .12, then the risk reduction is .12 minus .04, or .08. And like I said
before, relative implies a division, so the risk reduction compared to the original risk rate, .12, would be .08 divided by .12 = .6666 or
67%. This question could have been made slightly more challenging if they gave you actual numbers instead of already calculating
the stroke rate for you (i.e. listed the number of people with stroke out of the total population for each category).

Take home point: For relative risk reduction, determine the amount of risk reduction first by taking the original risk rate and
subtracting the new risk rate (given by the new therapy). Then take that risk reduction and divide by the original risk rate.

45. 61 yo man is brought to ER 30min after the onset of chest pain and shortness of breath. He had played tennis all day
and he does not remember how much fluid he had consumed. His temperature is 37.0 oC, pulse 124min, respirations
26min and BP 92/55 mmHg. PE shows dry skin and decreased capillary refill. An ECG and evaluation of cardiac enzymes
show no abnormalities. Which of the following findings in the nephron best describes the tubular osmolarity, compared
with serum in this patient?
PT //macula densa //medullary collection duct
- Isotonic, hypotonic, hypertonic (?)


Why it’s the right answer: This question is about the basic physiology of the nephron when it is in a state to conserve water – what
the kidney does best. Let’s start at the beginning of the nephron, Bowman’s capsule and the proximal tubule. At this point, the fluid
in the tubule is isotonic because the fluid just entered from the blood stream and there hasn’t been enough transport yet of
electrons and H20 to make it anymore hypo or hyper tonic yet (or at least that’s how I remember it). Next we move from the PT to
the descending loop of Henle, and as we dive down from the cortex to the medulla, water is removed and the fluid in the tubule
becomes more concentrated. From here, we make a turn and move through the ascending loop of Henle to the distal convoluted
tubule (DCT), during which NaCl is removed more than the H20 amount that was removed in the descending tubule, so now the
tubule fluid is hypotonic. The point at which the ascending loop of Henle meets the DCT is the location of the macula densa. Finally,
as we travel through the DCT and down the collectuing duct back into the kidney medulla, H20 is removed and the fluid becomes
hypertonic. Specifically, the fluid is hypertonic at the medullary portion – whereas at the cortical portion of the collecting duct, the
fluid is isotonic before becoming hypertonic as H20 continues to be removed.

Take home point: Tubule osmolarity through the nephron: Proximal tubule (isotonic) à descending loop of Henle (hypertonic) à
ascending loop of Henle (isotonic à hypotonic) à distal convoluted tubule (hypotonic) à cortical collecting duct (isotonic à
hypertonic) à medullary collecting duct (hypertonic)

19 ©Test Pirates, LLC. All Rights Reserved.


Step 1 Practice Test #18 Explanations

46. Persons are at risk of influenza are vaccination every year because of antigenic variation, which can be drift (minor)
or shift (major). Mechanism of antigenic shift?
- Reassortment


Why it’s the right answer: this is a straight definition question. The two major antigenic changes the influenza virus undergoes is
recombination and reassortment. Recombination is the exchange of genes between two homologous chromosomes. Reassortment
is the exchange of segments in viruses with segmented DNA. Reassortment can occur, for instance, when a person is infected with
two different strains of influenza virus, and the viruses exchange strains between each other to make a completely new strain.
Reassortment is more of a major change, therefore, and leads to influenza pandemics. An example would be H3N2 à H2N2.

Take home point: Antigenic drift is a minor change in the genome of a virus and results from DNA/RNA recombination. Antigenic
shift in a major change in the viral genome and results from DNA/RNA reassortment. Reassortment is responsible for viral
pandemics.

47. 45 yo man with mild emphysema has smoked 1 pack for 55 years sudden headache, myalgia and rising temperature.
A dry cough develops, with chill and chest pain. Examination of gram stain of sputum disclose neutrophils, but bacteria
are not evident. An X-ray of the chest is consistent with severe pneumonia. A culture of sputum is negative, but culture of
bronchoscopy specimen on a highly specialized bacteriologic medium yields gram negative rods. The identity of these
bacteria is confirmed by staining with specific fluorescent antibodies. The patient respond therapy with macrolide
antibiotic. Mechanism?
- Inhalation of aerosol from an environmental source (Legionella)


Why it’s the right answer: The diagnosis is Legionnaires’ disease, which is characterized by severe pneumonia, fever, CNS symptoms
(in this case, a headache), and GI changes (not presented here). This gram negative rod poorly gram stains, which is why no bacteria
were evident on the gram stain initially. The culture medium they allude to in the question stem is charcoal yeast extract with iron
and cysteine. The antigen can be detected in the urine using fluorescent antibodies. Treatment is with macrolide or quinolone. The
mode of transmission is inhalation of aerosolized water contaminated with legionella. The question could have presented a patient
who was exposed to one of those misters found in warm tourist destinations. Another important key fact to know about legionella is
that it can cause hyponatremia and it is a facultative intracellular organism that infects the alveolar macrophages once inside the
lungs.

Take home point: Legionella is a GN rod that can cause Legionnaires’ disease, which presents with severe pneumonia, fever, CNS
changes (headache or confusion), and GI symptoms. Its mode of transmission is unique in that it is breathed in from aerosolized
contaminated water (i.e. water misters).

48. A sexually active 23-year-old woman has vaginal pain with urination. Pelvic examination shows bilateral
vesicoulcerative lesions of introitus. Tx?
- Acyclovir


Why it’s the right answer: The diagnosis is herpes simplex virus-2. This patient is sexually active and at risk for several STI’s (sexually
transmitted infections). Herpes presents as several small vesicles that ulcerate with time and are very painful. The treatment is
acyclovir, but there is no cure and repeated episodes can occur.

Take home point: HSV-2 presents as painful vesicles that ulcer with time and can be treated with acyclovir. There is no cure for HSV-
2 and repeated outbreaks are common.

20 ©Test Pirates, LLC. All Rights Reserved.
Step 1 Practice Test #18 Explanations

49. 3-day-old boy with vomiting after breastfeeding. PE shows lethargy and dry mucous membranes. Labs: Na 140, Cl 91,
K 6.9, HCO3 15, Glucose 45, BUN 26, Cr 0.3, 17-hydroxyprogesterone increased. Enzyme deficiency?
- 21-hydroxylase


Why it’s the right answer: The diagnosis is congenital bilateral adrenal hyperplasia due to 21-hydroxylase deficiency, which is the
most common type of congenital adrenal hyperplasia. These diseases commonly present shortly after birth and all have low cortisol
levels. 21-hydroxylase deficiency also leads to low mineralocorticoids (unlike the other types of adrenal hyperplasia) because 21-
hydroxylase is responsible for converting progesterone to 11-deoxycorticosterone. So, also unique to this deficiency is low blood
pressure (the others present with high BP), lethargy, and volume deficiency due to lack of mineralocorticoids. Also due to lack of
mineralocorticoids, potassium is not excreted by the kidneys and is high, and a hyperkalemic metabolic acidosis results. Over the
long-term, an increase in sex hormones results due to shunting of the hormones towards the sex hormone pathway, and
masculinization results in pseudohermaphroditism in females (pseudo because the females still have XX DNA).

Take home point: 21-hydroxylase deficiency is the most common form of congenital adrenal hyperplasia and results in low
mineralocorticoids (and low BP, lethargy, volume loss, and elevated K+), low cortisol, and elevated sex hormones, which causes
pseudohermaphroditism in females.

50. 5-year-old boy is about undergo an appendectomy. An intravenous catheter needs to be inserted, but the patient is
fearful of being stuck with needle. The most appropriate anesthesia administered by mask to anesthetize this patient
quickly would have which of the following characteristics?
- Low blood solubility


Why it’s the right answer: The question pertains to general anesthetic principles. For a child, it is important to give them anesthesia
that has a fast induction (think of a child fighting anesthesia before going under, so we must quickly induce them) and low potency
(because they’re kids and should have less potent meds). Anesthetics that have these two properties have low blood and lipid (or
fat) solubility because they don’t dissolve in the blood/lipid as much, so there is more gas available for the brain for a fast induction.
Once the gas is removed, it’s quickly eliminated from the body because most of the gas is not dissolved in the blood and lipid (low
potency). Conversely, anesthetic gasses that have high blood and lipid solubility stay in the body longer because they’re trapped in
the fat layers and blood even after the gas is removed, and take time to get eliminated (high potency). They are at a higher
concentration in the blood and fat when first administered so there is less gas available to go to the brain, therefore induction is
slower.

Take home point: If that explanation didn’t make sense, just remember, low blood/lipid solubility equates with fast induction and
low potency of a general anesthetic gas, while high blood/lipid solubility equates with slow induction and high potency.

21 ©Test Pirates, LLC. All Rights Reserved.















Practice Test Explanations
Step 1 Practice Test # 18 – Block 2
Question Total: 50


Step 1 Practice Test #18 Explanations

2 ©Test Pirates, LLC. All Rights Reserved.


Step 1 Practice Test #18 Explanations


Block 2

1. 33-year-old woman g2p2, not felt well since delivery 1 week ago; ongoing fatigue, inability to breast-feed, light-
headedness in upright position. Pregnancy complicated by preeclampsia and required cesarean delivery, after she
required blood transfusions because of hypotension. PE appears lethargic. HR 76, BP 101/49, HR standing 80, BP
standing 82/443. Hb 11.3, hct 32%. Serum hormone concentrations?
Prolactin/ACTH/TSH/Aldosterone
- decreased, decreased, decreased, increased


Why it’s the right answer: This patient has Sheehan’s syndrome causing hypopituitarism. The patient had infarction of the pituitary
gland during labor, which we infer from the complicated cesarean delivery that’s described and required multiple blood transfusions
for hypotension. The presenting finding is typically failure to lactate since prolactin is affected. The other hormones produced by the
pituitary are also affected including FSH, LH, ACTH, TSH. The patient has low levels of ACTH, resulting in low cortisol levels, which are
causing her low BP and lethargy. In response, the adrenals are producing more aldosterone in response to the low blood pressure in
order to raise the BP.

Take home point: Sheehan’s syndrome is infarction of the pituitary gland during profound blood loss during pregnancy and results in
low levels of prolactin, FSH, LH, ACTH, TSH and elevated aldosterone due to low BP.

2. 18-year-old woman with 3-days history of urinary frequency and pain with urination. T 39 C (102.1 F), pulse 135, bp
94/50. Urine grows gram-negative bacteria. Virulence factor for adherence to bladder?
- Fimbriae


Why it’s the right answer: This patient has a urinary tract infection or cystitis (evidenced by urinary frequency and pain with
urination) due to E. coli, a gram negative rod. E. coli is the leading cause of urinary tract infections. The key virulence factor that
leads to cystitis and pyelonephritis is its fimbriae, which allows it to adhere to the urethra and bladder wall to cause an infection.
Another term to be familiar with for fimbriae is pilus. Specifically, the P pili allows it to adhere to the urinary tract and cause
cystitis/pyelonephritis. The F pili allows 2 bacteria to attach during conjugation (otherwise known as the sex (F) pilus).

Take home point: E. coli’s fimbriae (or the P pili) allow for urethra and bladder wall attachment to cause cystitis and pyelonephritis.

3. Man comes to doc for cast removal. Fracture of left humerus that required open reduction, internal fixation, cast
immobilization. Muscle strength is 2/5 with extension of elbow and 1/5 with extension of wrist and fingers. Patient most
likely sustained a fracture at (which location in humerus)?
- Radial groove


Why it’s the right answer: The radial nerve was injured in this scenario where it travels through the radial groove along the humerus
– typically injured when there is a break mid-shaft of the humerus. The radial nerve can also be injured from compression at thaxilla
3 ©Test Pirates, LLC. All Rights Reserved.
Step 1 Practice Test #18 Explanations

from either crutches or a chair (“Saturday night palsy”). The radial nerve innervates the arm and wrist extensors: brachioradialis,
extensors of the wrist and fingers, supinator and triceps = “BEST.” The radial nerve is also responsible for sensation over the
posterior arm, dorsal hand and thumb. Complete palsy of the radial nerve results in wrist drop.

Take home point: The radial nerve is injured in mid-shaft humerus fractures where the nerve travels in the radial groove of the
humerus or compression of the axilla (i.e. from crutches). Injury to the radial nerve results in decreased wrist and arm extension.

4. 39-year-old F with 3-week history of fatigue, bleeding of the gums, and bone pain. Physical examination shows pallor,
hepatosplenomegaly, and ecchymotic lesions over extremities. Labs:
Hb 7.9g/dl
Ht 24%
Leukocytes: 35,000
segm neutr 5%
eosinophils 3%
lymph 6%
mono 4%
promyelo 82%
platelets 22,000
Polymerase chain reaction test shows an mRNA corresponding to the retinoic acid receptor- alfa/promyelocytic leukemia
fusion gene resulting from a reciprocal translocation of chromosomes 15 and 17. Treatment w/ all-trans retinoic acid is
started. In response to the therapy, the fusion protein will most likely attract which of the following proteins to form a
pre-transcriptional complex?
- Histone acetylase


Why it’s the right answer: This patient has acute myelogenous leukemia (AML) based on her findings of anemia, easy bleeding and
bruisability, hepatosplenomegaly, and labs (high promyelocyte count). If a blood smear of this patient was shown, myeloblasts are
characterized by large nuclei with scant cytoplasm, 2-4 large nucleoli in the nuclei, and Auer rods in the cytoplasm (which do not
have to be present to make the diagnosis of AML). They also tell us that the genetic change is a translocation between chromosomes
15 and 17, which is the subtype M3 of AML that responds to treatment with all-trans retinoic acid (ATRA, vitamin A) by inducing
differentiation of myeloblasts. In terms of the mechanism of action of ATRA, it works by causing the dissociation of histone
deacetylase from the DNA and recruits histone acetylase. When histone acetylase is active, is turns genes on and leads to their
transcription and translation. The abnormal fusion protein in AML is a receptor for ATRA, and the interaction between ATRA and the
receptor causes histone acetylase to turn the gene on at the pre-transcription level, leading to transcription, translation, and the
maturation of promyelocytes into mature WBCs.

Take home point: The mechanism of action of all-trans retinoic acid (ATRA) in the treatment of subtype M3 AML causes the
dissociation of histone deacetylase from DNA and recruits histone acetylase, resulting in the maturation of promyelocytes into
mature WBCs.

5. 89-year-old woman who lives with daughter and grandson comes for routine exam with type 2 diabetes and
hypertension but no complaints. She has had 10-lb weight loss since last visit 2 months ago. BMI 17. PE shows multiple
ecchymoses in various stages of healing upper extremities and torso. Physician should ask?
- "Are you safe at home?"


Why it’s the right answer: Elder abuse is a common topic in standardized tests as well as in real life, something a practicing will at
least once experience. For the purposes of standardized tests, any time they give you an elderly patient living with the younger
4 ©Test Pirates, LLC. All Rights Reserved.
Step 1 Practice Test #18 Explanations

relatives, and just a hint of symptomatology, it is elder abuse until proven otherwise and must be ruled out. In this question, they
gave you everything – elderly person living with daughter and grandson, has weight loss with a dramatic BMI, and multiple bruises
(ecchymoses) at various stages of healing, implying that this is occurring over time, and the bruises are not due to a one-time,
accidental fall. The answer here is appropriate because it is a initially approach the topic without coming out and saying, “is
someone hitting you?” The patient may find that abrupt and become defensive.

Take home point: Elder abuse is a commonly tested ethics topic on standardized tests and should be investigated if there are any
signs of abuse such as weight loss or bruises in multiple stages of healing.

6. 20-year-old woman comes 14 hours after ingesting 80 aspirin tablets in suicide attempt. PE shows tachypnea. Labs?
pH/pCO2/HCO3-
- 7.30/14/10


Why it’s the right answer: The diagnosis is a respiratory alkalosis with a late metabolic acidosis. Salicylates (i.e. aspirin) first directly
stimulate the respiratory center in the brain and cause hyperventilation, resulting in an early respiratory alkalosis. The kidneys then
start to compensate, but the salicylate itself also causes an acidosis, resulting in a late metabolic acidosis. The lab values show a low
pCO2 due to the respiratory alkalosis and blowing off of CO2, a low bicarb (HCO3-) as a result of the acidosis, and a pH that is overall
acidotic because it has been 14 hours since the overdose and a metabolic acidosis occurs after the early respiratory alkalosis.

Take home point: Salicylates (i.e. aspirin) cause an early respiratory alkalosis through direct stimulation of the respiratory center in
the brain, and later a metabolic acidosis.

7. 9-day-old male newborn 2-day history red eyes with discharge. Pregnancy uncomplicated but no prenatal care in third
trimester. PE bilateral conjunctival injection with water discharge. Cause?
- Chlamydia trachomatis


Why it’s the right answer: There are two main eye infections in the postnatal period to be worried about in a pregnancy with no
prenatal care – Neisseria gonorrhea and chlamydia trachomatis. The general rule of thumb to follow is that, 1-3 days after birth, it is
more likely to be N. gonorrhea, while after 3 days, about day 4-10, it’s chlamydia. Another hint is that gonorrhea causes a much
more purulent (or pus-infused) infection, while chlamydia causes less purulent, a mucopurulent or even watery discharge along with
conjunctival injection.

Take home point: The causative organism of conjunctivitis in the post-natal period is typically N. gonorrhea if it is 1-3 days post-natal
and the discharge is purulent, while it is more likely chlamydia trachomatis if it is 4-10 days post-natal and the discharge is more
mucopurulent and watery.

8. 13-year-old girl with 3 months of heavy menstrual flow. Menarche at 12 with regular intervals. History of frequent
nosebleeds and easy bruisability. Father has problems with nosebleeds and clotting. PE shows mild gum bleeding and
ecchymoses. Labs: hb 8.7, hct 26.4, platelet 270,000, bleeding time 12, PT 15 (INR 1.5), PTT 55. Dx?
- von Willebrand’s disease


Why it’s the right answer: Typically, this disease presents, when in females, during menstruation since that is the first major
bleeding episode a patient has if they have not had any major surgery in the past. So that demographic is checked off with this
question. They could make this question more difficult on the actual test by presenting a male with a prolonged bleeding episode
like a nosebleed. This is a hereditary disorder (autosomal dominant!), so the dad has it too, but maybe not to the extreme of the
daughter (yes this disease comes in different severities as well). Usually the only abnormal lab value is bleeding time (elevated), but
5 ©Test Pirates, LLC. All Rights Reserved.
Step 1 Practice Test #18 Explanations

in more severe cases, the PTT is also elevated – and in this case the PTT is abnormally high. The reason being that vWF allows
platelets to adhere to the endothelium to form a clot and also protects Factor VIII, of the intrinsic pathway. Here, the patient is also
anemic because she cannot clot and is losing a lot of blood. This question could, again, be made more difficult by asking not only the
treatment (DDAVP/desmopressin), but also the mechanism of action of treatment – releasing the stored vWF in the endothelium.

Take home point: von Willebrand’s disease is a genetic disorder inherited in an autosomal dominant fashion that results in
symptoms of prolonged bleeding, evidenced by increased bleeding time, and in more severe cases increased PTT due to vWF
interaction with factor VIII of the intrinsic pathway.

9. Prevalence of high-grade cervical intraepithelial neoplasia in unscreened population is 4%. Prevalence in population
with negative Pap smear results is as high 0.4%. Prevalence decreases, which also decreases?
- Predictive value of a positive test result


Why it’s the right answer: Prevalence variations affect the positive and negative predictive values. The rule to remember is that
positive predictive value varies directly with changes in prevalence, while negative predictive value varies inversely with prevalence.
In this question, since the prevalence decreases, the positive predictive value will also decrease. This also makes sense, rules aside:
As the prevalence of a disease decreases, the probability of a test predicting that there is disease will be less accurate since there is
less disease – and vice versa for the negative predictive value.

Take home point: Positive predictive value varies directly with changes in prevalence, while negative predictive value varies
inversely with prevalence.

10. 48-year-old man with alcoholism dies of klebsiella pneumonia. Abscess cavities filled with purulent exudate on
autopsy. Pattern of necrosis in lung tissue?
- Liquefaction


Why it’s the right answer: this question is testing your knowledge of pathology, specifically, the difference types of necrosis. The
main types of necrosis to know for the test are coagulative, liquefactive, caseous, fatty, fibrinoid and gangrenous. The most
commonly ones asked will be in that order as well. Coagulative necrosis typically occurs in the setting of an infarction or loss of blood
flow to a tissue. Therefore, coagulative necrosis can be found in the heart after a myocardial infarction or in the kidney if a clot it
thrown to the kidney and part of the renal system loses blood flow. The exception to infarction and coagulative necrosis in an organ
is the brain, which undergoes liquefactive necrosis. Liquefactive necrosis is also the type of necrosis involved in infections that have
led to tissue death (i.e. in an abscess), which is the case in this question. Liquefactive necrosis is also found in chronic pleural
effusions that lead to tissue death.

Take home point: Liquefactive necrosis is tissue death due to brain ischemia, abscess formation (i.e. in an infection), and pleural
effusions.

11. 42-year old man w several episodes of squeezing chest pain gets angiogram... gets IV NE. Question shows a graph of
coronary blood flow with a drop after the NE and then a rise. Which substance causes increased total coronary blood
flow 1-2 mins after NE?
- Adenosine


Why it’s the right answer: Norepinephrine acts on both alpha and beta receptors, with a greater effect on alpha receptors. It is used
to treat hypotension, causing an increase in peripheral vascular resistance to raise blood pressure. It also causes an increase in heart
rate, and with an increased heart rate, there is less time for coronary artery blood fill and a subsequent drop in coronary blood flow.
6 ©Test Pirates, LLC. All Rights Reserved.
Step 1 Practice Test #18 Explanations

In order to increase blood flow, adenosine is given. Adenosine is a potent vasodilator, especially of the coronaries. Its effect is fast
onset and offset, but overall causes an increase in coronary blood flow. It is released in response to decreased coronary blood flow
from the effect of norepinephrine.

Take home point: Adenosine is a potent vasodilator especially of the coronary arteries and increased coronary artery blood flow.

12. 35-year-old woman overdose on drug X and has serum concentration of 32. Drug X follows first-order kinetics. 6
hours later, the serum concentration is 16. Cannot be moved from intensive care unit until concentration 1.1 or lower.
Which is minimum number of hours from first blood sample (32) that patient must remain in icu?
- 30 hours (5 half-lives)


Why it’s the right answer: This question is asking about half lives and the minimum number of half-lives for the concentration to
drop to less than 1.1. They tell you the half-life in the question stem, which is 6 hours since 32/2 = 16. Now we need to determine
how many half-lives it takes to get to less than 1.1. 16/2 = 8. 8/2=4. 4/2=2. 2/2=1. That was a total of 5 half-lives, but the question
wants to know the number of hours. If each half-life is 6 hours and there are a total of 5 half-lives, then that is 6 hours times 5 half-
lives or 30 hours to get to a concentration of 1.

Take home point: Know the concept of half-lives for the purposes of standardized tests. A half-life is the amount of time it takes a
substances concentration to be divided by 2.

13. 5-year-old boy with fatigue and irritability for 1 month. Family visited rural Louisiana 4 months ago and ran around
barefoot. Conjunctivae are pale. Labs shows normal wbc with 17% eosinophils. Stool prep shows parasite egg (picture).
Cause of fatigue?
- Microcytic anemia (Necator americanus/Ancylostoma duodenale)


Why it’s the right answer: The patient has a parasitic infection based on risk factors (barefoot in rural area), fatigue, and
eosinophilia. Specifically, the parasite is a hookwarm (either ancylostoma duodenale or Necator americanus). Another important
thing to note is that the parasite causes anemia by attaching to the intestinal wall and sucking up blood from the intestinal blood
vessels. Therefore, the anemia is a microcytic, hypochromic anemia from iron deficiency anemia due to blood loss.

Take home point: Necator americanus/Ancylostoma duodenale (hookworms) cause a microcytic anemia by attaching to the
intestinal wall and removing blood from intestinal blood vessels.

14. 42 yo F has congestive cardiomyopathy and pitting edema. Her serum urea nitrogen concentration is 27mg/dl and
serum creatinine is 1.9mg/dl. Furosemide therapy is started. After 4 days, labs show:
Na 131
K 4.6
Cl 91
HCO3 29
BUN 87
Creatinine 2.3
Albumin 2.8
Urine specific gravity 1.024, rbc 0 wbc 0 sediment none
Urinary fractional excretion of sodium is less than 1%. Explanation?
- Prerenal azotemia
7 ©Test Pirates, LLC. All Rights Reserved.
Step 1 Practice Test #18 Explanations



Why it’s the right answer: This patient has acute renal failure due to prerenal azotemia as a result of decreased renal blood flow.
The patient has congestive heart failure, and already has a poor pump (the heart) of blood that could starve the kidney, and this is
further exacerbated by starting the furosemide therapy. Furosemide is a diuretic, and will prevent the kidney from conserving water
to maintain total blood flow and renal blood flow. This loss of fluid (which may resolve the pitting edema) leads to decreased renal
blood flow evidenced by the BUN:Cr ratio of >20 and FENA <1%, both values represent an attempt by the kidney to preserve blood
flow.

Take home point: In heart failure or congestive cardiomyopathy, diuretic therapy can lead to acute renal injury from decreased
renal blood flow and prerenal azotemia. Labs in prerenal azotemia include BUN:Cr ratio >20 and FENA <1%.

15. Investigator studying immune response to fungi. Fungi express beta-glucans on cell surface and that triggers innate
immune response. Which tx decreases glucan expression?
- Caspofungin


Why it’s the right answer: The question asks about the anti-fungal whose mechanism of action inhibits beta-glucans on fungi.
Capsofungin specifically inhibits cell wall synthesis by inhibiting the expression of beta-glucans used in the cell wall. Capsofungin is
used to treat invasive aspergillosis and Candida.

Take home point: Capsofungin is an antifungal that acts by inhibiting the synthesis of beta-glucans used in fungi cell walls.

16. 79-year-old woman comes to the physician because of a 6-month history of increased bruising on her forearms. She
appears alert and well nourished. Physical examination shows extensive wrinkling, scaly erythematous patches on the
face, and irregularly shaped brown macules on the face and forearms. There are ecchymoses in various stages of healing
on both forearms; the ecchymoses are more numerous on the right side. Laboratory studies, including a complete blood
count and coagulation studies, are within the reference ranges. She has noticed no bleeding from her gums after
brushing her teeth. Which of the following is the most likely cause of the ecchymoses in this patient?
- Extensive solar elastosis


Why it’s the right answer: The description of extensive wrinkling, scaly erythematous patches is solar elastosis, so we know she has
that, and that is an answer choice (and in this case, the correct answer). But the question asks about the bruising, and that’s what
throws all the test-takers off the course. There may be another cause of the bruising, but nothing other than the solar elastosis
explains another cause of the bruising. Now the question is, can solar elastosis cause ecchymoses? The answer is yes – Bateman’s
purpura, otherwise known as actinic or solar purpura. The weakness in the skin and breakdown of connective tissues in solar
elastosis goes hand-in-hand with increased fragility of blood vessels and superficial, dermis hemorrhage. And the bruises even linger
in the skin longer than usual and leave behind brown pigment that may never clear – these are the “brown macules” described
above. Finally, one arm is effected more than the other because the patient may have sat in a certain window or was a passenger in
a car more than a driver throughout her life, so one arm was effected by the sun more than the other.

Take home point: Extensive solar elastosis is characterized by wrinkling and scaling of the skin, and is associated with increased
bruising and brown macules, known as actinic or solar purpura.


8 ©Test Pirates, LLC. All Rights Reserved.


Step 1 Practice Test #18 Explanations

17. Heart of 80-year-old woman shows concentric enlargement of left ventricle. Micro exam shows enlarged myocardial
cells with large nuclei. Dx that causes this cardiac enlargement?
- Hypertension


Why it’s the right answer: The patient has hypertrophic cardiomyopathy evidenced by concentric hypertrophy. The myocardial cells
are enlarged with large nuclei, which is hypertrophy of the cells themselves. Another important distinguishing finding is that
sarcomeres in hypertrophic cardiomyopathy are added in parallel. Causes of hypertrophic cardiomyopathy include hypertension
(increased peripheral vascular resistance), outflow obstruction, and genetics (autosomal dominant inheritance of beta-myosin heavy
chain mutation). This condition is compared to dilated or congestive cardiomyopathy – which is eccentric hypertrophy, also
enlargement of the myocardial cells themselves but to a lesser extent, and sarcomeres are added in series. Causes of dilated
cardiomyopathy include alcohol abuse, Coxsackie B (causes a viral myocarditis), chronic cocaine use, Chagas’ disease, peripartum
cardiomyopathy – a large proportion have no cause or are idiopathic.

Take home point: Hypertension is one of the leading causes of hypertrophic cardiomyopathy, and is evidenced by concentric
enlargement of the ventricles with sarcomeres added in parallel. The differences between hypertrophic and dilated or congestive
cardiomyopathy is high-yield for the purposes of standardized tests.

18. 42-year-old woman dx with cholelithiasis. She asks why abdominal pain is intermittent and not constant. Pain is
produced when gallbladder contracts against gallstone obstructing cystic duct. Where is hormone released from that
causes gallbladder contraction?
- Enteroendocrine cells of the small intestine


Why it’s the right answer: The question relates to the role and source of various GI hormones, all of which should be memorized
since they are not very difficult to remember, and of course, very high-yield for the test. The hormone that causes contraction of the
gallbladder is cholecystokinin (or CCK), which is released by the enteroendocrine or I cells of the duodenum and jejunum. Think of
CCK as the anti-gastric, pro-duodenum hormone since it also promotes pancreatic secretion, relaxation of the sphincter of Oddi, and
decreases gastric emptying. It is stimulated to be released by the presence of fatty acids and amino acids in the duodenum.

Take home point: Gallbladder contraction is stimulated by the hormone CCK (cholecystokinin), which is released by the
enteroendocrine cells or the I cells of the duodenum and the jejunum. CCK also increases pancreatic secretion, relaxes the sphincter
of Oddi, and decreases gastric emptying.

19. 80-year-old man has poor appetite and lost 30 lb over 2 months. Labs show normochromatic normocytic anemia. X-
ray of chest shows 2-cm perihilar mass. Biopsy shows small cell carcinoma of lung. Which is responsible for weight loss?
- Cytokine effect


Why it’s the right answer: Weight loss or cachexia from lung cancer (and other malignancies as well) is actually a paraneoplastic
effect mediated by cytokines. Cytokines such as TNF-alpha, IL-6, and INF-gamma have been shown to be upregulated in patients
with malignancy and are responsible for cachexia.

Take home point: Cachexia due to cancer is a paraneoplastic syndrome mediated by cytokines (including TNF-alpha, IL-6, INF-
gamma), which are upregulated in patients with malignancies.

20. 40-year-old man burned hands firing pots 2 months ago. He has no pain during or after burn. PE shows mild atrophy
of arms and hands, absence of deep tendon reflexes in upper extremities, and decreased pain and temperature

9 ©Test Pirates, LLC. All Rights Reserved.


Step 1 Practice Test #18 Explanations

sensations in C4 to T1 dermatomes. Touch is preserved. Dx?


- Syringomyelia


Why it’s the right answer: Syringomyelia is formation of a cyst in the central spinal canal, typically in the upper part of the spinal
cord (C8-T1). The cyst is empty space that pushes on the structures of the cord surrounding the central canal, so the closest fibers –
the crossing fibers of the spinothalamic tract – are usually first to be damaged. For this reason, the patient has no pain in his hands
due to damage to the spinothalamic tract that controls temperature and pain sensations. Going further lateral, the cystic
enlargement can damage the anterior horn, which are made of the motor neurons of the upper extremities. Therefore, the patient
also has signs of lower motor neuron lesions – atrophy and absent deep tendon reflexes. Touch is preserved because it is the more
peripherally-located dorsal columns. A final point to make here, is that this question could have been made more difficult by not
asking for the diagnosis, but what could have led to the syringomyelia in the first place; and syringomyelia is associated with both
spinal cord injury and Chiari I malformation.

Take home point: Syringomyelia is the physical formation of a cyst in the upper part of the spinal cord in the very center of the cord
itself. Therefore, symptoms result from loss of neuro function that travels close to the center of the cord: loss of pain and
temperature sensation from the spinothalamic tract crossing fibers, and then signs of lower motor neuron lesion by affecting the
anterior horns.

21. 4-year-old girl with febrile seizure. PE shows nuchal rigidity and bacterial meningitis suspected. LP and immediate abx
therapy planned, but parents unavailable for consent. Next step?
- Initiate the procedure and treatment without consent


Why it’s the right answer: The question addresses emergent procedures and the need for consent. In any case that is an emergency
and a patient is brought to the hospital, treatment should always be initiated even if the patient is able to consent. If someone
comes in with a gunshot wound to the abdomen, and is talking and coherent, you are not going to ask the patient to first sign a
consent before taking them to do an emergent, surgical ex-lap (exploratory laparotomy). The same goes for life-threatening
infections – and bacterial meningitis is a fast-moving, deadly infection that warrants treatment immediately. Now, if the parents
were around, and while the antibiotics were being ordered and the LP being set up, could a consent be completed? Sure. But if they
are not there, don’t go looking for the ethics committee or wasting time with a court order to get the consent – just treat!

Take home point: In emergency cases such as emergency surgeries or fast-moving, life-threatening infections, consent is second to
initiating treatment.

22. 49-year-old with Graves’ disease develops ulcerating pharyngitis after 7 months of propylthiouracil. What's the
underlying cause of ulcerative pharyngeal disease?
- Agranulocytosis


Why it’s the right answer: It is important to know the thyroid drugs – to treat hypothyroid and hyperthyroid – inside and out,
because they are overall high-yield since these diseases are more common than one wound think (at the least, they are definitely
not “Zebra” diseases). Propylthiouracil (or PTU) is used to treat hyperthyroidism by blocking peroxidase and inhibiting both the
organification of iodide and coupling of thyroid hormone. Agranulocytosis is a rare side effect, but life-threatening when it occurs, so
therefore it’s good to identify in real life, and also important to know for testing-taking purposes. Agranulocytosis is a low WBC
count (so low it’s close to zero), and this patient has that due to an ulcerative pharyngeal disease that he/she developed. With
agranulocytosis, the patient is immunocompromised and can develop a myriad of infections similar to that of a patient with HIV.

Take home point: Agranulocytosis is a rare side effect of the hyperthyroid medication, propylthiouracil, and can manifest as a severe
infection due to an immunocompromised state.

10 ©Test Pirates, LLC. All Rights Reserved.


Step 1 Practice Test #18 Explanations

23. 59-year-old man with normal renal functions gets a heart transplantation. One year later, bp 1650/115 and serum
creatinine 2.2. Which drug caused these findings?
- Cyclosporine


Why it’s the right answer: the patient had a heart transplant so he must be on anti-rejection medications. Cyclosporine is a
commonly used anti-rejection medications used in combination with others that unfortunately can cause hypertension and renal
toxicity, both of which this patient has evidenced by his elevated BP and creatinine level. Other side effects include gingival
hyperplasia (along with phenytoin and verapamil), hirsutism, hyperlipidemia, and hyperglycemia.

Take home point: cyclosporine is typically used as part of the medication regimen after an organ transplant to prevent rejection. The
most common side effects of cyclosporine include hypertension and renal toxicity.

24. 45-year-old woman 14 weeks' gestation with uterus consistent size with gestational age. Amniocentesis shows
increased alpha-fetoprotein (AFP). Fetus at greatest risk for which defect?
- Spina bifida


Why it’s the right answer: Spina bifida is incomplete closure of the bony spinal canal without any herniation of the spinal cord. It is
typically associated with a tuft of hair or dimpled skin over the defect and usually occurs in the lumbosacral region. On prenatal
testing, such as amniocentesis, AFP is abnormally elevated because it is released from the fetus, and with a neural tube defect, more
AFP leaks out into the amniotic fluid. AFP is a protein that normally crosses between fetus and mother, but abnormally high levels
indicate birth defects. And because AFP is also produced by the fetus liver, abdominal wall defects can also result in elevated AFP.
Other proteins that are normal in spina bifida, but abnormally elevated or low in other congenital conditions include betaHCG and
estriol. It is important to be able to distinguish between the differences in all these proteins and the main congenital birth defects.

Take home point: Spina bifida shows elevated AFP on prenatal testing since AFP is produced by the fetus and an open neural tube
defect like in spina bifida leads to abnormal leakage of the protein.

25. 30-year-old man infertility, weight lifter and takes anabolic steroids. Mechanism of infertility?
- Suppression of gonadotropins


Why it’s the right answer: Steroids (as well as androgens including testosterone, dihydrotestosterone and androstenedione)
suppress the release of GnRH (or gonadotrophin releasing hormone) from the hypothalamus. Without the release of GnRH, the
anterior pituitary does not release LH or FSH, and decreased LH and FSH leased to a decrease in endogenous testosterone and sperm
production. LH signals to the Leydig cells to secrete testosterone, while FSH signals to the Sertoli cells to support sperm synthesis.

Take home point: Androgens and steroids suppress the release of gonadotropin-releasing hormone, which in turn, decreases the
release of gonadotropins, LH and FSH, which lead to decreased levels of testosterone and impaired sperm production.

26. 43-year-old man with herniated lumbar intervertebral disc. Laminectomy and removal of hernia scheduled. CT scan of
vertebrae shown, which is surgical entrance location into neural canal?
- D (the lamina)

11 ©Test Pirates, LLC. All Rights Reserved.


Step 1 Practice Test #18 Explanations

Why it’s the right answer: Unless you’ve already done a fellowship in spine, or at least a surgery rotation in orthopedic surgery, this
question is little less obvious. Knowing the vertebrae and individual structures that make up vertebrae is a popular topic for the test,
and the anatomy of the vertebrae is very relevant to your future in medicine even if you don’t become a spine surgeon because
nowadays, back pain is a big deal, and laminectomy is often the treatment of choice for herniated discs. That’s all I can say on it –
know the each structure that makes up an individual vertebrae and how to identify them on CT imaging or grossly.

Take home point: In disc herniation surgery, the neural canal is accessed through the lateral, thin bony process on either side of the
spinous process, called the lamina. The surgery is called a laminectomy.

27. 70-year-old man with 3 days of increasingly severe chest pain, shortness of breath, stridor, hoarseness, difficulty
swallowing and nonproductive cough. Long-standing hx of hypertension. Smokes 2 packs of cigarettes for 50 years. T 98
F, pulse 78, rr 14, bp 161/95. PE shows visible pulsation above the manubrium of the sternum and displacement of the
trachea to the right. Murmur second right intercostal space. Dx?
- Aortic aneurysm


Why it’s the right answer: Specifically, this is a thoracic aortic aneurysm due to long-standing hypertension. Smoking also causes
thoracic aortic aneurysms (but more commonly, abdominal aortic aneurysms), so that’s another risk factor for this patient. We know
it is thoracic because of the location of pulsation. Symptoms of cough and difficulty swallowing are due to its location and
compression of the trachea and esophagus. The patient has stridor and hoarseness likely to compression of recurrent laryngeal
nerve, also located around where the aneurysm would present in the thoracic cavity. Remember the other causes of thoracic aortic
aneurysms that are frequently tested – Marfan’s syndrome and tertiary syphilis.

Take home point: Thoracic aortic aneurysms often present with symptoms of shortness of breath, difficulty swallowing, and
hoarseness due to its location near the trachea, esophagus and recurrent laryngeal nerve, respectively. Suspect a thoracic aortic
aneurysm is anyone with a long-standing history of hypertension and several pack years of smoking.

28. 39-year-old with Chagas disease and 1-hour history of moderate chest pain. 3.4 lb weight loss in 1 month. Vitals
stable. Barium swallow shows dilated esophagus with beak-like narrowing at lower esophageal sphincter (LES). Biopsy
shows?
- Loss of neurons in myenteric plexus


Why it’s the right answer: The diagnosis here is already given – Chagas disease. The other diagnosis that is only described –
achalasia, or a narrowing of the lower esophageal sphincter (LES) with a dilated esophagus more proximally due to outflow
obstruction from a tight sphincter – aka “bird’s beak esophagus.” The question asks about the mechanism by which Chagas disease
causes achalasia, and that is specifically the loss of neurons within the myenteric plexus. Even more specifically, the ganglion cells in
the myenteric plexus lose their ability to produce nitric oxide synthase, which normally produces nitric oxide and relaxes the LES.

Take home point: Achalasia from Chagas disease is caused by loss of neurons in the myenteric plexus.

29. 72-year-old woman with 2-day hx of fever, shaking chills, and left flank pain. Dx acute pyelonephritis and treated with
ciprofloxacin. Five days after, sudden onset watery diarrhea and lower abdominal cramps. T 100.8 F, pulse 79, rr 16, bp
125/80. PE moderate tenderness to palpation in lower quadrants especially on right and increased bowel sounds. Stool is
brown and occult blood negative. Next step?
- Test of the stool for Clostridium difficile toxin


12 ©Test Pirates, LLC. All Rights Reserved.
Step 1 Practice Test #18 Explanations

Why it’s the right answer: This is typical scenario for post-antibiotic pseudomembranous colitis due to C. difficile overgrowth. The
diarrhea is watery and not bloody, very foul-smelling, can even be green in color, and is associated with fever, lower abdominal pain,
and bloating. It is especially more common with antibiotics clindamycin and ampicillin. The diagnosis is made by toxins in the stool.
There are two toxins, A and B. Toxin A binds to the intestinal brush border. Toxin B attacks the cytoskeletal structure of the
enterocytes. Treatment is metronidazole or oral vancomycin.

Take home point: Suspect pseudomembranous colitis due to C. difficile overgrowth after any antibiotic therapy, especially
clindamycin or ampicillin. The diarrhea is characteristically watery without blood, foul-smelling, and can be green in color.

30. 35-year-old woman who three weeks ago, underwent oophorectomy for epithelial ovarian cancer. Recommends
adjuvant chemotherapy with paclitaxel. Mechanism?
- Inhibits microtubule disassembly


Why it’s the right answer: Chemo drugs are one of the main categories of drugs to know inside and out for pharmacology for the
test. You will be almost guaranteed a chemo question, and likely a side effect question. Another popular topic is mechanisms, since
they have several unique and novel mechanisms that are easy points to gain if you know the answer. The microtubule inhibitors do
one of two things – prevent assembly or breakdown of the mitotic spindle. Interference with the mitotic spindle prevents cancer
cells from continuing to divide controllably. Vincristine and vinblastine prevent the assembly. Paclitaxel and other taxols prevent the
breakdown. The former are used in solid tumors as well as leukemias and lymphomas, while the latter are used in ovarian and breast
carcinomas – hence the use of paclitaxel in this patient with ovarian cancer.

Take home point: Vincristine and vinblastine prevent the assembly of the mitotic spindle made of microtubules, while paclitaxel and
the other taxols prevent the breakdown.

31. 31-year-old man comes to physician for follow up after appendectomy. There is mild scleral icterus and well healing
surgical incision. Lab values show
Total bilirubin 3.1
Direct: 0.3
Indirect 4 mg/dL
Dx?
- Gilbert


Why it’s the right answer: Gibert’s syndrome is a benign hereditary hyperbilirubinemia with no clinical consequences aside from
mild icterus or jaundice. Stress and fasting increases the occurrence of the syndrome with elevated bilirubin. The mild elevation of
bilirubin is characteristically an elevation of indirect bilirubin or unconjugated bilirubin because patients either have less of the
enzyme UDP-glucuronyl transferase or decreased bilirubin uptake by hepatocytes.

Take home point: Gibert’s syndrome is a mild elevation of bilirubin brought on by stress or fasting and characterized by elevated
indirect bilirubin. It is due to a partial deficiency of UDP-glucuronyl transferase or decreased bilirubin uptake by hepatocytes.

32. 4-year-old boy 6 days after dx with severe chronic diarrhea due to Giardia. Immunological studies show peripheral
leukocytes that express both CD3 and CD4 and fail to express CD40 ligand. Based on this, the immunoglobulin isotype
that predominates has which biological properties?
- Complement activation


13 ©Test Pirates, LLC. All Rights Reserved.
Step 1 Practice Test #18 Explanations

Why it’s the right answer: The diagnosis is hyper-IgM syndrome. This disease is what it says – lots of IgM with the inability to class
switch to other immunoglobulin isotypes – so there are low levels of IgG, IgA, and IgE. It is due to defective or missing CD40L on
helper T cells which allow IgM to “differentiate” further to G/A/E. These patients are at an increased risk for pyogenic infections
early in life, and this patient has Giardia which is typically defended against by IgA immunoglobulin. This question is more difficult
because IgM isn’t in the answer choices. You also have to know that IgM is activator of the classic complement pathway. IgG also
fixes complement, but this disorder is predominated by IgM, not IgG.

Take home point: Hyper-IgM syndrome is due to defective or absent expression of CD40 ligand, resulting in low levels of IgG/A/E,
higher risk for pyogenic infections, and high levels of IgM, a prominent fixator of complement used in the classic complement
pathway.

33. 17-year-old woman got general anesthesia for wisdom teeth extraction. After 4 minutes, gets hypertonicity of
skeletal muscles and increased body temperature. The treatment is a drug that decreases?
- Sarcoplasmic Ca release


Why it’s the right answer: This patient has malignant hyperthermia evidenced by her hypertonicity and increased temperature in
the setting of an inhaled anesthetic and succinylcholine. The treatment is dantrolene, of which the mechanism is preventing the
release of calcium from the sarcoplasmic reticulum of skeletal muscle. Bonus points if you know that dantrolene does this by
inhibiting the opening of the ryanodine receptor. Dantrolene is also used in the treatment of neuroleptic malignant syndrome
(NMS). Another tangential point to make, is that you should know the difference between NMS due to antipsychotic medications
and serotonin syndrome (SS) due to SSRI intoxication. The key difference is that rigidity and hyporeflexia are characteristic of NMS,
while clonus (involuntary muscle contractions) and hyperreflexia are more representative of SS.

Take home point: The treatment for malignant hyperthermia, which is muscle hypertonicity and elevated body temperature during
inhaled anesthetic use, is dandrolene. Dandrolene’s mechanism of action is preventing the release of calcium from the sarcoplasmic
reticulum.

34. 26-year-old nulligravid woman has menses that occur at irregular 25- to 31-day intervals. Height 5 ft 3 in and weighs
180 lb. BMI 34. PE shows mild hirsutism and velvety brown, thickened skin at the base of the neck and around axillae.
Patient has which condition?
- Hyperinsulinemia


Why it’s the right answer: This patient has signs of diabetes type II and the early beginnings if not already of PCOS (polycystic
ovarian syndrome). These conditions go hand-in-hand because of the underlying effect of weight gain, adipose, and
hyperinsulinemia. The velvety brown and thickened skin on the flexor surfaces is representative of acanthosis nigricans, which is
most commonly associated with insulin resistance. Hirsutism and irregular menstrual cycles from PCOS is also associated with insulin
resistance and hyperinsulinemia. Remember insulin resistance means that higher levels of insulin are needed to have an effect,
resulting in hyperinsulinemia.

Take home point: Hyperinsulinemia is common in obese patients since these patients are more prone to metabolic syndrome, type
II diabetes, and PCOS.

35. 7-year-old girl with severe leg pain 2 day after fell off sled. Immigrate to USA from Iceland. Has had 4 bone fractures
since birth. Parents are vegan and don't give dairy products. PE shows tenderness, swelling, and limited range of motion
of left lower extremity. Xray shows fracture of fibula. Pt has vitamin deficiency that affects which of the following?
- Intestinal calcium absorption

14 ©Test Pirates, LLC. All Rights Reserved.
Step 1 Practice Test #18 Explanations


Why it’s the right answer: This patient is at risk for three vitamin deficiencies: B12 due to the lack of animal products in vegans; lack
of calcium due to lack of dairy products – also from being vegan since dairy products are a good (but not the only source) of calcium;
and vitamin D because she lives in Iceland. She also had an injury that doesn’t match the mechanism of injury, meaning that she
“simply” fell off a sled and then suddenly broke her ankle. So we know she is lacking in calcium, vitamin D, or both. The answer
choices lead us to vitamin D, because calcium deficiency doesn’t affect the absorption of calcium – vitamin D affects the absorption
of calcium; you need vitamin D to properly absorb calcium. Based on the answer choices, we realize that the most pertinent piece of
information in the question stem is… Iceland! And you don’t think this is how the actual test is going to be… then you will be very
surprised.

Take home point: Vitamin D is necessary for calcium absorption in the intestine. A deficiency in vitamin D, calcium, or both, results
in decreased bone mass, which may manifest as a higher rate of fracture or a fracture that doesn’t match the mechanism of injury.

36. 38-year-old woman diagnosed with HIV. Two months ago, started antiretroviral therapy with efavirenz,
emtricitabine, and tenofovir. Labs: CD4 350 and undetectable HIV viral load. Two months ago, CD4 count was 150 and
plasma HIV viral load was 6700. Next step?
- Continue efavirenz, emtricitabine, and tenofovir with no changes


Why it’s the right answer: The patient was recently diagnosed with HIV and started appropriate triple therapy. The labs show that
her CD4 count and viral load have responded appropriately – her CD4 count is above 200 and her viral load is undetectable. If the
CD4 count remained low or the viral load was still elevated, consideration of an additional therapy or a modification in treatment s
warranted since this may be a resistant strand of HIV. Another reason to modify the treatment would life-threatening or or other
organ-threatening (such as eye sight) side effects. Currently, this regimen is working for her and she should continue it in order to
maintain disease control.

Take home point: Triple therapy is the first line treatment for HIV and a good response to treatment is a CD4 count >200 and an
undetectable viral load. This therapy needs to be continued for a lifetime. Another medication needs to be added if there are signs
of disease resistance.

37. 29-year-old man with several episodes of dizziness and fainting during the past 4 months. Father and several paternal
uncles died suddenly. PE and lipid studies normal. Angiography shows no coronary artery blockage. ECG shows prolonged
QT. Decreased activity in which of following causes this?
- Outward (delayed) rectifying potassium channel


Why it’s the right answer: The diagnosis is long QT syndrome, which can cause syncope and sudden death due to cardiac
arrythmias. There are many causes of long QT syndrome, and clearly they are pointing towards a genetic cause, as the father and
paternal uncles also suffered from long QT syndrome. In terms of the genetic mutations that have been identified, the most
common genetic cause of long QT syndrome is type I, or Jervell and Lange-Nielson syndrome, an autosomal recessive disorder that is
characterized by long QT phase on ECG and sensorineural hearing loss starting at an early age. Specifically, the mutation (KCNE1 and
KCNQ1) affects the outward-rectifying potassium channel involved in ventricular repolarization (thereby causing a long QT phase).
The mutation also results in disrupted production of endolymph in the cochlea, which causes the hearing loss – another important
mechanism to be aware of that is less high-yield, but surely would gain you some extra points on the average test-taker if it was
asked.

Take home point: The most common genetic long QT syndrome is Jervell and Lange-Nielson syndrome which causes syncope,
cardiac arrhythmias, sudden cardiac death, and sensorineural hearing loss. The pathophysiology of the arrhythmia is decreased
activity of the outward (delayed) rectifying potassium channel involved in ventricular repolarization.

15 ©Test Pirates, LLC. All Rights Reserved.


Step 1 Practice Test #18 Explanations

38. 5-month-old boy with 1-day respiratory distress. Has progressive weakness and difficulty feeding for a month. RR
48/min. PE shows enlarged tongue and generalized hypotonia. Increased lactate, pyruvate, glucose, and uric acid. CXR
shows cardiomegaly. Biopsy of skeletal muscle shows increased glycogen. Enzyme impaired?
- alpha-1,4-Glucosidase


Why it’s the right answer: The diagnosis is Pompe’s disease, or type II glycogen storage disease. all the glycogen storage diseases
result in abnormal accumulation of glycogen, so it’s no wonder that the glycogen was in the skeletal muscle, and in this case, it’s also
in the heart leading to cardiomegaly. Glycogen can also accumulate in the liver. And because glycogen cannot be broken down to be
used as energy, other pathways are utilized to generate energy, leading to an increase in lactate, pyruvate and uric acid. The
defective or deficient enzyme is lysosomal alpha-1,4-glucosidase (also known as acid maltase). The infant form of Pompe’s does not
present as early as the more severe glycogen storage disease, Von Gierke’s disease. moreover, the adult form of Pompe’s does not
involve the heart and presents as gradual onset of weakness of the skeletal muscles.

Take home point: Pompe’s disease, or type II glycogen storage disease, is distinguished from the other glycogen storage diseases by
the profound heart involvement early in the disease (evidenced by cardiomegaly), and is due to a deficiency of lysosomal alpha-1,4-
glucosidase.

39. 62-year-old man with pulmonary embolus. Treated with intravenous heparin. 24 hours later, warfarin added. Day 2,
partial thromboplastin time is 58 seconds (control 26 sec), and prothrombin time is 12 seconds (control 12.1 sec; INR = 1).
Best explanation for normal prothrombin time and INR?
- Long half-life of factor II (prothrombin)


Why it’s the right answer: The approximate half-life of factor II, or prothrombin, is 60 hours. Heparin acts by enhancing the effect of
antithrombin, which inhibits prothrombin conversion to thrombin. Therefore, the PTT is followed when a patient is on heparin since
PTT is a measure of the common and intrinsic pathway. PT/INR measures the common and extrinsic pathways. Therefore, since
factor II is involved in both the extrinsic and intrinsic pathways, the PT/INR is still within normal limits because there is still factor II
around since the time elapsed since starting heparin has been less than 60 hours. The PTT is elevated because it measures more
factors and is less dependent on prothrombin compared to PT/INR measurement.

Take home point: Factor II, or prothrombin, has a long half-life of 60 hours, and this is the reason bridging from heparin to warfarin
should begin early because it will be 2-3 days before the PT/INR begins to increase.

40. An investigator conducts a meta-analysis of three genome-wide association studies of Crohn Disease. The studies
encompassed 3400 cases and 4900 controls, all of European descent. The initial studies identified 12 significant loci with
odds ratios above 1.4 and 1.6; the combined meta-analysis identified an additional 23 loci with odd ratio of 1.2 to 1.4. It
is estimated that the 35 loci identified explain about 10% of the variance in disease risk with 2 loci accounting for 2% of
the variance. Previous studies of twins indicated a 50% heritability rate for Crohn's disease. Which of the following best
explains these results?
- The identified loci account for a relatively small part of the variance


Why it’s the right answer: This type of question becomes more frequent as you move through the several “steps” of the
standardized tests for medicine. The first test will not include many of these types of questions, but it’s important to answer these
questions by carefully reading the stem and choosing the answer that is most appropriate. In this particular question, they want you
to understand what a meta-analysis is, which is a study design that pools the results from multiple studies to make its own
conclusions. Here, they show that in addition to prior studies, their study found more mutations, but these additional mutations only

16 ©Test Pirates, LLC. All Rights Reserved.


Step 1 Practice Test #18 Explanations

account for a small proportion of disease risk. The other studies found that the genes they identified conferred a 50-50 chance of
inheriting Crohn’s, but the new mutations only account for a 10% chance of inheriting Crohn’s – the reason being that the newly
identified mutations only make up a very small proportion of all genetic mutations that can cause Crohn’s.

Take home point: It is important to know the difference between various study designs. Meta-analysis is one such study, and it
combines the results of several prior studies to conclude new results using the combined power from all the other studies included
in the meta-analysis.

41. 8-year-old boy with 4-day history of intermittent vomiting and 2-month hx of progressive clumsiness. Can no longer
ride bicycle and difficulty getting out of car. Funduscopic exam shows bilateral papilledema. Neuro exam shows impaired
upward gaze and pupil response to light. Walks shuffling gait. CT shows enlarged lateral and third ventricles and a 1.8-
cm mass. Location of mass?
- Pineal gland


Why it’s the right answer: This patient has Parinaud syndrome, also known as dorsal midbrain syndrome, which is specifically a
lesion or ischemia at the superior colliculus of the midbrain that causes a paralysis of conjugate vertical gaze. The patient also has
signs of increased intracranial pressure (ICP) evidenced by bilateral papilledema, vomiting, progressive clumsiness, and shuffling gait.
The increased ICP is likely secondary to a mass blocking the flow through the ventricular system at the level of the cerebral aqueduct
of Sylvius, since the lateral ventricles and third ventricle are enlarged and need to flow through this aqueduct to reach the fourth
ventricle. In a young patient with evidence of a mass on CT, the most likely cause is a pinealoma. Other causes of Parinaud syndrome
(without signs of increased ICP) include MS and stroke at the superior colliculus in the midbrain.

Take home point: In a young patient with Parinaud syndrome and evidence of increased intracranial pressure with a mass on CT, the
most likely cause is a pinealoma, which is a benign mass of the pineal gland.

42. 69-year-old woman comes to the physician because of a 2-month history of low back pain. She has hypertension well
controlled with a thiazide diuretic. Her pulse is 130/min, and blood pressure is 139/80 mm Hg. Physical examination
shows tenderness to palpation over the L2-3 vertebrae. Laboratory studies show:
Hemoglobin 13.3 g/dL
Hematocrit 40%
Leukocyte count 10,600/mm3
Segmented neutrophils 65%
Eosinophils 1%
Basophils 1%
Lymphocytes 30%
Monocytes 3%
Platelet count 260,000/mm3
Serum electrophoresis shows a monoclonal spike of IgG kappa. A chest x-ray shows cardiomegaly with bilateral pleural
and pericardial effusions. An x-ray of the spine shows a lytic lesion. Echocardiography shows an echodense thickened left
ventricle and poor diastolic compliance. A photomicrograph of a specimen obtained on myocardial biopsy is shown.
Which of the following is the most likely cause of the cardiac findings in this patient?
- Amyloid infiltration


Why it’s the right answer: This patient has amyloid light chain (AL) amyloidosis, which can occur in the setting of multiple myeloma,
the cause in this case of the patient’s lytic lesions and back pain. The monoclonal spike of IgG kappa is the light chain that is being

17 ©Test Pirates, LLC. All Rights Reserved.


Step 1 Practice Test #18 Explanations

overproduced. Light chains include kappa and gamma. Amyloidosis is simply abnormally folded protein, so several different types of
protein can cause amyloidosis. The amyloidosis in this case has led to diastolic heart failure evidenced by the thickening of ventricle
and inability to relax (during diastole), with a backup of fluid into the lungs leading to pleural and pericardial effusions.

Take home point: Amyloidosis is the abnormal folding of protein in the body and its accumulation that can lead to organ failure.
Amyloidosis, and specifically amyloid light chain (AL) amyloidosis, can be seen concurrently with multiple myeloma due to the
overproduction of IgG and the buildup of protein.

43. 42-year-old woman with joint pain due to rheumatoid arthritis comes for treatment with over-the-counter agents.
Initiate disease-modifying antirheumatic drug (DMARD). Delayed onset of action of DMARD, so physician prescribes
another until DMARD is effective. Drug?
- Prednisone


Why it’s the right answer: High-dose, oral prednisone is often prescribed in the treat autoimmune diseases such as rheumatoid
arthritis. Due to the multiple side effects of high-dose prednisone, it is not appropriate to prescribe it in the long-term. A disease-
modifying antirheumatic drug (or DMARD) should be initiated to help a patient wean down on prednisone and even off it.
Prednisone is also not a DMARD, and therefore may not stop the progression of the disease like a DMARD can. DMARDs can take up
to 6 months to take effect, and therefore it is appropriate to prescribe high-dose prednisone in the short-term to control a patient’s
symptoms before the chosen DMARD begins to take effect. In general, the rule is that if more than 10mg of PO prednisone are
needed to control a patient’s symptoms, then a DMARD should be started or another DMARD should be added to keep the patient
at 10mg or less of prednisone.

Take home point: It is appropriate to treat autoimmune diseases with short-term use of high-dose prednisone to control a patient’s
symptoms while a DMARD is beginning to take effect.

44. 7-year-old boy with persistent disruptive behavior. Teacher says he's in "constant motion" and never completes
assignments. Treated in ER several times because of skateboarding injuries. Tx?
- Methylphenidate


Why it’s the right answer: These findings are all characteristic of a child with attention deficit hyperactivity disorder (ADHD) – being
disruptive in school, short attention span with inability to complete assignments, and overly active with repeated accidents as a
result. Another key finding here is that this behavior occurs in multiple locations – at school and at home. Onset is typically before
age 7 years. The first-line treatment for ADHD is methylphenidate. Other treatments include amphetamines, atomoxetine, and
behavioral interventions.

Take home point: First-line treatment for attention deficit hyperactivity disorder is behavioral interventions and methylphenidate.

45. A transcription factor that activates expression of HMG CoA reductase is identified, this TF is initially synthetized as a
large precursor protein, with 2 membrane spanning domains. The transcriptional domain is released by proteolysis when
the membrane has a decreased cholesterol concentration. The precursor protein is most likely initially produced in which
of the following labeled area on this labeled diagram of a hepatocyte?
- Rough Endoplasmic Reticulum (ER)


Why it’s the right answer: This is a cell biology question and is asking where protein synthesis occurs in the cell. The labeled
structure is the rough endoplasmic reticulum, where ribosomes are located to carry out translation and synthesize proteins. Smooth

18 ©Test Pirates, LLC. All Rights Reserved.


Step 1 Practice Test #18 Explanations

endoplasmic reticulum is endoplasmic reticulum without ribosomes and is where steroids are synthesized and drugs and poisons are
detoxified.

Take home point: The site of protein synthesis in the cell includes both the ribosomes and the rough endoplasmic reticulum
(endoplasmic reticulum with ribosomes).

46. 2-year-old boy with progressive fever and skin lesions during 24 hours. T 102.8 F, pulse 125, RR 22, bp 115/65. PE
shows large, flaccid, bullous lesions over trunk and abdomen. Another finding in pt?
- Positive nares culture for toxin-producing Staphylococcus aureus


Why it’s the right answer: The diagnosis is scalded skin syndrome, most commonly found in children with fever and a skin rash
characterized by flaccid, bullous lesions localized to the trunk and abdomen. The lesions later weep from the skin when the infection
begins to clear. The cause is staph aureus producing a specific toxin – exfoliative toxin. Staph aureus also produces other toxins that
cause specific diseases: toxic shock syndrome toxin produces toxic shock syndrome (fever, rash and shock); enterotoxin causes food
poisoning that has a rapid onset. It’s important to be aware of all the exotoxins, the specific bacteria that produce them, their
effects, and mechanisms for effect.

Take home point: The cause of scalded skin syndrome, a disease more common in children than adults, is an exotoxin called
exfoliative toxin produced by S. aureus.

47. A study is conducted to compare the incidence of myocardial infarction in patients undergoing two different types of
angioplasty or an operative procedure to manage single-vessel coronary artery disease. A total of 2000 patients are
enrolled. Through a chance process, 1000 are assigned to undergo the operative procedure, 500 are assigned to undergo
one type of angioplasty, and 500 are assigned to undergo a second type of angioplasty. All patients are followed for 4
years to determine the incidence of myocardial infarction. Which of the following best describes this study design?
- Randomized clinical trial


Why it’s the right answer: A randomized control trial (RCT) is an experimental study involving people (vs. animals) and involves a
therapeutic intervention – in order to compare either two different types of treatment or a treatment and placebo. RCTs are used to
test the safety and efficacy of new drugs on the market that had already passed animal and human safety studies. A well-designed
RCT is typically a double-blinding study, meaning that the physicians and the patients in the study do not know who is receiving the
treatment or the placebo. A triple-blinding study also includes the statisticians conducting the analysis. Do not confuse RCTs with
cohort studies. Cohort studies do not involve a therapeutic intervention like RCTs, but evaluate exposure or a risk factor that may
lead to a disease. Both RCTs and cohort studies are prospective studies.

Take home point: RCTs are prospective, experimental studies involving humans that evaluate the safety and efficacy of a
therapeutic intervention.

48. 52yo man with possible hypertension. Based on 25 measurements, his average diastolic pressure is 90mmHg, SD is
6mmHg. If only 5 measurements were made rather than 25, which of the following statements would best describe the
width of the 95% CI with regard the mean blood pressure?
- Larger


Why it’s the right answer: This statistics question is making sure you understand the meaning of 95% confidence interval. In
layperson’s terms, if you have many measurements (or a large n, or sample size), then you can say with 95% confidence that the
19 ©Test Pirates, LLC. All Rights Reserved.
Step 1 Practice Test #18 Explanations

patient’s blood pressure falls within a certain range; however, if you have only a few measurements (or a small n), then you can say
with 95% confidence that the patient’s blood pressure falls within a larger range. You are still 95% confident, but to be 95%
confident you had to make the range larger for which the patient’s blood pressure falls.

If we think about this in terms of standardized deviations (SD), the 95% confidence interval falls within 2 standardized deviations of
the mean if the data is normally distributed (or follows a bell-shaped [aka Gaussian] curve). The SD is 6 with 25 measurements. With
fewer measurements, the SD goes up, so the 95% confidence interval of 2 SDs is larger.

Take home point: The fewer measurements you have in calculating a mean (smaller n, or smaller sample size), the larger the SD and
the 95% CI.

49. 67-year-old man with 1-month hx of nonproductive cough and 5-months shortness of breath. Bilateral end-
inspiratory crackles heard. CT chest shows diffuse reticular opacities at periphery and bases of lungs. Micro exam of
biopsy shows patchy interstitial fibrosis, several fibroblastic foci and no granulomas. Dx?
- Idiopathic pulmonary fibrosis


Why it’s the right answer: Idiopathic pulmonary fibrosis (IPF) is characterized by reticular opacities on CT – reticular is the key word
here, and is synonymous with pulmonary fibrosis. Another key hint is the distribution of the opacities – at the base of the lungs and
in the periphery. This finding is also characteristic of IPF. The time course if very typical of IPF as well – it is usually a slow and
progressive onset over several months. IPF is due to increased collagen deposition, so fibrosis is seen on biopsy without evidence of
granulomas, which would be more consistent with sarcoidosis. A final point to make regarding IPF is that there is sometimes an
underlying cause (such as an allergy or being around a bird/animal) that once identified can halt the lung damage and reverse the
disease process. Severe IPF has a high mortality rate.

Take home point: Idiopathic pulmonary fibrosis is a slowly progressive disease with a high mortality rate. It is due to collagen
deposition and fibrosis. CT shows characteristic reticular opacities concentrated at the lung bases and periphery.

50. 68-year-ld man with cancer of cecum found to have metastatic lesion to liver. Venous route of mets?
- Ileocolic --> superior mesenteric --> portal --> right hepatic branch of portal


Why it’s the right answer: This is a high-yield question on venous anatomy because it is very clinically relevant in regard to the
metastasis of colon cancer, the second and third most common cancer in women and men, respectively. This is either a know it or
you don’t type of question. The cecum is drained by the ileocolic vein which drains into the superior mesenteric vein (SMV). From
there, the SMV drains into the portal vein of the liver, which then drains into the right hepatic branch. It’s important to know that
the portal vein drains blood from the SMV, the inferior mesenteric vein (IMV), and the splenic vein. It’s also key to know that the
SMV receives its supply from the large and small intestine while the IMV receives its supply from the just the large intestine (from
the left colic vein and distal to that).

Take home point: The superior mesenteric vein receives blood from the small intestine and part of the large intestine and ultimately
drains into the portal vein of the liver. The cecum specifically drains into the ileocolic vein, which drains into the SMV.

20 ©Test Pirates, LLC. All Rights Reserved.















Practice Test Explanations
Step 1 Practice Test # 18 – Block 3
Question Total: 50


Step 1 Practice Test #18 Explanations

2 ©Test Pirates, LLC. All Rights Reserved.


Step 1 Practice Test #18 Explanations


Block 3

1. 59-year-old man for routine health examination. Has had normal blood pressure measurements. BP today
165/99mmHg. Physical examination shows no other abnormalities. Serum show hypokalemia and metabolic alkalosis.
Plasma renin activity and serum aldosterone concentrations are increased. Following the administration of captopril,
there is a marked increase in plasma renin activity. Which of the following is the most likely cause of the findings in this
patient?
- Renal artery stenosis


Why it’s the right answer: This question addresses the physiology (and pathophysiology) or the renin-angiotensin-aldosterone
system and concept of positive and negative feedback. The patient previously had normal BP, but it is now elevated with elevated
renin and aldosterone. Therefore, we know that the signal to increase BP is coming from renin and aldosterone, but what is causing
the renin to elevate? Renal artery stenosis in this case is starving the kidney of blood, so renin is released from the kidney. Renin
promotes the conversion of angiotensinogen to angiotensin I, and angiotensin I is converted to angiotensin II in the lung.
Angiotensin II then has a number of effect to promote increased blood flow and blood volume, and one way is through an increase
in aldosterone. Aldosterone promotes Na+ absorption and K+ excretion. Aldosterone also promotes the excretion of H+, for which
every H+ excreted, bicarb is absorbed à this explains the hypokalemia metabolic alkalosis. Finally, an ACE-I captopril increased in
renin because now the angiotensin II has been shut off and BP will drop again, so renin receives feedback to further increase and
help promote increased blood flow to the kidney.

Take home point: Renin is released from the kidney in response to renal artery stenosis. Renin increases the amount of angiotensin
II and increases aldosterone, which can cause a hypokalemic metabolic alkalosis.

2. 9-month-old girl is brought to the physician by her mother because of a 2-week history of poor feeding chronic
diarrhea and pale foul smelling stools. Mother says the symptoms began with colicky abdominal pain following
introduction of solid food to the infant's diet. History of gluten sensitivity. 70th percentile for length and 30th percentile
for weight. Which cell is dysfunctional?
- Enterocyte


Why it’s the right answer: The diagnosis is celiac sprue, which is a dysfunction of enterocyte secondary to an autoimmune response.
The patient has typical signs and symptoms celiac disease, notably foul-smelling stool due to steatorrhea characteristic of celiac and
also colicky abdominal pain after the introduction of solid food, which likely has some gluten in it. Aside being on the lower end of
the weight scale, there are no other abnormalities, so this is less likely something more severe such as a glycogen storage disease.
Blunting of villi and crypt hyperplasia is a key finding on biopsy of the small intestine.

Take home point: Celiac disease can present early in life in infants once solid food is introduced as foul smelling stool and diarrhea
from steatorrhea. Celiac disease affects the GI tract at the level of the enterocyte with blunting of the villi and crypt hyperplasia.

3 ©Test Pirates, LLC. All Rights Reserved.


Step 1 Practice Test #18 Explanations

3. 30-year-old woman comes for counseling prior to conception. Has seizure disorder on valproic acid. Fetus at greatest
risk for drug-related adverse effect during which pregnancy stage?
- 3 to 8 weeks


Why it’s the right answer: Valproic acid is highly teratogenic due to its depletion of sufficient folic acid for the fetus, leading to
neural tube defects. The greatest risk occurs early in pregnancy in the first trimester, specifically weeks 3-8 because this it eh period
of organogenesis. Before 3 weeks, taking valproic acid will likely lead to an early failure of conception, and taking it after 8 weeks will
lead to impaired growth and fetal function to a lesser degree than prior to 8 weeks.

Take home point: Teratogens place the fetus at the highest risk for birth defects between 3 to 8 weeks of pregnancy, which is when
the most organogenesis occurs.

4. Study designed to test the effectiveness of a new drug in the treatment of endometriosis, 200 women randomly
assigned to one of two groups. 98 of women receive new drug, 102 receive standard therapy. The primary purpose of this
method of assigning patients to different groups is to create which of the following ?
- Two groups with similar underlying characteristics


Why it’s the right answer: Randomizing patients to either the new treatment or the standard treatment is done to improve study
quality of a randomized control trial. Putting an equal number of patients in each group randomly will control for differences and
ensure the two groups have similar baseline characteristics. Randomization though risks that, by chance, patients with one particular
characteristic were assigned to one group more than the other. For this reason, statistical analysis is done at the end of the study to
search for confounding factors and control for these.

Take home point: Randomization of patients into study groups in randomized control trials is done to improve study quality and
control for differences in baseline characteristics between the two groups.

5. 68-year-old woman with 4 hour Hx fever and shaking chills. 2 hours ago, she took 325 mg aspirin tablets. Temp 39.5 C,
pulse 98/m, respiration 16/m and blood pressure 100/62 mmHg. Physical examination shows marked tenderness
bilaterally in the costovertebral areas. Lab studies show:
Hb 14 g/dl
Ht 40%
Leukocyte count 33.000/mm3
Urine ph 6.1
Nitrites 3+
Leukocyte esterase 3+
A photograph of wright stained peripheral blood smear, the most likely cause this patient leukocytosis ?
- Leukemoid reaction


Why it’s the right answer: The diagnosis is pyelonephritis based on the patient’s symptoms of CVA tenderness, fever, and urine
analysis signifying infection. The extremely elevated WBC is not due to an acute or chronic leukemia, but due to leukemoid reaction,
as this is an acute inflammatory response to infection. The peripheral blood smear would likely demonstrate immature leukocytes or
bands due to a left shift and the fast production of WBCs and neutrophils. The key distinguishing lab value to differentiate leukemoid
reaction from CML, which can have very similar lab findings, is leukocyte alkaline phosphatase, which is increased in leukemoid
reaction and decreased in CML.

4 ©Test Pirates, LLC. All Rights Reserved.
Step 1 Practice Test #18 Explanations

Take home point: Leukemoid reaction is an acute leukocytosis in response to infection, and is characterized by the presence of
bands, immatures leukocytes, and elevated leukocyte alkaline phosphatase.

6. 70-year-old man with a 8-month history of shortness of breath and swelling of his feet, family history of cardiovascular
disease. He smoked 2 packs of cigarettes daily for 50 years. Pulse 78/min, rr 22/min, BP 155/85. PE: 2+pitting edema of
lower extremities. Diffuse, scattered wheezes are heard bilaterally on auscultation of the chest. Grade 3/6 pansystolic
mumur heard best at lower left sternal border, which increases on inspiration. Maximal impulse palpated in sub-xiphoid
area. S1 and S2 sounds are distant. Liver span 15 cm. Diagnosis?
- Cor pulmonale


Why it’s the right answer: Cor pulmonale is right heart failure secondary to left heart failure. Backup of fluid from the left side of the
heart, through the lungs, into the right heart causes overload and right sided failure. This patient likely started with left heart failure
due to a family history of CVD and extensive smoking history (100 pack years!). The patient pulmonary edema (bilateral, diffuse
scattered wheezes) due to left heart failure. There is evidence of right heart involvement as the pansystolic murmur that increases
with inspiration is tricuspid regurgitation. Mitral regurgitation is best heard on expiration. Right heart failure is also evidenced by
pitting edema and hepatomegaly (“nutmeg” liver on pathology; also known as cardiac cirrhosis).

Take home point: Cor pulmonale is right heart failure secondary to left heart failure with signs of right and left-sided heart failure,
and in more severe cases, liver involvement.

7. 52-year-old woman with gradual onset back pain past 3 weeks. No trauma. Doesn't smoke cigarettes, drink alcohol or
use drugs. Hemogram, serologic studies and urinalysis unremarkable. Xray of spine shows two lytic lesions, in T-9 and L-2.
Dx?
- Metastatic carcinoma of the breast


Why it’s the right answer: This question is truly an epidemiology question since other answer choices can cause lytic bone lesions.
Breast cancer is the most common cancer in women, and more likely to metastasize to the bone than other cancers listed, so for
that reason, the lytic lesions in a patient with no other predisposing history is due to undiagnosed breast cancer.

Take home point: Common things are common, and breast cancer is the most common cancer in women and can present as lytic
bone lesions after the cancer has already metastasized.

8. 17-year-old boy brought by mother with concern that puberty is delayed. When mother leaves room, patient states,
"I'm fine. I don't know what's the matter with her. She wants me to be tall like my dad." Patient is 170 cm (5 ft 7 in) tall
and weighs 71 kg (158 lbs); BMI 25. Sex development is Tanner stage 4. Most appropriate next statement to mother?
- "Tell me more about your concerns about your son's height."


Why it’s the right answer: Boys often mature later than girls, and it is not uncommon for their height to increase much later into
their late teens. However, this patient has met all milestones with a height and weight within normal limits. Sex development is also
within normal limits at a Tanner stage 4 for a 17-year-old, which is enlarged testes and penis with adult type pubic hair that is not
quite to full size or distribution of an adult, or the last Tanner stage, stage 5. The most appropriate next step is to ask an open-ended
question about mom’s concern with her son’s height and then give reassurance.

Take home point: Boys often mature later than girls, therefore the Tanner stages have age ranges for boys that are older than those
for girls. If findings are normal, but parents are still concerned, it’s important to give reassurance.
5 ©Test Pirates, LLC. All Rights Reserved.
Step 1 Practice Test #18 Explanations

9. 33-year-old from group home comes for worsening behavior for 3 weeks. He believes CIA is spying on him through
television set. Reports hearing voices in hall outside and that CIA now plans to kill him. Appears disheveled with unkempt
hair and poor hygiene, difficulty answering questions because listening to internal stimuli. Mental status exam will show
which?
- Flattened affect


Why it’s the right answer: This patient has signs and symptoms of schizophrenia with several positive symptoms. Believing that the
CIA is planning to kill him and hearing voices hallucinations (hearing voices) and delusions (believing people are out to get you). The
difference between hallucinations and delusions is that hallucination are seeing/hearing things that should not be there (i.e. aliens),
and delusions are abnormal perception of reality (i.e. people are plotting against you). Another sign is disorganized behavior – being
unkempt but also having difficulty answering questions because the patient is listening to internal stimuli. Negative symptoms
include flat affect, social withdrawal, lack of motivation, thought blocking, and alogia or lack of speech. It’s not uncommon to
demonstrate both positive and negative symptoms, therefore this patient would most likely demonstrate a flat affect on mental
status exam.

Take home point: Schizophrenia disorder involves both positive symptoms (hallucination, delusions, disorganized behavior,
catatonic behavior) and negative symptoms (flat affect, thought blocking, alogia).

10. 18-year-old man in septic shock unresponsive to ADH (vasopressin). Treatment is discontinued, and high-dose
dopamine in started. Which receptors are stimulated?
- alpha1-adrenoreceptors


Why it’s the right answer: The use of dopamine in septic shock is common because in septic shock, blood pools in the peripheral
vasculature due to the bacterial toxins causing vasodilation. Dopamine acts on both D1 receptors, beta1 and 2 receptors and alpha 1
and 2 receptors. As low doses, dopamine acts on the CNS receptor, D1, only. At medium doses, dopamine acts on beta1 and 2
receptors to promote heart contractility and increase the heart rate. At high doses, dopamine acts on both alpha1 and alpha2
receptors, and for the purposes of shock, causes vasoconstriction via the alpha1 receptors.

Take home point: Dopamine has different effects at various potencies. Though it acts on dopamine, beta, and alpha adrenergic
receptors, at high-doses it acts on alpha1 receptors to cause vasoconstriction and is used to treat septic shock.

11. After operation, 45-year-old patient has lung region that is underventilated but well perfused. Which increases?
- Physiologic shunt


Why it’s the right answer: Knowing V/Q mismatch scenarios is high-yield, and this question is very straightforward in terms of
testing that knowledge. Ventilation is air in the bronchi, bronchioles, and alveoli while perfusion is the blood in lungs surrounding
the alveoli and carrying o2-enriched blood back to the heart. When either is composed, the oxygen in blood goes down, which is
why the lungs are the only organ system with blood vessel vasoconstriction when pO2 goes down (all other organ systems vasodilate
in response to low pO2 in order to increase the delivery of O2). It’s simple to remember if you think about it logically: As air/O2
ventilation goes down and blood perfusion remains stable, a shunt is created because that goes directly from the arterial to venous
side without obtaining O2 from the alveoli. Conversely, when perfusion goes down and ventilation remains the same, dead space is
created, meaning that O2 is in the alveoli, but there is no blood taking away the O2, so it just stays there in a “dead space.” 100% O2
only improves overall oxygenation in the dead space scenario because with less perfusion in one part of the lung, blood is rerouted
to other parts of the lung to carry more O2 away from the alveoli (now with 100% O2) back to the heart. 100% O2 does not improve
oxygenation in the case of a shunt.
6 ©Test Pirates, LLC. All Rights Reserved.
Step 1 Practice Test #18 Explanations


Take home point: A physiological shunt is created when ventilation decreases and perfusion remains constant or high. Dead space is
creased with perfusion decreases and ventilation remains constant or high.

12. 28-year-old man with X-linked recessive disease has deafness, hematuria and progressive renal failure. Protein
abnormality?
- Type IV collagen


Why it’s the right answer: This is Alport syndrome, which has many causative genes, but the most common form is X-linked
recessive. These patients have deafness and progressive nephritis (symptom of hematuria), and occasionally ocular changes
(anterior lenticonus, AKA cone shape to lens in the eye). Type IV collagen is abnormal in these patients. Type IV collage is used in
basement membranes of the eyes (lens capsule), ears, and kidney.

Take home point: Alport syndrome is an X-linked recessive condition that results in a defect in type IV collagen, leading to deafness,
progressive kidney disease, and occasionally vision changes.

13. 65-year-old with chronic pancreatitis is deficient in which enzymes that causes inability to digest triglycerides?
- Colipase


Why it’s the right answer: The pancreas secretes enzymes that digest starch, fat, and protein. The enzymes that digest fat, including
triglycerides, include lipase, phospholipase A, and colipase. In someone with chronic pancreatitis, there is calcified and damaged
pancreas, resulting in deficient enzymes, decreased fat digestion, vitamin malabsorption (because the fat-soluble vitamins need fat
for absorption), and steatorrhea due to fat in the stool. For starch digestion, the pancreas secretes alpha-amylase. For protein, the
pancreas secretes proteases and trypsinogen.

Take home point: The pancreas secretes enzymes, lipase, phospholipase A, and colipase, to digest fat.

14. 74-year-old woman with irregular, raised, multicolored dark lesion on left forearm with frequent sunlight exposure.
Biopsy shows malignant pigmented cells. Worst prognosis with involvement of which layer?
- Subcutaneous tissue


Why it’s the right answer: This patient has malignant melanoma. The worst prognosis would be a lesion with the highest risk of
metastasis, and the risk for metastasis varies directly with the depth of the melanoma. Subcutaneous tissue is the deepest layer
listed, so therefore it has the greatest risk for metastasis and the worst prognosis.

Take home point: Risk for malignant melanoma metastasis and a worse prognosis varies directly with the depth of the melanoma.

15. 35-year-old man dx with epilepsy age 10 years. Most recent generalized tonic-clonic was 6 years ago. Medication was
adjusted. Current meds include carbamazepine. He's never had any collisions while driving his motor vehicle. Patient's
status with respect to driving?
- He is medically qualified to drive

7 ©Test Pirates, LLC. All Rights Reserved.


Step 1 Practice Test #18 Explanations

Why it’s the right answer: Patients with epilepsy are at a risk for motor vehicle accidents due to the risk of a seizure occurrence
during driving. This question asks you to use your judgment in giving the patient driving priviledges. In general, the question will
clearly show signs that the patient is safe to drive, or give warning signs that the patient is unfit to drive – i.e. a recent crash. Epilepsy
in this case is under good control since the last seizure was 6 years ago after which medication was adjusted and the patient has not
had another seizure since then. Reason to not allow the patient to drive would be not taking his seizure medication, having a recent
seizure without follow-up or adjustment to medication, and recent or even remote, multiple vehicle collisions in the past.

Take home point: Physicians often have to make judgment decision regarding a patient’s driving ability and their safety. In patients
with epilepsy, if the disorder is under control, then they can be given driving privileges. The question stem will give clear warning
signs if driving privileges taken away – i.e. recent seizure, motor vehicle crash.

16. 46-year-old woman with 3-month joint pain, muscles aches and fatigue. Had a renal calculus 5 months ago and had
cholecystectomy 2 year ago. Appears fatigue, depressed, there is mucosal pallor. No masses palpated in neck. Labs:
erythrocyte count 2 million, K+ 4.2, Cl- 108, Ca2+ 13.8, phosphorous 2.8, alk phos 124. Technetium-99m scan shows 1.4
cm nodule in neck. Mutation?
- MEN1


Why it’s the right answer: Multiple endocrine neoplasia (MEN) 1 is a disease characterized by endocrine tumors of three organs: the
parathyroid, pituitary, and pancreas. Most of the symptoms in this patient come from the parathyroid tumor found on the
Technetium-99m scan. The tumor is causing hyperparathyroidism, the most common manifestation of MEN 1. The patient has
hypercalcemia and hypophosphatemia with an elevated alkaline phosphatase, all signs of hyperparathyroidism. The elevated
calcium has led to renal stones (calculus) and the patient has mood changes, weakness, and joint pain – all due to
hyperparathyroidism. Elevated calcium can also cause stomach ulcers, which may be the cause of the patient’s anemia secondary to
a bleeding ulcer or iron malabsorption overall. In terms of pituitary tumors in MEN 1, most commonly the tumor secretes prolactin
or growth hormone resulting in amenorrhea, galactorrhea, and infertility. In terms of the pancreatic tumor, the tumor is usually of
endocrine type such as an insulinoma, VIPoma, or glucagonoma, the rarest of the pancreatic tumors. But if you ever see
glucagonoma on the test, don’t miss it – symptoms include a rash described as necrolytic migratory erythema (because it migrates
around your skin!), anorexia, anemia, diarrhea, venous thrombosis, and glossitis.

Take home point: MEN1 is a disease of three types of tumors – parathyroid, pituitary and pancreatic tumors. The most common
manifestation of MEN1 is hyperparathyroidism that causes signs of hypercalcemia including fatigue, joint pain, and renal stones.

17. 32-year-old man just returned from work as worker from Africa begins oral chloroquine therapy for malaria caused
by Plasmodium vivax. His initial therapeutic response is good, but he develops recurrent parasitemia 3 months later.
Which of the following best explains the recurrence ?
- Chloroquine is ineffective on the exoerythrocytic malaria tissue stages


Why it’s the right answer: Malaria due to plasmodium vivax has two phases: one phase involves the liver (aka the exoerythrocytic
phase) and one that involves RBCs (the erythrocytic phase). Plasmodium vivax can remain dormant in the liver for months to years in
as a hypnozoite in the exoerythrocytic phase. In this question, the man was treated successfully with chloroquine for the
erythrocytic malaria. However, chloroquine does not treat the dormant phase – the hypnozoites. After 3 months, the hypnozoites
reactivated and produced merozoites, explaining the recurrence of the disease. Full treatment for Plasmodium vivax would have
been to add primaquine, which treats the hypnozoite form in the liver.

Take home point: Malaria due to plasmodium vivax can remain dormant in liver in the hypnozoite phase or exoerythrocytic phase
for months to years. Treatment with chloroquine does not successfully treat the exoerythrocytic phase, so primaquine should be
added to prevent disease recurrence.

8 ©Test Pirates, LLC. All Rights Reserved.


Step 1 Practice Test #18 Explanations

18. 40-year-old man with HIV infection follow up examination, has been treated with HAART for the past 8 years HIV
plasma viral load has been undetectable. HIV viral load now increase, antiretroviral resistance suspected. HIV genotype
analysis confirms that the virus has resistance mutations, which of the following most likely mutated?
- Reverse transcriptase and protease


Why it’s the right answer: This question is regarding triple therapy with antivirals for HIV. In anyone diagnosed with HIV, three
antiretroviral medications are always begun in order to prevent the development of resistance. However, with the continued
presence of HIV, viral resistance is not uncommon even when three medications are utilized. Typical regimen begins with 2
nucleoside reverse transcriptase inhibitors (NRTIs) plus either 1 non-nucleoside reverse transcriptase inhibitor or 1 protease
inhibitor or 1 integrase inhibitor. Knowing that these are the targets of antiretrovirals, the only answer choice that contains targets
of these medications that likely mutated includes reverse transcriptase and protease. Because the medications were working before,
the mutations must have occurred in these target proteins, and now the virus is able to evade current treament.

Take home point: HAART therapy for HIV is started with three antiretroviral medications. These can include: 2 nucleoside reverse
transcriptase inhibitors (NRTIs) plus either 1 non-nucleoside reverse transcriptase inhibitor or 1 protease inhibitor or 1 integrase
inhibitor. Viral resistance can occur when there are two or more mutations in one of these target proteins.

19. 61-year-old woman with 6-month shortness of breath and chronic nonproductive cough. 1-year history difficulty
swallowing, joint stiffness, diffuse tightening of skin on face, neck, shoulders, arms, fingers. Sensitivity to cold weather,
turn white. Hx of esophageal reflux. Biopsy showed atrophy of epidermis and deposition of collagen throughout dermis
with loss of dermal appendages. PE cutaneous ulceration, claw-like flexion deformity, decreased joint mobility. At risk for
which pulmonary disorder?
- Pulmonary hypertension


Why it’s the right answer: The diagnosis is scleroderma, almost a CREST-like syndrome, however, she doesn’t fulfill the C = calcinosis
and T = telangiectasia parts of CREST. CREST is also more of a benign course with less visceral involvement, and this clinical picture
appears to be more severe. Scleroderma is idiopathic fibrosis and collagen formation throughout the body, and CREST is a more
superficial form of systemic scleroderma. This patient already has lung involvement evidenced by SOB and chronic nonproductive
cough. Pulmonary involvement is the most common cause of death in these patients because the fibrosis involves in the lungs and
when extensive can cause pulmonary hypertension. Skin tightening can be so severe that it results in the disappearance of wrinkles
and a claw-like formation of the hands with inability to fully extends fingers. She also has esophageal dysmotility manifesting as
esophageal reflux, another common finding in these patients. Diffuse scleroderma is associated with slc-70 antibody (anti-DNA
topoisomerase I); CREST is associated with anticentromere antibody.

Take home point: Diffuse scleroderma and CREST are progressive fibrosis and collagen disorders. Scleroderma can lead to lung
fibrosis, which causes pulmonary hypertension. Lung involvement is the most common cause of death in these patients.

20. Workbench wiped down with alcohol, successfully inactivates viruses with which characteristic?
- Enveloped virion


Why it’s the right answer: This question is repeatedly, and I mean repeatedly asked. And here it is again! First, viruses have an
envelope or they don’t – they’re naked!! The ones with the envelope, need that envelope to fuse with a host’s cell membrane and
enter the cell to procreate. Without that envelope, entrance into the cell is impossible and the virus cannot replicate. Alcohol breaks
down the viral enveloped, preventing fusion of the envelope with host’s cell membrane. However, alcohol does nothing for those
buggers that are running naked, wild and free.

9 ©Test Pirates, LLC. All Rights Reserved.


Step 1 Practice Test #18 Explanations

Take home point: Enveloped viruses are more prone to alcohol disinfectant than naked viruses because the alcohol dissolved the
envelope and prevents viral fusion with host cell membrane to replicate.

21. 45-year-old woman pain in left leg 3 days. PE shows deep venous thrombosis. Labs: platelet 220,000, PT 1.12 (INR 1),
PTT 36. Heparin started. Five days later, platelet 100,000. Reason for decreased platelets?
- Drug-related antibodies


Why it’s the right answer: This is heparin-induced thrombocytopenia (HIT), which is the development of IgG antibodies by the host
against heparin-PF4 (platelet factor 4). The antibodies against heparin-PF4 activate platelets and lead to thromboses and decreased
platelets. The time course is as decribed above – usually after 3-5 days of heparin use, HIT can develop since the immune system is
becoming sensitized to the heparin-PF4 complex. The treatment is to stop heparin ASAP, and start a heparin alternative i.e. lepirudin
or bivalirudin, which are direct thrombin inhibitors.

Take home point: Suspect heparin-induced thrombocytopenia (HIT) when a patient is on heparin and begins to develop a drop in
platelets and/or new or advancing thromboses. Stop heparin right away and start heparin alternative.

22. 7-year-old girl with 10-minute history of severe shortness of breath. Diagnosed with throat tumor 2 years ago. RR 38.
PE nasal flaring. Laryngoscopy shows multiple raised, finger-shaped lesions from vocal cords and epiglottis. Lesions
excised and shows finger-shaped fibrovascular cores lined with benign squamous epithelium. Causal virus?
- Human papillomavirus, type 6


Why it’s the right answer: This is an uncommon presentation of warts or papillomas secondary to HPV type 6 of the throat. Type 6
and 11 account for 90% of genital warts. The gross and histopathological findings are consistent with papilloma – finger-shaped
lesions with fibrovascular core and benign squamous epithelium. HPV 6 and 11 are associated with benign warts, while HPV 16 and
18 are associated with higher rates of malignancy. This patient is acutely compromised from tracheal obstruction by the warts,
leading to respiratory distress, increased RR, and nasal flaring, the patient’s response to attempt to improve air flow. The treatment
in this case would be intubation followed by surgical removal of the warts. Follow-up should also include child and sexual abuse
assessment.

Take home point: HPV type 6 and 11 account for 90% of genital warts. Gross and histology show papillomas or finger-shaped
projections and a fibrovascular core lined by benign squamous epithelium, respectively.

23. A 59-year-old man comes to the physician because of a 2-month history of light-headedness and tightness in his chest
with exertion. He adds that the pain is worse after arguing with his wife, and the symptoms resolve with rest. He has a
past history of lower gastrointestinal bleeding; evaluation at that time was negative on upper endoscopy and
colonoscopy. His temperature is 37°C (98.5°F), pulse is 80/min, respirations are 14/min, and blood pressure is 112/70 mm
Hg. Physical examination shows no abnormalities. His hemoglobin concentration is 7.9 g/dL, and hematocrit is 22%. Test
of the stool for occult blood is positive. An ECG shows no abnormalities. Repeat colonoscopy shows no abnormalities.
Which of the following is the most likely cause of this patient's gastrointestinal symptoms?
- Angiodysplasia


Why it’s the right answer: This patient has a lower GI bleed that has progressed to anemia evidenced by the patient’s H&H, which
may be one reason the patient feels light-headed and is easily fatigued with exertion. The first and second most common causes of
lower GI bleed is diverticulosis and angiodysplasia, respectively. Since the colonoscopy was normal and you would see diverticuli on
the exam, the next most likely cause is angiodysplasia.
10 ©Test Pirates, LLC. All Rights Reserved.
Step 1 Practice Test #18 Explanations


Angiodysplasia is dilation and tortuosity of blood vessels in the mucosal layer of the colon that can easily bleed. Angiodysplasia is not
always seen on colonoscopy and angiogram is sometimes needed to confirm the source and site of the bleeding. Another key point
to note, which the question may or may not be alluding to with the patient’s symptom of chest tightness (though no murmur was
heard on exam), is that angiodysplasia is associated with aortic stenosis and von Willebrand’s factor (vWF) deficiency. The reason
being is that the shearing forces of the stenosis of the aortic valve, further breakdown vWF, and people who are already deficient in
it are more prone to bleeding, especially in an area of weakened and exposed blood vessels of angiodysplasia in the colon.

Take home point: The first most common cause of lower GI bleed is diverticulosis. The second most common is angiodysplasia,
which is not always seen on colonoscopy like diverticuli and angiogram is needed to confirm. There is an association between
angiodysplasia, vWF deficiency, and aortic stenosis.

24. A 32-year-old woman comes to the physician because of a 1-year history of intermittent, diffuse, cramping lower
abdominal pain. The pain is usually associated with 4 to 5 days of loose, watery stools, and is typically relieved with
defecation. Between these episodes, her stools are normal. Her vital signs are within normal limits. Physical examination
shows no abnormalities. Laboratory studies, including complete blood count, metabolic panel, and thyroid function tests
show no abnormalities. A drug targeting which of the following mechanisms of action is most appropriate for this
patient?
- Accentuation of μ-opioid myenteric plexus receptor


Why it’s the right answer: This patient has signs and symptoms of irritable bowel syndrome (IBS) because she has pain and diarrhea
that is relieved once she has a bowel movement, and normal bowel movements in between episodes. If she did not have normal
bowel movements between episodes and/or pain not relieved by having a bowel movement, then this would be consistent with
crohn’s/IBD. The treatment in her case is to slow down the transit of stool in her bowel to promote the reabsorption of water, and
resolve her diarrhea. An example of a medication used to treat this is loperamide, which is an anti-diarrheal agent that agonizes the
mu-opioid myenteric plexus receptor. If you didn’t remember the action of loperamide or that specific drug, recall that opiates when
used to treat pain have the side effect of constipation because of their action on the opioid receptor in the intestine. So, using an
agonist at the opioid receptor would treat her diarrhea of IBS.

Take home point: IBS is abdominal pain associated with constipation or diarrhea that is relieved with defecation and resolves
between episodes. It is treated symptomatically, and for the diarrhea form, can be treated with loperamide, an agonist of the mu-
opioid myenteric plexus receptor.

25. 59-year-old woman with 2-week history of muscle cramps, weakness, abdominal pain, and constipation.
Hypertension treated with metoprolol and hydrochlorothiazide for past 3 months. Labs show hypokalemia. Which drug
should be added?
- Triamterene


Why it’s the right answer: Due to the diuretic hydrochlorothiazide (HCTZ), this patient has hypokalemia and also likely
hypercalcemia causing muscle cramps, weakness, pain, and constipation. Recall HCTZ causes hyperGLUC (hyperglycemia, -lipidemia,
-uricemia and -calcemia). Although the culprit medication could be stopped to improve this, her blood pressure may then spike. So,
a potassium-sparing diuretic can be added to her medication regimen, which will improve her BP and prevent the loss of K+. Other
potassium-sparing diuretics include spironolactone, eplerenone, and amiloride. The mechanism of action of triamterene and
amiloride is to block the Na+ channel in the cortical collecting tubules.

Take home point: HCTZ side effects include hypokalemia which can cause muscle cramps and weakness and hypercalcemia (and
hyperGLUC) which can cause constipation. Potassium-sparing diuretics should be added if patients are symptomatic.

11 ©Test Pirates, LLC. All Rights Reserved.
Step 1 Practice Test #18 Explanations

26. A 68-year-old woman comes to the physician for a routine pelvic examination. During speculum examination of the
vagina and cervix, the Valsalva maneuver causes a bulge of the anterior vaginal wall. Which of the following is the most
likely cause of this finding?
- Cystocele


Why it’s the right answer: Things start to prolapse when you get older, and the urethra in women is no exception. A cystocele is a
commonly asked concept on the test because it is a common benign finding in older women. It can even present as spontaneous
episodes of urinary incontinence due to the fact that the cystocele is collecting urine while sitting in the vaginal vault, and then
prolapses back into place and drain urine down the urethra. A rule of thumb to go by is for the vaginal exam is: anterior prolapse =
cytocele, posterior prolapse = rectocele. Based on the anatomy of urethra, vagina and rectum, this makes sense.

Take home point: Cystocele can present as bulging of the anterior vaginal wall during Valsalva on vaginal exam, with symptoms of
episodes of spontaneous urinary incontinence.

27. 1-month-old boy well child exam, mother with no concerns. 30th percentile for length and 40th percentile for weight.
Cardiac exam shows blowing holosystolic murmur best heard at lower left sternal border. Cause?
- Ventricular septal defect (VSD)


Why it’s the right answer: This question could be made more difficult on the actual test by, instead of describing the murmur,
having the test-taker listen to the murmur and a few background facts on the patient. A VSD is classically a holosystolic murmur
similar to mitral or tricuspid regurgitation (MR/AR). The difference between these other murmurs and a VSD is the location of the
loudest component of the murmur and the intensity. A VSD is best heard at the tricuspid region, while MR is best heart at the apex
with radiation towards the axilla. TR is loudest at the tricuspid region with radiation to right sternal border. A VSD will more likely be
harsher than MR/TR. One similarity between TR and VSD is that they both increase with inspiration (VSD also increases with
increased afterload i.e. handgrip), so distinguishing them may pose additional difficulties on the test. Therefore, the only way they
would help you to distinguish the two is to give more supporting information – i.e. IV drug user is more likely to have TR, while a
young child with no other pertinent past medical history is more likely to present with a previously undiagnosed VSD.

Take home point: VSD is a harsh holosystolic murmur that increases with afterload (i.e. handgrip) and inspiration. TR is another
holosystolic murmur with radiation to the right sternal border, and also increases with inspiration.

28. 3-year-old boy 3-day history of fever, sore throat, rash. T 102.1 F (38.9 C). Widespread, red, sandpaper-like rash on
extremities. Purulent exudate over tonsils. Dx streptococcus pyogenes (group A) infection. Cause of skin findings?
- Erythrogenic toxin


Why it’s the right answer: This patient has scarlet fever due to the toxigenic causes of erythrogenic toxin (or exotoxin A) of S.
pyogenes. Scarlet fever is characterized by fever, purulent exudates over tonsils, headache, a finely diffuse and sandpaper-like rash,
and also a red and bumpy tongue (strawberry tongue). The rash also characteristically spares the face. The toxin here can also cause
a toxic shock-like syndrome as well as necrotizing fasciitis. Finally, when S. pyogenes causes other manifests of disease, it is not
secondary to the toxin of the bacteria. These include the pyogenic sequelae, pharyngitis, cellulitis, impetigo, and the rheumatic
sequelae, rheumatic fever and acute glomerulonephritis.

Take home point: Streptococcus pyogenes causes many different diseases including pharyngitis, cellulitis, impetigo, scarlet fever,
toxic shock-like syndrome, necrotizing fasciitis, rheumatic fever, and acute glomerulonephritis. The toxin of S. pyogenes is
erythrogenic toxin or exotoxin A, and it can cause scarlet fever, toxic shock-like syndrome, and necrotizing fasciitis.

12 ©Test Pirates, LLC. All Rights Reserved.


Step 1 Practice Test #18 Explanations

Scarlet fever presents as fever, headache, red throat, red rash and red tongue. The rash is characteristically finely diffuse like
sandpaper, lightly red, with facial sparing.

29. 2-year-old boy with rash for 3 weeks. 12th percentile for height and weight. PE scaly, seborrheic eruption over scalp,
palms, back, diaper region and soles of feet. Generalized lymphadenopathy and hepatosplenomegaly. Xray of skull shows
osteolytic lesions. EM biopsy of skin shows tennis racket-shaped bilamellar granule in cytoplasm. Immuno studies show
CD1a antigen expression. Abnormal cells in patient are derived from which cell?
- Dendritic cells


Why it’s the right answer: This is a repeated question on Langerhans cell histiocytosis. The disorder is of the Langerhans or dendritic
cells. In children, the presentation described in the question stem is characteristic – a diffuse, eczema- or seborrheic-like rash with
lytic bone lesions. This is a proliferative process of white blood cells, so there is generalized lymphadenopathy as well as
hepatosplenomegaly secondary to extramedullary hematopoiesis. The tennis racket-shaped granules in the cells are key and also
described as a bilamellar granule in the cytoplasm (which creates the head of the tennis racket). The abnormal cells are positive for
CD1a and S-100 antigens. Remember when this disease presents in patients 2 years or younger, Langerhans cell proliferation is more
advanced involving multiple organ systems with a high rate of mortality.

Take home point: Langerhans cell histiocytosis presents in child as a diffuse eczema (or seborrheic) type of rash with lytic lesions on
X-ray. It is a disorder of Langerhans, or dendritic, cells.

30. 24-year-old man loses 18% total blood volume 5 minutes after motor vehicle collision. Finding most likely?
- Increased sympathetic nerve traffic to sinoatrial node


Why it’s the right answer: This question is asking what happens to the heart when total blood volume decreases. In order to perfuse
vital organs, when total blood volume decreases, there is vasocontraction peripherally and increased heart contractility and rate.
Both of these responses attempt to continue to push blood to vital organs despite the drop in blood volume. In order to increase
heart rate, the SA node has to fire more often, and to do this sympathetic nerve tone carrying signal to the SA node increases. Vice
versa, to slow the heart rate down (maybe after a blood transfusion is given to the patient and total blood volume increases),
parasympathetic nerve activity to the SA node via the vagus nerve would increase in order to slow down the heart rate.

Take home point: When total blood volume decreases, the heart reflexively increases in rate and contractility. Rate is increased via
increased sympathetic nerve activity to the SA node. Conversely, heart rate decreases through increased parasympathetic activity
via the vagus nerve to the SA node.

31. 38-year-old man comes to physician for benzodiazepine prescription for situation at work. Gives 2-month regimen of
benzodiazepine. Wife calls, "My husband got fired! I know it was because he was taking too much of that drug you gave
him. Didn't you know he had history of alcoholism?" Initial action?
- Contact the patient to discuss the situation


Why it’s the right answer: This is a great ethics question – it’s relevant and a very plausible situation. The underlying concept that
this question is trying to address is discussing information with family members and also discussing medical information over the
phone. Generally, phone conversations regarding patient information are always with the patient only – and this also excludes
leaving pertinent patient information in a voicemail. Even if the family member says it’s okay to talk about it with her over the phone
and that she obtained her husband’s permission, it’s not okay. If she came to the office, because it’s not he patient, the physician
should also not discuss the patient’s information with her unless given permission by the patient himself. The wife does bring up a
relevant point – was it okay to give the patient more benzo’s in light of his alcoholism the physician did not know about? Again, we
13 ©Test Pirates, LLC. All Rights Reserved.
Step 1 Practice Test #18 Explanations

cannot confirm he has a history of alcoholism without speaking with the patient directly. And though this is a concerning situation,
it’s not a medical emergency, and the patient should be called in to the office to discuss the it.

Take home point: Medical information should not be given to family members in-person or over the phone without the patient’s
direct permission. When leaving a voicemail, no patient information should be given as well.

32. 28-year-old man has operation for hyperparathyroidism. Three parathyroid glands found but one does not appear in
normal superior location on right side. Embryologic event that led to this?
- Abnormal migration of endoderm from fourth pharyngeal pouch


Why it’s the right answer: The brachial arch and pouch derivative should be memorized because they are high-yield. This question
pertains to a favorite question to ask about – the counterintuitive brachial pouch derivatives of the superior and inferior parathyroid
glands. It’s easy to remember these because it is opposite of what you would think – the third brachial pouch becomes the inferior
parathyroid glands and the fourth becomes the superior parathyroid glands. The third and fourth pouches migrate and switch
places. It’s also important to know that the ventral wings (the dorsal wings becomes the parathyroid glands) of the third pouch
become the thymus.

Take home point: The third brachial pouch becomes the inferior parathyroid glands and the thymus, while the fourth becomes the
superior parathyroid glands.

33. 60-year-old female with a history of breast cancer who received 6-week course of radiations 8 months ago comes for
follow-up. Exam shows no cancer recurrence. RR 28/min. CT chest shows b/l atelectasis in upper lung fields. Primary
pathophysiological cause?
- Contraction


Why it’s the right answer: Radiation can lead to fibrosis and also new malignancies. In this case, the patient has atelectasis in the
upper lung fields, which is directly where the radiation was given, and likely caused fibrosis. It’s less likely that this patient has a new
malignant process bilaterally in the lungs. It’s also less likely that this atelectasis represents a consolidative or obstructive process
such as a pneumonia because the patient has no other symptoms. Reabsorption may cause the collapse of some of the lung air
spaces, but there is nothing to reabsorb like fluid, infection, etc. The fibrosis in this case results from the radiation, and then it
shrinks or contracts months later, causes collapse of the lung air spaces (atelectasis).

Take home point: Radiation to the chest can cause fibrosis of the lungs, secondary contraction, and atelectasis.

34. 3-year-old boy (pedigree shown) has clumsy gait for past year. Exam calf hypertrophy and proximal muscle weakness.
Creatine kinase increased. Muscle biopsy shows loss of muscle tissue, regenerating muscles fibers and fibrosis. Maternal
uncle had similar findings and died at 12 years. Patient's sister is pregnant. Ultrasound identifies male fetus. Probability
fetus has disorder?
- ¼


Why it’s the right answer: The diagnosis is Duchenne’s muscular dystrophy. This muscular dystrophy usually prevents prior to age 5
and results in muscle weakness with elevated creatine kinase due to accelerated muscle breakdown. The calf hypertrophy here is
pseudohypertrophy since it’s not the muscle that’s enlarged with more muscle tissue, but replacement of the muscle with fibrofatty
tissue. The mutation (deletion of the dystrophin gene) is inherited in an X-linked recessive pattern. For the patient to have the
disease, his mother must have been a carrier. The chances that the daughter (his sister) is a carrier is 50-50 (or ½) because one of her

14 ©Test Pirates, LLC. All Rights Reserved.


Step 1 Practice Test #18 Explanations

X chromosomes is from the mother, and she has a 50-50 chance of getting the X with the mutation. If she, the daughter, is a carrier,
then the chances of her son inheriting the mutated X is also 50-50, since we know the fetus is male and there are only two
d
possibilities: X Y or XY. The probability of two events occurring in the multiplication of the probability of each event: ½ X ½ = 1/4.

Take home point: Duchenne’s muscular dystrophy is a mutation in dystrophin gene inherited in an X-linked recessive pattern. The
probability of two events occurring in genetic disorders is the multiplication of each individual event. First calculate the probability of
a person being a carrier, then treat the situation as if the person is a carrier, and calculate the probability of the event occurring.

35. 60-year-old man northern European descent with 4-month weakness, altered skin color, bilateral knee pain. Siblings
have type 2 diabetes and cirrhosis. PE bronzed skin, hepatomegaly, arthritis. Increased saturation of transferrin and
ferritin. Liver biopsy increased iron content and cirrhosis. Greatest risk?
- Hepatocellular carcinoma (HCC)


Why it’s the right answer: This patient has hereditary hemochromatosis, which is the deposition of hemosiderin (iron) due to a
genetic mutation. Hemochromatosis can also be secondary to multiple blood transfusions. Probably the highest yield facts to know
about this disease concept are the lab values, the gene association (which not everyone memorizes and could gain you extra points),
and secondary cancer risk, which is the answer to this question. The cancer risk comes from the destruction of the liver from the
iron, causing cirrhosis, and anyone with long-standing, advanced cirrhosis is at an increased risk for HCC. The lab values are also key
to understand. Everything is increased in terms of iron values except for TIBC (total iron-binding capacity), because there is so much
iron, transferrin is saturated, and the remaining capacity available to bind iron is normal to low; transferrin binds iron and transferrin
is saturated. Lastly, the gene that is mutated is the HFE gene (associated with HLA-A3), and the mutation is specifically either the
C282Y or H63D mutation.

Take home point: Patients with hereditary hemochromatosis are at an increased for hepatocellular carcinoma due to advanced
cirrhosis from hemosiderin (iron) deposition.

36. 27-year-old woman with infertility, receive injection of contrast material into cervix. On hysterosalpingogram
(shown), contrast material (indicated by arrows) also seen in peritoneal cavity, which explain this finding?
- Spillage of contrast


Why it’s the right answer: I think middle school health class taught us some strange concept that the ovaries and attached to the
fallopian tubes with a very tight seal, so that after ovulation, the egg is sucked into the fallopian tube down to the uterus. This is not
the case in real life. Anatomy shows us that the ovaries are suspended by ligaments and so are the fallopian tubes, but there is no
direct attachment between the two, and the fimbriae of the fallopian tubes are what draw the released egg from the ovary into the
tube. Therefore, the hysterosalpingogram shown is testing this knowledge, and your ability to identify the spillage of contrast into
the peritoneal cavity as a normal finding.

Take home point: At the end of a hysterosalpingogram test, contrast spills out from the fallopian tubes into the peritoneal cavity,
and this finding is normal.

37. 31-year-old man for psychiatric eval sent by employer because he is "very odd." Is a computer repair specialist and
lives alone. Refuses to socialize and has no friends. Extremely preoccupied with science fiction, occult, afterlife.
Personality disorder?
- Schizotypal

15 ©Test Pirates, LLC. All Rights Reserved.


Step 1 Practice Test #18 Explanations

Why it’s the right answer: Schizotypal is schizoid personality disorder plus odd thinking. Schizoid personality disorder is
characterized by social withdrawal and a desire to be alone (unlike avoidant patients who desire to be with other people but have
social anxiety disorder). These patients typically have no friends. This patient is schizotypal with the addition of his preoccupation
with sci-fi and magical thinking. He is not schizophrenic because he doesn’t have the additional criteria required – increased odd
thinking/behavior with signs of paranoia that inhibit overall function. The key to diagnosing schizophrenia is an inability to function
in society and at home – i.e. not holding a job, not bathing, not brushing teeth.

Take home point: Schizotypal personality disorder is schizoid personality disorder (desire to be alone) plus odd or magical thinking.

38. 34-year-old woman with 2-week low-grade fever and joint pain. Has chronic headaches and takes ibuprofen several
times. PE diffuse maculopapular rash. UA 3+ protein, 18 WBC and eosinophils. Renal biopsy would show what?
- Inflammatory infiltrates in the interstitium


Why it’s the right answer: The diagnosis is interstitial nephritis induced by ibuprofen. Typically, when a drug is the cause of
interstitial nephritis, the onset of the kidney disease is within days to weeks (i.e. with diuretics, penicillin, cephalosporin, sulfa drugs,
rifampin). However, with NSAIDs, the onset can be months after taking the NSAID, which is the case here. The UA also shows
nephrotic range protein levels, increased WBCs and eosinophils. Eosinophils give the diagnosis away in this case – both the patient’s
serum and urine will often be positive for eosinophils. Biopsy will show inflammation in the kidney interstitium with sparing of the
glomeruli and blood vessels. Treatment targets the autoimmune cause of this disease and includes at least a 2-week course of
corticosteroids.

Take home point: Drug-induced interstitial nephritis typically occurs days to weeks after the inciting drug is started unless it is
NSAIDs, which can induce a kidney reaction even months after starting the NSAID. Renal biopsy shows interstitium inflammation
with sparing of the glomeruli and blood vessels.

39. 24-year-old man 1-day history of pain in left index finger. Injured it when catching a ball. PE shows erythema of left
index finger. Unable to flex the distal phalanx when proximal interphalangeal joint metacarpophalangeal joints
restrained. Xray normal. Injured structure?
- Flexor digitorum profundus tendon


Why it’s the right answer: This patient is unable to flex his index finger and the flexors of the fingers is the flexor digitorum
profundus tendon which comes from the flexor digitorum profundus muscle. The muscle originates at the upper anterior and medial
surface of the ulna and inserts onto the palmar base of the distal phalax in digits 2-5 (does not include the thumb). Its action is
flexion of the risk, MCP and interphalagial joints. The lumbricals also attach to the tendon, and the lumbricals are the extensors of
the interphalangeal joints. Thumb flexion is controlled by the flexor pollicis longus.

Take home point: Flexion of the wrist, MCP, and interphalangeal joints (digits 2-5) is controlled by the flexor digitorum profundus
tendon.

40. 45-year-old woman who has a history of facial flushing and rapid heart rate each time he consumes a small amount
of alcohol participates in study of alcohol intolerance. Molecular analysis shows presence of a lysine (K487, oriental
variant) for glutamate (E487, native variant) substitution in aldehyde dehydrogenase. Kinetic characteristics of the
enzyme variants shown:
Enzyme Km (NAD+) microM Kcat (min-1)
E487 42 190
K487 6100 10.5

16 ©Test Pirates, LLC. All Rights Reserved.


Step 1 Practice Test #18 Explanations

based on these findings, which of the following is the most likely cause of this man's condition?
- Decreased catalytic efficiency of K487


Why it’s the right answer: Long stems like these should not scare you because the longer they, the more information they are giving
you to figure out the correct answer. It is also not uncommon for a long question stem to have irrelevant facts, for which the answer
choices are usually of the ethics variety. This question may appear intimidating especially with its talk of K487, but if you’ve studied a
lot for this test, you know that you are not expected to know anything about K487, so they’re basically going to tell you the correct
answer. You don’t really need to know anything about ethanol metabolism to answer this question, but knowing a bit helps.

Ethanol is metabolized by two enzymes – alcohol dehydrogenase first in the cytosol of the hepatocyte, and then by acetaldehyde
dehydrogenase in the mitochondria. NAD+ is the limiting reagent. In patients who are deficient in the enzyme acetaldehyde
dehydrogenase, acetaldehyde builds up and causes facial flushing and an increase in heart rate (and also a bad hangover). In the
question stem, they are telling you that the Asian variant of this enzyme K487 is much slower than native variant (even if you don’t
understand the kinetic terms), so the efficacy of the enzyme’s catalytic effect is less. The drug disulfiram and disulfiram-like drugs
have a similar effect after alcohol ingestion by inhibiting acetaldehyde dehydrogenase.

Take home point: Ethanol is metabolized by alcohol dehydrogenase to acetaldehyde, which is then metabolized by acetaldehyde
dehydrogenase to acetate. Disulfiram (and disulfiram-like drugs) inhibit acetaldehyde dehydrogenase, leading to the buildup of
acetaldehyde, facial flushing, and elevated heart rate.

41. 52-year-old woman bmi 28, total cholesterol 200, HDL 52, triglycerides 610. Which drug to prescribe?
- Fenofibrate


Why it’s the right answer: The triglycerides are very elevated in this scenario. If we assume the patient fasted prior to the blood
draw (since fasting only affects the TG measurement), the medication that specifically targets triglycerides and works to lower them
are the fibrates – gemfibrozil, clofibrate, bezafibrate, fenofibrate. They are easy to remember because they all have the word
“fibrate” in some form or another. Other lipid-lowering agents can lower triglycerides but not as profoundly as fibrates, and this
patient only needs his triglycerides lowered. Fibrates also lower cholesterol and raise HDL, though, not to the extent of statins.
Fibrates work by upregulating lipoprotein lipase, which clear triglycerides.

Take home point: In a patient with profoundly elevated triglycerides, the first-line of treatment is a fibrate, the lipid lowering agent
that has the greatest effect on triglycerides.

42. 31-year-old woman with 5-days difficulty walking. One year ago had loss of vision in left eye which improved. Neuro
exam shows decreased visual acuity in left eye with pallor of optic disc. Has past-pointing on a finger-nose test. Broad-
based gait. MRI shows brain lesions in white matter of cerebellum. Pathogenesis?
- CD4+ T lymphocytes are activated by myelin basic protein


Why it’s the right answer: This patient has multiple sclerosis. Her transient vision loss in her left eye was an episode of optic neuritis.
In optic neuritis, there can be complete recovering of visual function, limited recovery, or no recovery (rare) depending on the
extent of inflammation and inflammatory damage. Here, the patient had partial recovery, with evidence of damage in the form of
optic disc pallor. She also difficulty with coordination due to cerebellum involvement of the lesions, as also demonstrated on MRI.
These patients also develop an intention tremor (tremor upon movement and with an outstretched arm) due to cerebellar
involvement. The pathogenesis is autoimmune, with IgG antibody production against the myelin sheath around nerves in the CNS
(brain and spine) – this is why the lesions are called “demyelinating,” since myelin is destroyed by the IgG antibodies and
inflammatory response. CD4+ T cells commence the immune response by creating a sensitization to myelin basic protein.

17 ©Test Pirates, LLC. All Rights Reserved.


Step 1 Practice Test #18 Explanations

Take home point: Multiple sclerosis is an autoimmune disease of CD4+ T cells and IgG production against myelin basic protein
(which initiates the immune response) and the myelin sheath around CNS nerves, resulting in demyelinating lesions and neurological
sequelae.

43. 68-year-old man 3-week sensation of fullness in left upper quadrant. Has lethargy and shortness of breath. 25-lb
weight loss during 2 months. PE shows pallor. Spleen tip palpated. Labs: hb 8.9, hct 26%, wbc 5000, serum uric acid 16.
Peripheral blood smear numerous erythrocytes with abnormal shapes and sizes, nucleated erythrocytes and myelocytes.
Aspiration of bone marrow dry tap. Biopsy shows thickened bony trabeculae with increased reticulum. Dx?
- Myelofibrosis


Why it’s the right answer: Myelofibrosis is a chronic myeloproliferative disorder due to the fibrotic replacement of bone marrow.
Fibrous tissue replaces the bone marrow, and on bone marrow tap, because there is simply fibrin, the tap is dry. Blood smear is also
characteristic for tear drop cells, which, when produced by the still functioning fibrous bone marrow, are RBCs that have squeezed
through the fibrous tissue and become misshapen upon entrance into blood circulation. Biopsy of bone showing thick bone with
increased reticulum is also characteristic – the reticulum is increased from the fibrous tissue. Finally, the patient overall shows signs
and symptoms of a myeloproliferative disorder – lethargy, weight loss, pallor (due to anemia), and splenomegaly (palpation of the
spleen tip is usually not possible with a normal-sized spleen). Again, the spleen enlarges because it becomes the site of
extramedullary hematopoiesis.

Take home point: Myelofibrosis is a chronic myeloproliferative disorders. Key findings include abnormally shaped or tear drop-
shaped RBCs, immature RBCs (nucleated) and WBCs (myelocytes), dry bone marrow tap (due to fibrous tissue replacement of bone
marrow), and bone biopsy demonstrating thickened trabeculae with increased reticulum.

44. 2-year-old has numerous infections of skin and oral mucosa since birth. Infections slow to respond to antibiotic
therapy. T 100.5F. PE multiple erythematous lesions of skin some with superficial ulceration. WBC 22,000 78% segmented
neutrophils, 8% bands, 13% lymphocytes and 2% monocytes. Biopsy shows no neutrophils in dermis or epidermis. Culture
of lesion grows Staphylococcus aureus. Defective?
- An integrin


Why it’s the right answer: The diagnosis is leukocyte adhesion deficiency syndrome which is a deficiency of LFA-1 integrin protein on
phagocytes and neutrophils. These patients are immunocompromised and have recurrent bacterial infection without pus formation.
Another key finding synonymous with this disease process is delayed separation of the umbilical cord after birth due to the lack of
available phagocytes to breakdown and separate the cord. Labs typically demonstrate neutrophilia since none of the neutrophils can
exit the blood stream to fight infection in the dermis or epidermis and stay in the blood stream during a superficial infection. It is
also high-yield to know the steps and each key modulator (i.e. integrin/LFA-1) of leukocyte extravasation: rolling à tight binding
(integrin needed to bind to ICAM-1 on the endothelium) à diapedesis à migration.

Take home point: Leukocyte adhesion deficiency syndrome is a defect in integrin (LFA-1), preventing neutrophils from existing the
blood stream to fight infections in the dermis and epidermis. Labs show neutrophilia and there is a history of delayed umbilical cord
separation after birth.

45. 39 yr old man with SOB on exertion x 8 months, nosebleeds since adolescence, 2 pictures: clubbing + hemorrhagic
lesions in tongue, inhaled albuterol doesn't improve his symptoms. Cause of symptoms?
- Pulmonary AV shunting


18 ©Test Pirates, LLC. All Rights Reserved.
Step 1 Practice Test #18 Explanations

Why it’s the right answer: The diagnosis is hereditary hemorrhagic telangiectasia (also known as Osler-Weber-Rendu syndrome).
HHT is an autosomal dominant disorder of blood vessels that leads to telangiectasias and AV malformations throughout the body.
Symptoms include recurrent nosebleeds, GI bleeds, skin discolorations, and problems breathing due to AVMs in the lungs. Also, it is
not uncommon for patients to present later in life in their 40s like this patient with delayed disease onset. What makes this question
tricky is the prominent pulmonary findings and clubbing, leading the test-taker to alternate diagnoses. However, if the AVMs are
present on the tongue, then they may also be present in the lungs, creating multiple shunts and leading to chronic hypoxemia. In
this patient, the hypoxemia has been so profound and long-standing, he has developed clubbing.

Take home point: Hereditary hemorrhagic telangiectasia is an autosomal dominant disorder of blood vessels that leads to
telangiectasias, AVMs, recurrent epistaxis, GI bleeds, and, if AVMs are present in the lungs, shortness of breath with signs of chronic
hypoxemia.

46. 32-year-old primigravid woman at 16 weeks’ gestation comes to physician for routine prenatal examination. The
uterus consistent in size with 16-week gestation. Ultrasonography shows a male fetus. The collecting system and pelvis of
the left kidney is dilated and the renal cortex appears compressed. The left and right ureters are not dilated. The right
kidney appears normal. Amniotic fluid volume is normal. Which causing renal finding in this fetus ?
- Incomplete recanalization of proximal ureter


Why it’s the right answer: The ureteropelvic junction is the last part of the ureter to canalize. Failure of ureter canalization,
therefore, most commonly occurs at this junction point, leading to unilateral hydronephrosis as described above – dilated renal
pelvis with a compressed cortex. The amount of amniotic fluid is within normal limits because the other kidney is not affected and
producing a sufficient amount of fluid to maintain the amniotic fluid. This fetus will likely have only one functioning kidney at birth
due to damage from the hydronephrosis in utero, but will not suffer additional consequences since the level of amniotic fluid is
normal. If both kidneys were disrupted, oligohydramnios would develop and lead to Potter’s syndrome.

Take home point: The most common location for ureter obstruction in utero from maldevelopment is at the ureteropelvic junction
because it is the last to canalize.

47. 31-year-old man with 2-day progressive numbness of both feet ascended to thighs. Last 24 hours, numbness and
tingling of hands. PE ataxic gait. Deep tendon reflexes diminished in upper extremities and absent in knees and ankles.
Vibration and joint position absent in fingertips and feet bilaterally. Mild weakness distal upper extremities ad moderate
weakness of lower extremities. Structure involved?
- Myelinated primary afferents


Why it’s the right answer: The rule is: ascending weakness = Guillain-Barre (or acute inflammatory demyelinating
polyradiculopathy), descending weakness = botulism. The disease course described above is ascending weakness that started in the
feet and progressed upwards. In G-B syndrome, the peripheral nerves and motor fibers are affected – so touch, vibration, position,
motor function are all affected. Patient can also have autonomic function impairment (i.e. hypotension, hypertension). The specific
antigenic target is the Schwann cell, which is the myelinating cell of the peripheral nervous system (versus oligodendrocytes, which
are the myelinating cell of the central nervous system). Therefore, the structure that is demyelinated are the primary afferents of
the various pathways described above.

Take home point: The rule is: ascending weakness = Guillain-Barre (or acute inflammatory demyelinating polyradiculopathy),
descending weakness = botulism. The target of G-B syndrome is the Schwann cell, resulting in demyelination of the peripheral
nervous system.

19 ©Test Pirates, LLC. All Rights Reserved.


Step 1 Practice Test #18 Explanations

48. 76-year-old woman comes to the physician because of constant severe lower abdominal pain and fever for 24 hours.
Laproscopic examination shows severe diverticulosis and perforated diverticulitis. In spite of appropriate therapy, she dies
3 days later. Liver autopsy shown. Which of the following is the primary component of the material shown on the hepatic
surface?
- Fibrin


Why it’s the right answer: This is a pathology question on the occurrence of fibrin after inflammation. Inflammation during acute
infections and inflammation during wound healing, leads to the deposition of fibrin. Within 2-3 days of an infection and
inflammatory process spilling over into the peritoneal cavity, fibroblasts and myofibroblasts join neutrophils and macrophages to
begin laying down fibrin. In this case above, there was both an infection and a surgery that led to a fibroblast response. Liver biopsy
would show fibrin in patches on the surface of the liver where the infection had spilled over, and inflammation was replaced by
fibrin. As a side note, the most common cause of bowel obstruction post-surgery is adhesions, which are made of fibrin laid down by
fibroblasts after the surgery in healing and post-inflammatory process.

Here’s an interesting case report demonstrating the presence of fibrin after perforated diverticulitis:
https://www.ncbi.nlm.nih.gov/pmc/articles/PMC4357822/

Take home point: Post-infection and inflammatory states lead to fibroblast proliferation and an increase in fibrin deposition. Surgery
also leads to similar fibroblast recruitment and fibrin deposition, which make up the adhesions commonly found post-operatively.

49. 30-year-old woman comes to the physician because of a 6-day history of palpitations, severe neck pain, fatigue, and
malaise. Her pulse is 118/min and regular. Physical examination shows a diffusely tender, mildly enlarged thyroid gland.
There is no exophthalmos. Serum studies show a thyroid-stimulating hormone concentration of 0.01 μU/mL. Which of the
following is the most likely diagnosis?
- Subacute granulomatous thyroiditis


Why it’s the right answer: This is one of those challenging hypothyroid/hyperthyroid questions, and of course the answer is a
hypothyroid variant that can also present early on in the disease course as hyperthyroid. The important thing to remember about
hyper and hypothyroidism, is that it is a spectrum of diseases, and just because someone is hyperthyroid at one point does not mean
they cannot become hypothyroid later on (or vice versa, which is much less common). Patients also with thyroid eye disease (or
Graves’ ophthalmopathy) can also be hyperthyroid, hypothyroid, or euthyroid; though hyperthyroid is the most common association
with thyroid eye disease. The patient presented here is clearly hyperthyroid (signs/sx of palpitations, elevated HR, low TSH), but the
key is that the entire thyroid gland is diffused tender. That point is what makes this patient hyperthyroid with subacute
granulomatous thyroiditis. Remember, tender gland = thyroiditis (which is granulomatous inflammation).

Take home point: Subacute granulomatous thyroiditis can present early in the disease course as hyperthyroidism and then progress
to hypothyroidism. The unique feature of thyroiditis is the diffusely tender and mildly enlarged thyroid gland.

50. 29-year-old primigravid woman 35 weeks' gestation with 4-hour history of heavy vaginal bleeding. No prenatal care.
Ultrasound shows placenta over cervical os. Can't stop bleeding and has cesarean. Dx?
- Placenta previa


Why it’s the right answer: Placenta previa is the physical attachment of the placenta to the lower uterine lining near or overlying the
internal cervical os. The characteristic finding in placenta previa is that the bleeding that occurs is usually painLESS (versus abruptio
st nd rd
placentae). Another key fact is that the bleeding from previa can occur during any trimester – 1 , 2 , or 3 (versus abruptio is only
rd
during the 3 ). Risk factors for previa include smoking, advanced maternal age, prior previa, multiparity, and prior C-section (since
20 ©Test Pirates, LLC. All Rights Reserved.
Step 1 Practice Test #18 Explanations

the incision is low-lying in the uterus, and if the placenta attaches here it can overlay the cervical os). Placenta previa is also
associated with placenta accreta (the attachment of the placement to the myometrium due to defective decidual layer). The last
important point to make that may come up as a future question is that the diagnostic ultrasound should never be a vaginal
ultrasound and also rectal exams should be avoided – both can risk precipitation of hemorrhage from the placenta previa.

Take home point: Placenta previa is when the placenta is near or overlies the cervical os, and can lead to painless bleeding during
any trimester of pregnancy.

21 ©Test Pirates, LLC. All Rights Reserved.















Practice Test Explanations
Step 1 Practice Test # 18 – Block 4
Question Total: 50


Step 1 Practice Test #18 Explanations

2 ©Test Pirates, LLC. All Rights Reserved.


Step 1 Practice Test #18 Explanations


Block 4

1. 42-year-old man with polycystic kidney disease has 6-month history intermittent blood in urine. T 98.6 F HR 98 RR 22
BP 155/86. Urea nitrogen 95, creatinine 7.1. UA shows blood. Arterial blood gas shows?
pH/pCO2/HCO3-
- 7.24/30/12


Why it’s the right answer: This patient is in end-stage renal failure with gross blood in his kidneys. The question addresses the acid-
base state in the serum in a patient with kidney failure not yet on dialysis. In kidney failure the acid-base state is a partially
compensated metabolic acidosis due to the kidney’s decreased ability to excrete H+ ions in the urine. Therefore, the ABG shows an
acidotic pH with a low bicarb, and a low pCO2 since the lungs are trying to compensate and blow off CO2 to create an alkalosis.

Take home point: The acid-base state in kidney failure is a metabolic acidosis due to inability of kidneys to extreme H+ ions.

2. 56-year-old man 2-month increasing abdominal girth and inability to achieve erection. Smoked 1.5 pack cigarettes for
30 years and drank 1-2 pints of liquor daily. Vitals normal. PE shows scleral icterus, spider angiomata, gynecomastia,
ascites and prominent umbilical venous pattern. Testes small. Cause of gynecomastia?
- Failure of liver to degrade estrogen

Why it’s the right answer: This patient has alcoholic liver cirrhosis – 1-2 pints of hard liquor a day will do that to a person. He has all
the signs of cirrhosis – scleral icterus, spider angiomata, gynecomastia, ascites, and caput medusa (or a prominent umbilical venous
pattern). Scleral icterus is due to the buildup of bilirubin because the liver can no longer conjugate it. Spider angiomas are a type of
telangiectasia with superficial arteriolar dilation with radiating capillaries that blanch and refill from the center outwards when
pressure is applied. Estrogen buildup is also the cause of spider angiomas. Ascites is secondary to hypoalbuminemia, while caput
medusa is a porto-systemic anastomosis due to portal hypertension. In terms of the gynecomastia, the liver largely contributes to
the breakdown of estrogen in the body, and in a cirrhotic liver, estrogen causes gynecomastia.

Take home point: In cirrhosis, the liver fails to breakdown estrogen which leads to gynecomastia and spider angiomata.

3. 22-year-old woman is admitted to the hospital for antibiotic treatment of meningococcal meningitis. She is stabilized.
Three days later, her pulse is 130/min, and blood pressure is 65/39 mm Hg. Physical examination shows bilateral flank
tenderness. Serum studies show a sodium concentration of 126 mEq/L, potassium of 5.8 mEq/L, and bicarbonate of 19
mEq/L. Which of the following is the most appropriate next step to determine the cause of this patient's hypotension?
- Adrenocorticotropic hormone stimulation test


Why it’s the right answer: The diagnosis is Waterhouse-Friderichsen syndrome, which is bilateral adrenal hemorrhage from a
coagulopathy secondary to N. meningitis. Specifically, the hemorrhage is in the adrenal cortex causing low mineralocorticoid levels
that result in low blood pressure, low serum Na+, high serum K+, all of which are described in the stem. The question then asks what
3 ©Test Pirates, LLC. All Rights Reserved.
Step 1 Practice Test #18 Explanations

test should be ordered to determine the cause of the hypotension, and not necessarily the immediate treatment of the hypotension
– which would likely include IV fluids and corticosteroids. Because this is primary adrenal insufficiency (there is hemorrhage in the
adrenal glands), a ACTH sitmulation test will show no response to ACTH since the adrenal glands are not functioning. This test would
help distinguish primary adrenal insufficiency from secondary because in secondary adrenal insufficiency there is a deficiency of
ACTH, and giving ACTH would stimulate the adrenal glands to produce cortisol.

Take home point: Waterhouse-Friderichsen syndrome is hemorrhage into bilateral adrenal glands that occurs in the setting of N.
meningitidis septicemia and DIC. It results in primary adrenal insufficiency manifesting as hypotension. The ACTH stimulation test
can be used to distinguish primary from secondary adrenal insufficiency.

4. 62-year-old woman undergoes operative repair of inguinal hernia. Surgeon has received written informed consent
from patient. After epidural received, patient decides she does not want to have surgery. Best response?
- "Tell me about your concerns and why you want to cancel the operation."


Why it’s the right answer: Even if the consent is signed, the patient has the prerogative to cancel the surgery. However, instead of
agreeing with the patient and canceling the surgery, it is important to first address his concerns, and if there is something that can
be explained further to the patient that he may have not understood, then the surgery can still proceed with his consent. The
general guideline for the ethics questions is that the open-ended questions or solutions are the first-line best response.

Take home point: Though a consent for a surgery or procedure may be signed, a patient has the right to cancel the procedure. The
best response is to first inquire about his/her concerns to avoid canceling the procedure from which the patient will benefit.

5. A 31-year-old woman is admitted to the hospital because of severe crush injuries to the chest and extensive burns over
40% of her body surface area. Four hours later, she develops tachypnea and dyspnea. Arterial blood gas analysis on room
air shows a decreased Po2 and Pco2. A chest x-ray shows bilateral interstitial and alveolar infiltrates. The patient is
intubated and mechanically ventilated. Damage to which of the following is most likely to preclude restoration of normal
tissue architecture and pulmonary function in this patient?
- Basement membranes


Why it’s the right answer: This patient has acute respiratory distress syndrome (ARDS) secondary to trauma and burns. The
diagnosis is ARDS based on the inciting mechanism of injury (other causes of ARDS include sepsis, shock, and acute pancreatitis),
decreased pO2 and CO2 (meaning oxygen is not diffusing across the damaged/collapsed alveoli, and CO2 is being blown off by a
reflexively increased respiratory rate), and the chest X-ray is consistent with ARDS (bilateral interstitial and alveolar infiltrates). In
ARDS there is an influx of proinflammatory cytokines and neutrophils in response to the trauma/infection, and these cytokines
damage the delicate pulmonary capillary endothelium and alveolar wall, leading to a leakage of protein-rich fluid into the lung
interstitium and lung air spaces (the reason for b/l interstitial and alveolar infiltrates on chest X-ray). Several structures of the lung
are damaged by cytokines and neutrophils, but the basement membrane takes a particular hit because a protein layer is laid down
to create a hyaline membrane lining the basement membrane of the alveoli. This hyaline membrane along the BM limits the
recovery of the lungs after ARDS.

Take home point: ARDS can be secondary to trauma or infection, and results in a proinflammatory cascade involving cytokines and
neutrophils. Protein rich fluid fills the lungs, leading to alveoli destruction through the creation of a hyaline membrane on the
basement membrane and further capillary leakage.

6. Studying epithelial repair of small intestine in experimental animal. Wants to identify most active cell division location.
Where is this cell activity found?
- Base of the crypt
4 ©Test Pirates, LLC. All Rights Reserved.
Step 1 Practice Test #18 Explanations


Why it’s the right answer: Stem cells are found at the base of the crypt within the intestinal villi that line the intestinal wall. The
migration of the new cells is from the base of the crypt upwards. The cells at the apex, the oldest cells, of the villi are shed. The stem
cells are by far the most active cells because they are needed to reproduce more cells and constantly regenerate the lining of the
intestine.

Take home point: In the lining of the intestine, mitotically active stem cells are found at the base of the crypt where they produce
new intestinal cells that migrate from the base of the crypt up the apex of the villi.

7. 28-year-old woman calls the physician on a Friday night 1 hour after a condom broke during sexual intercourse with
her boyfriend. She asks the physician to prescribe an emergency oral contraceptive. The physician on call is not the
patient's regular physician and does not dispense emergency contraception for moral reasons. After the physician
respectfully informs the patient that he does not prescribe this contraceptive, it is most appropriate for the physician to
state which of the following?
- "One of my colleagues will call you back to address your concerns."


Why it’s the right answer: One general rule to follow is that a physician should never refer the patient out to another physician
without first attempting to address the problem – but this question is one of the few exceptions. The question stem pretty much
gives you all the reasons to refer the patient to your colleague even though this is an emergency-type of situation. One reason is the
use of emergency contraception, which is an ethically charged topic in and of itself, and something that physician on-call does not
believe prescribe for moral reasons. If this is the case, the physician should not have to break his moral code, and can refer the
patient to someone else. Another reason is that the physician is no the patient’s regular physician. When a group practice is on-call,
in general treatment can still be given even if the doctor on-call isn’t the patient’s primary doctor. I think the question stem threw
this point in there though to give the physician the option of referred to his colleague – because if he didn’t have a colleague, then
he would have to refer the patient to the emergency department to obtain contraception. Lastly, a final tangential point to make it
that in real life, you don’t need a prescription for emergency contraception.

Take home point: physicians should usually not refer a patient to another physician without first attempting to address the problem.
However, one of the exceptions to this rule is giving treatment that goes against the physician’s morals– i.e. emergency
contraception, abortion.

8. 31-year-old woman with 1-week fever, shaking chills, headaches, fatigue, and joint and muscle pain. Spent summer
working as lifeguard in Long Island, New York. Has splenectomy for motor vehicle collision several years ago. T 101.9 F.
Peripheral blood smear shows small intraerythrocytic rings. Polymerase chain reaction for Plasmodium is negative.
Causal organism from bite of?
- Tick


Why it’s the right answer: The diagnosis is babesiosis, which you can think of as the North American version of malaria. Babesiosis
presents as fever, headache, muscle and joint pain, as well as a hemolytic anemia due to the RBC invasion of the organism. Blood
smear not only shows intraerythrocytic ring similar to that of malaria, but can also show a Maltese cross, which is a dead giveaway
for Babesia. The organism is transmitted via a tick bite, specifically the Ixodes tick, the same tick as Lyme disease. In fact, coinfection
with both Lyme (or Borrelia burgdorferi) and Babesia is can occur. Treatment for babesiosis includes dual therapy with both
atovaquone and azithromycin.

Take home point: Babesiosis is the North American version of malaria – presenting with fever, headache, joint pain, and anemia
with intracytoplasmic RBC rings. Characteristic of Babesia is also the Maltese cross in the RBC. Infection is secondary to a bite from
the Ixodes tick.
5 ©Test Pirates, LLC. All Rights Reserved.
Step 1 Practice Test #18 Explanations

9. 72-year-old man who eight months ago dx with primary lung carcinoma involving adrenal glands, liver, and bone. Had
35-lb weight loss during 4 months. PE shows cachexia and significant muscle wasting. Intracellular components increased
in patient's muscle cells?
- Autophagic vacuoles


Why it’s the right answer: This patient has both cachexia and muscle wasting, which is mediated by the paraneoplastic effect of
malignancy via cytokines TNF-alpha, IL-6, and IL-6. Weight loss and muscle wasting on the microscopic, cellular level is cell apoptosis,
or programmed cell death. In this process, the cell undergoes nuclear shrinkage (pyknosis), membrane blebbing, nuclear
fragmentation (or karyorrhexis), and the formation of apoptotic bodies. The cell then is phagocytosed. The process of apoptosis is
initiated by caspases. Therefore, autophagic vacuoles form within the muscle cells during apoptosis, ultimately leading to muscle
atrophy/wasting.

Take home point: During cachexia and muscle atrophy associated with malignancy, individual cells undergo apoptosis. The process
of apoptosis involves pynknosis, or nuclear shrinkage, membrane blebbing, autophagic vacuole formation, and nuclear
fragmentation, or karyorrhexis.

10. 6-month-old female with T 101.8F, vomiting, diarrhea, dehydration. Exam of stool shows viral particles with wheel-
like shape. Properties of virus?
Type of nucleic acid/envelop/capsid symmetry
- Double-stranded RNA, segmented/no/icosahedral


Why it’s the right answer: The virus is rotavirus, the only double-stranded RNA virus you need to be aware of for the purposes of
the test – all the other RNA viruses are single-stranded. The DNA is also segmented into about 10 segments, there is no envelope,
and the capsid is icosahedral (vs. helical). Rotavirus is the usual causal organism of diarrhea in infants – which this patient is, an
infant with profound diarrhea. Finally, on electron microscopy, the viral particles resemble wheels.

Take home point: Rotavirus, a cause of diarrhea in infants, is a double-stranded RNA virus that is segmented without an envelope,
and an icosahedral capsid. If you see an electron microscope image and a round viral particle that looks like a wheel, it is likely
rotavirus.

11. 8-year-old girl with type 1 diabetes mellitus and 2-hour history of lethargy, confusion, disorientation. Symptoms
gradually developed and she did not take her usual insulin dose during a sleepover. HR 112, RR 26 deep and rapid, bp
90/65. Labs: glucose 550, arterial pH 7.11. ABG?
pCO2/HCO3-/Anion gap
- decreased/decreased/increased


Why it’s the right answer: This patient has diabetic ketoacidosis (DKA), which causes an anion gap metabolic acidosis. The anion gap
exists because when we normally calculate the anion gap we take into account the usual anions in the body. If there is a gap, then
that means there are additional ions not being measured and accounted for. In this case, it’s the ketone bodies (ketoacids) that are
unmeasured and account for the anion gap. This is the meaning of an “anion gap” acidosis – there’s not truly a gap, just a gap in our
own measurement.

Increased insulin demand such as stress or infection often precipitates DKA – or not taking insulin like in this case of a young girl who
forgot to take her insulin at a sleepover. Her symptoms of lethargy, confusion and disorientation are characteristic of DKA along with
6 ©Test Pirates, LLC. All Rights Reserved.
Step 1 Practice Test #18 Explanations

her breathing, also known as Kussmaul breathing, which is rapid and deep. In DKA, the basic chain of events is that glucose cannot
be used by the cells for energy because there is no insulin, so the breakdown of fat occurs to provide the cells with energy. The fat
breakdown leads to free fatty acids, which are then made into ketone bodies (namely beta-hydroxybutyrate and acetoacetate) in
the process of ketogenesis. Therefore, the bicarb is low due to metabolic acidosis, the pCO2 is low in an attempt to compensate and
create an alkalosis and blow off CO2, and the anion gap is increased because of the ketones.

Take home point: DKA results in an anion gap metabolic acidosis due to ketogenesis and the production of ketoacids (beta-
hydroxybutyrate and acetoacetate).

12. Investigator studying vancomycin-resistant strain of Enterococcus faecalis. Twenty generations created. Culture
inoculated and resulting bacterial colonies are screened for vancomycin resistance. Vancomycin-sensitive colonies
observed at frequency of two per 353 cells. Mechanism of decreased vancomycin resistance?
- Plasmid loss


Why it’s the right answer: Bacteria can require resistance through the transfer of plasmid, which is a small amount of DNA that
codes for antibiotic resistance, toxins, and enzymes. The plasmid is transferred between bacteria through conjugation (or cell sex).
And the bacteria that gain the antibiotic-resistant DNA-encoding plasmid, can lose the plasmid just as easily. The plasmid is lost
during bacteria replication – when DNA is being replicated, the plasmid is not always replicated and passed on to subsequent
generations, especially if there are several generations of replication. The new bacteria that are no longer resistance and are
vancomycin sensitive, survive because the continued presence of the resistant bacteria protects them.

Take home point: Bacteria resistance is due to conjugation between bacteria and the transfer of plasmid, which contains the DNA
that codes for resistance. During subsequent bacteria replication, the plasmid can be lost if its DNA is not replicated and included in
subsequent generations.

13. Female newborn delivered at 25 weeks, neonatal respiratory distress syndrome, ABG shows decreased pH, decreased
Po2 increased PCO2. A deficiency in which of the following most likely caused the disorder?
- Dipalmitoylphosphatidylcholine


Why it’s the right answer: The diagnosis is neonatal respiratory distress syndrome due to premature birth, 25 weeks. The arterial
blood gas shows signs of a respiratory acidosis, with a decreased pH, decreased pO2 and increased pCO2 that is causing the acidosis.
The question is asking about the lecithin:sphingomyelin ratio that indicates a deficiency in surfactant. Premature birth leads to
respiratory distress because not enough surfactant has been produced yet. Surfactant is necessary because it decreases alveolar
surface tension and prevents the collapse of alveoli. Without surfactant, the alveolar surface tension increases and they collapse –
hence, respiratory distress. A marker of sufficient surfactant is the lecithin:sphingomyelin radio >1.5 in the amniotic fluid. Basically,
there needs to be enough lecithin in a >1.5 ratio with sphingomyelin. Therefore, in the case of respiratory distress, there is a
deficiency in lecithin. The most challenging part of this question for those who were able to identify that this is respiratory distress
syndrome, is that many test-takers may not know that dipalmitoylphosphatidylcholine is a type of lecithin. Types of lecithin that they
may try to trick you with usually end in the following: phosphatidylcholine, phosphatidylethanolamine, and phosphatidylinositol.

Take home point: Neonatal respiratory distress syndrome (RDS) is due to a deficiency of surfactant indicated by a ratio of
lecithin:sphingomyelin <1.5 in the amniotic fluid. Dipalmitoylphosphatidycholine is a type of lecithin that is deficient in RDS.

14. 54-year-old man with 12-hour history of intermittent severe pain in flank area radiating to genital region. History of
hypercalciuria and renal calculi. Drug decrease the urinary excretion of calcium?
- Hydrochlorothiazide

7 ©Test Pirates, LLC. All Rights Reserved.
Step 1 Practice Test #18 Explanations


Why it’s the right answer: Thiazide diuretics can cause hypercalcemia while loop diuretics can cause hypocalcemia (“Loops Lose
calcium”). Thiazide diuretics for this reason can be useful in osteoporosis and in patients who are more prone to renal calculi like in
this question. The patient’s presentation is consistent with a recurrent episode of kidney stones evidenced by flank pain radiating to
the genital or groin region. Using a thiazide diuretic would prevent the transport of calcium from the blood into the tubule of the
nephron, and into the urine. Less calcium in the tubule will prevent renal calculi made of calcium from forming. Stones made of
calcium represent 80% of all kidney stones. Aside from thiazide diuretics, citrate is also used to prevent calcium kidney stones
because it binds with calcium and prevents it from being excreted in the urine.

Take home point: Thiazide diuretics can be used to prevent the recurrence of calcium kidney stones because thiazides decrease the
excretion of calcium in the urine.

15. 54-year-old man comes to the physician because of an enlarging face shoulders and trunk and thinning of his arms
and legs. Physical examination shows a large plethoric face, fat pad over the upper thoracic spine and purple striae on
the abdomen. Serum studies show undetectable ACTH and an increased cortisol concentration. Administration of low
dose dexamethasone would most likely result in which of the following sets of serum findings?
- ACTH no change, cortisol no change


Why it’s the right answer: This question tests your knowledge of the dexamethasone suppression test. This patient, without a
doubt, has Cushing’s syndrome – plethoric face (moon facies), fat pad over back (buffalo hump) and purples striae are all signs and
symptoms of Cushing’s. That’s the easy part. The more difficult question is what is the cause of Cushing’s. There are many causes
and some of them can be ruled out based on the undetectable ACTH and increased cortisol levels – i.e. we can rule out an ACTH-
secreting pituitary or ectopic tumor. The negative feedback from the increased cortisol has suppressed ACTH, therefore, there must
either be a cortisol-producing tumor or steroids are being excessively used. When low-dose dexamethasone is used, cortisol levels
will decrease only in normal patients. When high-dose dexamethasone is used, normal patients and an ACTH-producing tumor will
demonstrate a suppression in the level of cortisol. In an ectopic ACTH-producing tumor or in a cortisol-producing tumor,
dexamethasone suppression test will leave cortisol levels unchanged since tumors that are producing these two hormones are
tumors and are unresponsive to feedback mechanisms.

Take home point: Low-dose and high-dose dexamethasone suppression tests will not suppress the elevated cortisol levels
associated with a non-pituitary ectopic ACTH-producing tumor or a cortisol-producing tumor because these tumors do not respond
to feedback mechanisms.

16. 14-year-old boy with mother concerned about enlarged left breast and family history of breast cancer. Left breast is
slightly larger than right, nipple mildly tender. Penis slightly enlarged and pubic hair curling and beginning to darken at
base. Best next step?
- Reassure the mother that physical findings are not uncommon for his age


Why it’s the right answer: First of all, breast cancer in young girls and boys is extremely, extremely rare. Secondly, these are normal
changes in a young boy going through puberty due to fluctuations in hormone levels. Estrogen is also increasing, even in boys, and
this increase in estrogen causes asymmetrical breast enlargement and nipple changes. The liver breaks down estrogen, and even
men have estrogen because when they develop cirrhosis, they also develop gynecomastia due to excess estrogen. The best next step
then in this case is to reassure the mother that these findings are normal. The boy is also meeting all the other stages of
development according to the Tanner scale, with an enlarging penis and pubic hair that is beginning to darken and curl.

Take home point: It is not uncommon for boys going through puberty to experience breast enlargement and nipple changes due to
an increase in all sex hormones including estrogen.

8 ©Test Pirates, LLC. All Rights Reserved.


Step 1 Practice Test #18 Explanations

17. 64-year-old woman who is an anesthesiologist has fractured femur during motor vehicle collision. 3 days after
admission, has tachycardia, restlessness, diaphoresis, anxiety, seeing "vague shapes" on walls. Dx?
- Alcohol withdrawal


Why it’s the right answer: This question on alcohol withdrawal is presented in a way that is not uncommon on standardized tests –
little information is given regarding this patient’s history of alcoholism, but when she is hospitalized, symptoms of withdrawal are
unmasked. The reason this question is presented like this without much history is that common things are common, and on day 2-3
of hospital admission, it is not uncommon for alcoholics (who may have even denied alcohol use on admission to the hospital) to
unmask their addiction by showing signs of withdrawal – a patient can’t drink in the hospital, so the patient withdraws. The time
course is also a giveaway here. Sometimes it can even take longer, especially if the patient is being medicated with low dose
benzodiazepines; even on benzos, it may not be a high enough dose and the patient will still experience withdrawal. With the GABA
receptor no longer agonized, withdrawal symptoms include cardiac signs of tachycardia and hypertension, as well as psychological
symptoms of anxiety, psychosis, and tactile hallucinations. Treatment is benzodiazepines, alcohol, anti-seizure medications and
seizure precautions, high blood pressure treatment, and other supportive care. Ultimately, these patients need to be referred to a
detoxification program.

Take home point: When a patient is admitted to the hospital and on day 2-5 starts to experience tachycardia, hypertension, anxiety
and hallucination, suspect alcohol withdrawal even when no history of alcohol use or abuse is given.

18. 27-year-old man with fever and sharp chest pain for 5 days. T 99.8F. Friction rub heard. All causes of secondary
pericarditis ruled out. Cause of primary pericarditis?
- Virus


Why it’s the right answer: There are several causes of pericarditis including bacterial, autoimmune (RA, SLE), metabolic (uremia),
and iatrogenic (post-radiation). When all treatable causes are rules out (almost all of these causes are treatable), it is likely that a
viral infection that led to the pericarditis, as viruses and bacteria are the most common cause of pericarditis. This patient has other
signs of a viral infection including fever. Viral pericarditis usually resolves spontaneously, so treatment is typically symptomatic and
consists of supportive care with NSAIDs.

Take home point: In idiopathic pericarditis (when no cause of the pericarditis can be clearly identified) the cause is typically viral. In
these cases, the pericarditis usually resolves spontaneously and treatment includes supportive care with NSAIDs.

19. 40-year-old man with severe back pain, gained 18 kg in the past 8 months, decreased peripheral vision, compression
fracture spine at T12 and L3, healing left rib fracture at T8, imaging shows pituitary macroadenoma. This tumor produces
which?
- ACTH


Why it’s the right answer: The most common pituitary adenoma is a prolactinoma. This patient clearly does not have a
prolactinoma, which would present with signs of galactorrhea, low libido, and in a female, amenorrhea. Prolactinomas do not cause
bone fractures either. What does cause a higher rate of fractures is osteoporosis, and osteoporosis can result from increased levels
of cortisol. Cortisol also causes weight gain, another change in the patient. The loss of peripheral vision is due to the
macroadenoma’s compression on the optic chiasm, resulting in a bitemporal hemianopsia characteristic of pituitary tumors. Another
common hormone secreted by a pituitary adenoma is growth hormone. However, this patient does not show signs of increased GH
like a large tongue, deep voice, course facial features, and insulin resistance. This tumor must, therefore, be producing ACTH,
causing the release of cortisol from the adrenal glands. A final tangential but important point to make is that when osteoporosis
causes fractures, compression fractures of the spine are usually presented. A question may describe an anorexic patient with

9 ©Test Pirates, LLC. All Rights Reserved.


Step 1 Practice Test #18 Explanations

compression factures and the cause is osteoporosis. I remember this by thinking of the bones being so fragile from the osteoporosis,
that even the spine spontaneously collapses and fractures.

Take home point: Pituitary adenomas that release ACTH cause increased levels of cortisol resulting in weight gain, osteoporosis
(which can lead to bone fractures, especially spinal compression fractures), and other consequences of increased cortisol.

20. 27-year-old woman passed out while washing dishes. Began to feel weak and dizzy, husband shares with nursing staff
history of bulimia nervosa, and has been binging more frequently. Suspected she has been vomiting. Labs of patient?
K+/HCO3-/Anion Gap/pH
- decreased/increased/normal/increased


Why it’s the right answer: Emesis in general leads to a metabolic alkalosis. The reason for this being that in vomit, gastric acid, or
HCl, is lost in the emesis. With the loss of acid, an alkalosis results. Fluid is also lost, resulting in a contraction alkalosis, which is
when, in attempt by the kidneys to preserve water and NaCl to raise blood pressure, bicarb is also reabsorbed and H+ is excreted,
resulting in an alkalosis. Potassium is also lost because for every Na+ reabsorbed by the kidneys, a K+ is excreted. Finally, there is no
anion gap because there are no extra anions being added to or taken away from the system to account for the metabolic alkalosis.

Take home point: Emesis causes a hypokalemic metabolic alkalosis due to the loss of HCl in the emesis and the excretion of K+ by
the kidney in an attempt to absorb more Na+ and conserve fluid.

21. 72-year-old man with 4 months’ fatigue and shortness of breath. Vitals HR 88, RR 14, bp 160/95. PE conjunctival
pallor. Labs: hb 9.0, hct 29%, MCV 92, RDW 13.2% (N=13-15%), Cr 3.1, Ferritin 154, Iron 30, Transferrin saturation 34%
(N=20-50%). Besides iron supplementation, most appropriate tx?
- Erythropoietin


Why it’s the right answer: This patient has a normocytic anemia based on his low hemoglobin and hematocrit and an MCV within
normal limits. He is also symptomatic with fatigue and shortness of breath due to anemia. The cause of the anemia is kidney disease
based on the patient’s elevated creatinine. In kidney disease, the kidney parenchyma and interstitium is damaged and starts to fail
to produce erythropoietin (EPO). EPO stimulates the bone marrow to produce RBCs. In kidney disease, standard treatment includes
the supplementation of EPO. This patient also has low iron, normal serum ferritin, and normal transferrin saturation, so iron should
also be replaced with iron supplementation. Iron deficiency is not, however, the primary cause of this patient’s anemia, and would
present as a microcytic anemia (MCV< 80).

Take home point: The cause of a normocytic anemia in kidney disease is low erythropoietin (EPO) since EPO is produced by the
kidneys. EPO should therefore be supplemented to treat the anemia.

22. 13-year-old girl comes with lack of improvement of facial acne. Tried topical clindamycin and benzoyl peroxide PE
severe acne vulgaris. Asks, "Can you help me?" She denies sexual activity. Tx?
- Isotretinoin


Why it’s the right answer: Although isotretinoin is highly teratogenic, and before beginning use, the patient should have a
pregnancy test even though she says she has never been sexually active, it should be third or even second line in treatment-resistant
acne. The first-line of treatment for acne typically starts with benzoyl peroxide, so if she hadn’t tried this already, that would be the
answer. Next, topical clindamycin or even an oral antibiotic can be tried in conjunction with continuing the benzoyl peroxide since
these antibiotics have anti-inflammatory properties and can clear up an infection that may also be causing the acne. If these
treatments do not work, then a trial of topical or oral Isotretinoin, a vitamin A derivative, can be tried, especially in cases of large
10 ©Test Pirates, LLC. All Rights Reserved.
Step 1 Practice Test #18 Explanations

comedones and cystic acne. Oral isotretinoin is indicated when the acne is over a large surface like a patient’s back, and the
application of a topical medication is not practical.

Take home point: Isotretinoin, a vitamin A derivative, should be used as second or third-line treatment of treatment-resistant acne,
in cases of severe cystic acne, and when acne is spread over a large surface like a patient’s back. Pregnancy should always be
checked because this drug is highly teratogenic.

23. 2-year-old girl with 3-day progressive cough and hoarseness. T 101.7F, HR 85, RR 26, bp 110/60. Oxygen saturation
94%. PE mild erythema of oropharyngeal and laryngeal mucosa, no exudate, harsh, barking cough heard. Improves
within 5-6 days. Cause?
- Parainfluenza


Why it’s the right answer: The diagnosis is croup, also known as laryngotracheobronchitis, caused by parainfluenza virus type 1,
which is in a family of viruses called paramyxoviruses. Paramyxoviruses also include RSV (causes bronchiolitis in infants), mumps and
measles. Croup presents as a sudden onset of acute inflammation in the larynx, leading to a harsh, “barking” cough, which can
progress to respiratory stridor. This patient already has a decrease in oxygen saturation secondary to oropharyngeal inflammation
and swelling, limiting respiration. This viral illness typically improves on its own sans treatment aside from symptomatic therapy.

Take home point: Croup, aka laryngotracheobronchitis, is a disease in children caused by parainfluenza virus type 1, a type of
paramyxovirus, and is characterized by sudden onset of symptoms with a harsh, barking cough that can lead to inspiratory stridor.

24. 42-year-old man 2-week shortness of breath with exertion, light palpation of carotid artery shows upstroke is
abnormally brisk and downstroke falls precipitously. Cause of finding?
- Aortic Regurgitation


Why it’s the right answer: This patient has what is described as bounding pulses – brisk upstroke and fast, precipitous downstroke
on palpation of the carotid artery. There are only a few things that cause bounding pulses and shortness of breath with exertion, and
aortic regurgitation (AR). The reason for this is that during upstroke, all the blood from the heart plus the blood that fell back
through the aortic valve is pumped up and through systemic circulation. Then when the aortic valve closes (S2), blood leaks back
through the valve and into the left ventricle, resulting in a precipitous fall in the pulsation and also a collapse of the blood vessel as
the blood falls back into the heart and pulls more distant blood with it. AR presents as a decrescendo, diastolic murmur that begins
right after S2 is heard. It can also present with head bobbing, again, due to the bounding pulses. The cause of AR for this patient is
not clear. Causes of AR include aortic root dilation (i.e. from syphilis, Marfan’s, which this patient could have), bicuspid aortic valve,
endocarditis, or rheumatic fever.

Take home point: Aortic regurgitation is associated with head bobbing and bounding pulses, or an abnormally brisk upstroke and
fast downstroke.

25. Retrospective study of incidence and outcome of subdural hematoma. Incidence is 15 per 100,000 people, mortality is
6 per 100,000 people. Incidence remains constant, case fatality rate?
- 40%


Why it’s the right answer: The case fatality rate is the number of deaths due to a disease divided by the number of cases of the
disease. Put another way, it is the proportion of deaths due to a disease out of the incidence of the disease. The incidence is
constant meaning the total number of people and the total number of people with the disease (the incident cases) remains the

11 ©Test Pirates, LLC. All Rights Reserved.


Step 1 Practice Test #18 Explanations

same. Therefore, we can calculate the case fatality rate by dividing the number of deaths (6) by the number of cases (15). 6/15 = 0.4,
or 40%

Take home point: The case fatality rate is the number of deaths due to a disease divided by the number of cases of disease over the
disease course. If the incidence is constant, take the number of deaths divided by the number of cases = case fatality rate.

26. 70-year-old with non-Hodgkin lymphoma (NHL) and 4-day history of abdominal pain and nausea. T 98.6F, HR 110, bp
140/90. Abdominal exam tenderness of flanks and lower quadrants. BUN 37 and creatinine 3.9. CT shows bilateral
hydronephrosis and lymphadenopathy compressing ureters. Tx to improve renal function?
- Bilateral stents in the ureters


Why it’s the right answer: This patient has lymphoma with lymphadenopathy and secondary hydronephrosis due to ureter
compression in the retroperitoneal space, where the ureters are located and where lymph nodes are also located. The patient is
suffering from post-renal acute kidney injury based on his BUN:Cr <10, and his 4-day history of abdominal pain and flank tenderness
on exam. To prevent permanent damage to the kidneys, bilateral ureter stents should be placed (consult urology!) in order to
properly drain the kidneys and improve renal function back to baseline. Long-term treatment would include treatment of the NHL
with chemotherapy, radiation, or both, which may improve the lymphadenopathy and relieve the pressure on the ureters. In the
interim however, again to prevent permanent kidney damage, stents must be placed bilaterally.

Take home point: In post-renal acute kidney injury, the post-renal block must be relieved, and in the case of a compressive tumor or
lymphadenopathy, bilateral stents in the ureters offer the best solution.

27. 82-year old woman with coronary disease and well-controlled hypertension is brought to the physician 4 hours after
sudden onset of weakness of her left leg. Her BP 140/75 mmHg. Neurologic exam: weakness and decreased sensation
over the left lower extremity. There are no other sensory or motor deficits. Which labeled structure is site of injury?
- A


Why it’s the right answer: This patient has signs of both motor and sensory deficits. There are only few regions in the brain that
would cause both deficits, and the labeled region straddles the central gyrus. When we have a sagittal (or side view) of the brain, the
central gyrus divides the frontal lobe and parietal lobe, and also divides the precentral gyrus and postcentral gyrus. The precentral
gyrus is the principal motor region and the postcentral gyrus is the principal sensory region. When we view the brain through a
coronal cut in order to visualize the homunculus, we can identify exactly which area of the brain controls the region in question.
Here the patient has leg weakness, so the lesion must be more superiorly where the leg is located at the top of the brain, or the top
of the homunculus. If the question was about arm or hand weakness, we would move further down the homunculus or further down
the side of the brain to locate the lesion.

Take home point: The central gyrus visualized on a side view of the brain divides the pre- and postcentral gyri. The precentral gyrus
is in charge of principal motor function, while the postcentral gyrus controls principal sensory function.

28. Girl has chronic cough with thick sputum, abdominal cramps in RLQ, and frequent resp infections. Clubbing of fingers,
hyperresonance on chest percussion. Diffuse crackles and scattered wheezes. On xray - diffuse hyperinflation of the right
upper lobe. She has a healthy brother. What's the likelihood he is a carrier for the condition?
- 2/3

12 ©Test Pirates, LLC. All Rights Reserved.


Step 1 Practice Test #18 Explanations

Why it’s the right answer: The diagnosis is cystic fibrosis (CF), an autosome recessive genetic disease. Her symptoms are
characteristic of cystic fibrosis – thick sputum with chronic cough, frequent respiratory infections (these patients are at particular
risk for pseudomonas infections), abdominal pain secondary to decreased fat absorption from pancreatic insufficiency, and finger
clubbing from chronic hypoxemia. In terms of the genetics, this patient must have both recessive genes to have CF, so each parent
must be a carrier since you inherit one gene copy from each parent. If we create the Punnett square, we have the following possible
outcomes if both parents are carriers: CC, Cc, Cc, and cc. The daughter has cc. The brother does not have CF, so he cannot have cc,
ruling out that possibility. Therefore, we’re only left with three possibilities in total, and two of the three are carriers. Therefore, the
correct answer is 2/3 likelihood for being a carrier for CF.

Take home point: Cystic fibrosis is an autosomal recessive disease. When calculating the likelihood of a sibling being a carrier,
remove the possibility of the sibling having the disease, since we know the sibling doesn’t have it. Then calculate the likelihood out
of 3 possibilities instead of 4 possibilities in total.

29. 6-year-old male with recurrent UTIs, left kidney found small and nonfunctional; right is normal. Nephrectomy is done,
and the picture shown (dilated ureter and renal calyx). Microscopic exam of kidney will show which?
- Tubular atrophy


Why it’s the right answer: This is a young patient with hydronephrosis. The two most common cause of hydronephrosis in infants
and children are pelvic-urethral obstruction and vesicourethral reflux. In this case, the patient likely has hydronephrosis secondary
to vesicoureteral reflux, since reflux also causes repeated urinary tract infections. An easy way to remember this association is that
the urine refluxing from the bladder back into the ureters results in urine buildup and stasis – and any time there is stasis there is
risk for the development of an infection or UTI. Moreover, this buildup of urine backs all the way up to the kidney and begins to
dilate the renal pelvis, causing hydronephrosis. The picture of the gross kidney a dilated ureter and renal calyx due to the buildup of
urine. And where does urine come from in the first place? The renal tubules. Therefore, the urine has backed all the way up into the
tubules and puts pressure on them, causing them to atrophy.

Take home point: Hydronephrosis causes tubular atrophy from compression of the tubules by urine. Two common causes of
hydronephrosis in infants and young children include pelvic-urethral obstruction and vesicourethral reflux.

30. Patient with orthostatic hypotension, loose stools for 6 months, and history of type 1 DM. Stool studies are normal.
What is the pathophysiological mechanism of the diarrhea?
- Motility disorder



Why it’s the right answer: This patient has autonomic degeneration due to diabetic neuropathy. Diabetes results in small vessel
disease – leading to retinopathy, nephropathy and neuropathy, because all these organ systems rely on the function of small vessels.
In terms of neuropathy, more commonly diabetics will present with motor and sensory deficits, and as a result, diabetic foot ulcers
à infection à amputation. Diabetics will also have degeneration of their autonomic nervous system. This patient shows signs of
autonomic neuropathy from the hypotension and the loose stool, since the autonomic nervous system prevents hypotension and
also controls the movement of food in the gut. Thus, the underlying mechanism is a motility disorder, or decreased movement and
peristalsis in the gut from autonomic degeneration.

Take home point: Diabetics suffer from small vessel disease that results in retinopathy, nephropathy, and neuropathy. The
neuropathy that results affects motor, sensory, and autonomic nerves. In autonomic degeneration, patients can present with
constipation/diarrhea due to decreased peristalsis of the gut. Orthostatic hypotension is another result of autonomic dysfunction.

13 ©Test Pirates, LLC. All Rights Reserved.


Step 1 Practice Test #18 Explanations

31. In a 30-year-old patient who underwent splenectomy, target cells are seen on peripheral smear. These cells are seen
due to loss of function of which portion of spleen?
- Red pulp


Why it’s the right answer: The red pulp of the spleen is made of connective tissue and blood vessels (hence the name “red pulp”)
called the cords of Billroth. Its primary function is to filter the blood of antigens and old red blood cells. The white pulp is made of
lymphoid tissue (“white pulp”) and contains T cells in the periarterial lymphatic sheath (PALS), germinal centers/follicles made of B
cells, and antigen presenting cells (APCs) in the marginal zone between the white and red pulp.

The target (also known as codocytes) are old blood cells that are normally removed by the red pulp of the spleen. They obtain a
target-like appearance due to the loss of hemoglobin and loss of volume relative to their surface membrane. If the spleen is
removed, then target cells are not removed by the spleen and can be seen on a peripheral blood smear. They have little function in
terms of oxygen delivery to the vital organs. Target cells can also be seen in HbC disease, liver disease, thalassemia (alpha and beta),
and autosplenectomy (i.e. sickle cell disease).

Take home point: The red pulp of the spleen filters and removes old RBCs. Target cells, or cells that are “worn out” with loss of
hemoglobin and cell volume, remain in circulation in asplenia patients since there is no spleen to remove them. Target cells also
occur liver disease, HbC disease, and the thalassemias.

32. 65-year-old man comes to the physician because of a 4-year history of recurrent cough productive of increased
sputum. Use of over-the-counter cough suppressants has not resolved his symptoms. He has smoked 2 packs of
cigarettes daily for 50 years. He has no family history of lung disease. His temperature is 37°C (98.6°F), pulse is 80/min,
and respirations are 15/min. Physical examination shows cyanosis. Diffuse wheezing is heard on auscultation. Which of
the following pulmonary cell types is most likely to be abnormal in this patient?
- Pseudostratified columnar epithelial cells


Why it’s the right answer: The diagnosis is chronic obstructive pulmonary disease (COPD) based on the patient’s extensive smoking
history (2 packs for 50 years = 100 pack years!), long-history of productive cough (qualified as chronic bronchitis type of COPD),
cyanosis on physical exam, diffuse wheezing, and I am certain if his pulse oximetry was checked it would be low – otherwise he
wouldn’t be chronically hypoxemic and cyanotic! This question is tricky because it’s asking about the pathophysiology of COPD, and
here you have to know that COPD destroys the terminal bronchioles first and the most extensively. Remember the respiratory tree:
trachea à bronchi à bronchioles à terminal bronchioles à respiratory bronchioles (where alveoli begin to branch off of) à
alveolar ducts à alveoli. The particles inhaled and the damage created in patients with COPD affects the terminal bronchioles to the
greatest degree, possibly because there is less surface area in the terminal bronchioles than the respiratory bronchioles so there is
more exposure of carcinogens/smoke per square unit of area within the terminal bronchioles. Knowing this, you also have to know
that the terminal bronchioles are lined by pseudostratified ciliated columnar epithelial cells; respiratory bronchioles are lined by
mostly cuboidal cells and alveoli are lined by simple squamous cells.

Take home point: COPD renders the most damage to the terminal bronchioles (right before the respiratory bronchioles), which are
lined by pseudostratified ciliated columnar cells (vs. cuboidal cells that line the respiratory bronchioles).

33. Researching new cancer drug, effective at killing rapidly dividing cells, in mice caused profound myelosuppression. In
patients, most appropriate to follow which when at risk for infectious complications?
- Neutrophil counts

14 ©Test Pirates, LLC. All Rights Reserved.


Step 1 Practice Test #18 Explanations

Why it’s the right answer: In patients on chemotherapy, the absolute neutrophil count (or ANC) is trended to monitor the risk for
infection because neutropenia is the most common predisposing factor to infection in cancer patients. Myelosuppression agents will
suppress all type of WBCs including lymphocytes as well, but the ANC still remains the strongest indicator for infection risk in
patients on immunosuppressive chemotherapy.

Take home point: Follow the ANC, or absolute neutrophil count, in cancer patients receiving immunosuppressive therapy to assess
infection risk.

34. 33-year-old recently diagnosed with lupus and difficulty working as cashier because of severe pain and swelling of
joints. Current meds include prednisone, hydroxychloroquine, and oxycodone for pain. Best action to adapt to illness?
- Encourage the patient to participate in a support group for persons with her condition


Why it’s the right answer: When “support groups” is among the answer choices, unless the answer that contains support groups is
completely incorrect, it is usually the correct answer. Support groups are an important adjuvant to treating disease because several
of these lifelong diseases come with drastic life changes, coping, and treatments with side effects. In this case, if the patient is having
joint pain and swelling, it may be a good idea to address her pain first and prescribe another DMARD to improve her disease control.
However, there a few key words in the question stem that points us towards encouraging her to also join a support group. First, she
was recently diagnosed with lupus, meaning she is new to the disease and should speak with other people diagnosed with lupus
regarding disease coping, treatment, and remission. Another key point is that the question asks, what is the best action to help her
adapt. Again, adaption requires having a support group at home and also within a group of people suffering from the same
condition.

Take home point: In helping patients adapt with a recently diagnosed lifelong disease, referral to support groups is almost always
the correct answer.

35. 22-year-old man with gastrointestinal bleeding. Laparotomy done and 5-cm blind outpouching on antimesenteric side
of terminal ileum 13 cm from ileocecal valve resected. Pathology shows?
- Heterotopic gastric mucosa


Why it’s the right answer: This diagnosis is Meckel’s diverticulum due to the persistence of the vitelline duct. It causes symptoms of
melena, or GI bleeding from the upper GI tract, typically within the first two years of life, so the age of this patient is slightly older
than typical. It is commonly about 2 inches (5 cm) long and can contain heterotopic tissue (tissue that is misplaced and not in the
correct location). Most commonly this tissue is acid-secreting gastric mucosa, pancreatic tissue, or both. Therefore, pathological
studies of the resected tissue would show heterotopic gastric mucosa.

Take home point: Meckel’s diverticulum can contain heterotopic tissue of gastric or pancreatic origin.

36. 21-year-old man with syncopal episode, fever, nausea, muscle aches, progressive confusion. T 102.9F, palpable bp 70.
PE rash on lower extremities. WBC 28,000. Blood and CSF cultures grow oxidase positive, gram negative diplococcus.
Brother with similar infection at same age. Immune disorder?
- Late component of complement deficiency


Why it’s the right answer: The infection is Neisseria bacteremia, an oxidase positive, gram negative diplococcus that can cause a
severe meningitis (evidenced in this patient by positive CSF cultures, fever, confusion, syncope) and blood infection (sepsis
evidenced by high fever, high WBC, and low BP 70) is individuals, especially those with weakened immune systems. In particular,
those who are deficient in complement are at risk of developing fulminant infections (like the one described in the question stem)
15 ©Test Pirates, LLC. All Rights Reserved.
Step 1 Practice Test #18 Explanations

from gram negative bacteria because the membrane attack complex formed by complement factors defends against gram-negative
bacteria. In this patient, he is specifically deficient in one of the late complement factors (C5-9), since deficiencies in the earlier
factors cause other disorders. For instance, patients with C3 complement deficiency are predisposed to recurrent pyogenic sinus and
respiratory infections; they also have an increased susceptibility to type III hypersensitivity reactions.

Take home point: Deficiency in C5-C9 complement creates a predisposition to gram negative infections, in particular a fulminant
infection from Neisseria leading to meningitis and sepsis.

37. Randomized controlled trial evaluating tx of pharyngitis. No statistically significant differences found between
patients receiving the antibiotic and those with standard tx. Which aspect results in type II error?
- Number of subjects in the study


Why it’s the right answer: Think of type II error (or beta, or false-negative error) as a person who was found to be innocent but
really was guilty. Type II error means just that – no significant findings were found by the study, but in reality, there are significant
findings and a difference between the new antibiotic and the standard treatment really does exist. Type I error (or alpha, or false-
positive error) is finding a difference between the two therapies, when the difference does not actually exist. Compare to our
criminal analogy – finding a person guilty, when in reality he/she is innocent!

Type II error is influenced by the number of subjects in the study, or the power. Logically, this makes sense: If there are only a few
patients enrolled in the pharyngitis antibiotic trial, and only a few of the new treatment group patients experience improvement
compared to the standard of care, well when you run this through a statistical analysis, those “few” may not be enough to prove
your point. Then, you go and enroll 1,000 in the new treatment group and 1,000 in the standard treatment group. Your statistician
will love you because if there is a true difference, you will surely have the power (or a high number of subjects in the study) to
support it. There’s also the equation: power = 1 – type II error (beta). The greater the power is, the smaller the type II error.

Take home point: Type II error (beta, false-negative error) is affected by power, or the number of subjects in a study.

38. 16-year-old boy whose mother concerned about physical development. 15th percentile for height and 20th for
weight. FSH is 6, testosterone 9 (N=10-35). Physical finding?
- Gynecomastia


Why it’s the right answer: This question is tricky if you think about in too much depth. Based on the low testosterone, one would be
tempted to lean towards a diagnosis of Klinefelter’s syndrome and choose gynecomastia, which is this case, gynecomastia is correct
so you wouldn’t necessarily be wrong. But the question stem really doesn’t give enough information to support Klinefelter’s, his FSH
is not high as you would except in this syndrome, and his height is in the lower percentile, which is the opposite of Klinefelter’s – tall
with long extremities. What this question is trying to show is that this boy is a bit behind in development – he’s in the lower weight
percentile, lower height percentile, and he has slightly low testosterone. For these reasons, all the other answer choices except
gynecomastia do not fit with someone who has low levels of testosterone. The remaining answer choices all support normal to high
levels of testosterone. I think this patient is a bit delayed in puberty and development, and has gynecomastia because it’s not
uncommon for boys during puberty to develop enlarged breasts due to estrogen-testosterone imbalance. Decreased levels of
testosterone increases the release of LH, which in turn, increases estrogen. This could even be anabolic steroid use, which would
suppress FSH, testosterone, stunt growth, and cause gynecomastia. Or it’s possibly secondary hypogonadism, in which case, the
patient would still have low testosterone, an estrogen-testosterone imbalance, and resulting gynecomastia. A final point is that the
imbalance of hormones is not due to testosterone creating estrogen. The low testosterone signals an increase in LH, which in turn
creates estrogen from other sources in the body. Annoying question, but this is the nature of the beast.

Take home point: In males going through puberty, gynecomastia is not uncommon, especially in those with delayed onset of
puberty and low levels of testosterone.

16 ©Test Pirates, LLC. All Rights Reserved.
Step 1 Practice Test #18 Explanations

39. To decrease risk for cv disease 30 yo man begins diet. 99 kg BMI 35, Intends to lose 20 kg by limiting caloric intake to
2000 cal. to maintain the recommended protein intake (56g day); a balanced decreased in carbs and fat is required
(caloric radio of fat and carbs is 30:55). which best describes number of calories that should be provided by fat in this its
diet each day?
- 630


Why it’s the right answer: I know, I know, you didn’t expect to have to know the number of calories of protein, carbs, fat. Well now
you know it’s fair game for the test. At least you know now! The numbers are actually not that bad, so here they are: 1g carb = 4
calories, 1g protein = 4 calories, 1g fat = 9 calories, 1g alcohol = 7 calories. Now for the math. Remember, they know you are limited
on time, so the math is not going to be that intense. The answer is even asking for the calories, so we don’t have to convert the
answer we obtain in calories bag to grams of fat. And we also don’t need to think about the fact that he wants to lose 16kg because
they gave you his caloric limit = 2000 calories. First, let’s get the protein out of there: 56g of protein x 4calories/g = 224 calories from
protein. Subtract this from 2000 = 1776 calories, which is the amount of fat and carb calories. Next just make two equations with
two variables: x = fat calories, y = carb calories
x + y = 1776
30/55 = x/y
Then solve for x: x = 626 fat calories à round up to 630 calories

Take home point: This as hard as the math gets on the test. Remember 1g carb = 4 calories, 1g protein = 4 calories, 1g fat = 9
calories, 1g alcohol = 7 calories.

40. 26-year-old woman with palpable lump in right breast 6 months, no pain, swelling or nipple discharge. Lump is
smooth, firm, round, mobile, non-tender, well delineated. No skin change. Increasing in size. Dx?
- Fibroadenoma


Why it’s the right answer: This is a fibroadenoma until proven otherwise based on this description that pretty much spells out the
word fibroadenoma in the question stem. This is a fibroadenoma because it’s mobile, that’s probably the strongest key word that
makes this what it is. Other findings are consistent with a fibroadenoma as well – no pain, no swelling or discharge (so not an
intraductal papilloma which could present with discharge), and it’s firm and round. If it was not round or mobile, it would likely be
fibrocystic change (even more so if the mass became larger and more sensitive during menstruation, findings very characteristic of
fibrocystic change). Breast cancer is extremely, extremely rare in women <30 years, so that is almost always the incorrect answer in
a young person – it’s not even a “zebra” that is testable.

Take home point: A firm, round, mobile, painless breast mass in a female is almost always going to be a fibroadenoma until biopsy
proves otherwise.

41. 53-year-old woman comes to the physician because of a 2-week history of a swollen, painful left knee. Her
temperature is 38°C (100.4°F), and blood pressure is 115/70 mm Hg. Examination of the left knee shows erythema and
swelling of the joint and decreased range of motion. A photomicrograph of synovial fluid obtained by arthrocentesis is
shown. This patient's synovial fluid most likely contains which of the following?
- Uric acid


Why it’s the right answer: This patient has gout and the synovial fluid would show elevated uric acid with urate crystals, which are
needle-shaped and are negatively birefringent. When you shine line through any crystal it changes the wavelengths and therefore
the color reflecting back. Negative birefringence means that the light is yellow when the crystal is parallel to the light. Positive
birefringence means that the light is blue when the crystal is parallel to the light. This is the case for pseudogout, though the calcium
17 ©Test Pirates, LLC. All Rights Reserved.
Step 1 Practice Test #18 Explanations

pyrophosphate crystals are rhomboid-shaped and are weakly positive birefringent. The patient’s symptoms are consistent with gout
– she has unilateral joint swelling, pain, and erythema. High uric acid levels in the serum can precipitate a gout attack such as the
intake of alcohol because alcohol completes with uric acid for excretion from the kidney.

Treatment is another popularly asked question regarding this topic because there is a difference between the acute treatment and
the chronic or preventative treatment. Acutely, indomethacin is always the answer even if colchicine is there in the answer choices.
Colchicine and glucocorticoids (or steroids) are also used acutely, but indomethacin and NSAIDs are first line. Preventative treatment
includes xanthine oxidase inhibitors such as allopurinol, febuxostat, and probenecid. First-line for these treatments is typically
allopurinol before trying the other two.

Take home point: Gout typically presents as unilateral joint pain, erythema and swelling. Synovial fluid is characteristic for needle-
shaped urate crystals from elevated uric acid levels.

42. 44-year-old exposed to possible chemical attack. Respirations labored, diaphoresis, excessive lacrimation, increased
salivation, muscle strength 1/5, urinary and fecal incontinence. Besides atropine, another tx?
- Pralidoxime


Why it’s the right answer: This patient has cholinesterase inhibitor poisoning due to organophosphates with symptoms of impaired
breathing due to bronchospasm, diaphoresis, excessive lacrimation or crying, excessive salivation, decreased muscle strength, and
urinary and fecal incontinence – which indicates the poisoning is extreme if continence is affected. One such organophosphate that
can cause the poisoning is parathion. It irreversibly inhibits acetylcholinesterase, preventing the breakdown of acetylcholine. Always,
the answer for the antidote is atropine. If both pralidoxime and atropine are among the answer choices, pick atropine! Pralidoxime
regenerates active acetylcholinesterase to breakdown acetylcholine. But first atropine, and then pralidoxime. If atropine is not
among the choices or already given, pralidoxime is then the correct answer.

Take home point: Cholinergic poisoning with organophosphates is treated with atropine and pralidoxime. The mechanism of
pralidoxime is to regenerate acetylcholinesterase.

43. 89-year-old man admitted for prostatic resection. Normal mental status. 1 day after surgery, confused and restless.
Cannot sleep and restless, seeing little men coming through window. Dx?
- Delirium


Why it’s the right answer: It is not uncommon for the acute hospitalization of an elderly patient to precipitate an acute episode of
delirium, especially in the setting of someone who already suffers from dementia. And typically these patients usually present at
night, which is referred to as sun-downing. This is more likely delirium than dementia because the presentation is acute with a
dramatic change in demeanor (dementia is progressive over months to years). Also, the visual hallucinations further support a
diagnosis of delirium. This episode of mental status change could also be confused for alcohol withdrawal, however, this patient had
no other symptoms to support alcohol withdrawal like cardiac stress (high BP, high HR), and hallucination are more commonly tactile
(i.e. crawling bugs) than visual.

Take home point: Delirium can present during an episode of hospitalization in an elderly patient and is typically evidenced by acute
changes in mental status, confusion, and visual hallucinations.

44. 42-year-old woman has new neuro finding while being tx for acute infection of sphenoid sinus. Imaging shows
cavernous sinus thrombosis on right. Additional finding most likely?
- Inability to abduct the eye

18 ©Test Pirates, LLC. All Rights Reserved.
Step 1 Practice Test #18 Explanations


Why it’s the right answer: The cavernous sinus is full of venous blood that drains from the face and frontal skull. This venous blood
can become static and a thrombosis can form, resulting in a cavernous sinus thrombosis. A question on cavernous sinus anatomy is
often presented as a cavernous sinus thrombosis because they’re basically asking, which of following answer choices is a structure in
the cavernous sinus. In this case, one of the structures is the abducens nerve (CN VI), which controls abduction of the eye. The
deficit in abduction would be ipsilateral to the side of the cavernous sinus that is thrombosed. The other structures include the
pituitary gland in the center, the internal carotid artery, CN III, CN IV, CN V1, and CN V2.

Another important point is that not all of the structures in the cavernous sinus have to be affected. The question may just list a few
symptoms that indicate CN VI, CN V2 and CN III lesions. This could still indicate cavernous sinus pathology and warrants
investigation, namely, imaging. Finally, cavernous sinus pathology is not always from venous stasis and a thrombosis. A c-c fistula
(cavernous sinus-carotid artery) and a mass can also cause symptoms similar to that of a thrombosis, indicative of a lesion in the
cavernous sinus.

Take home point: Know the structures of the cavernous sinus. Cavernous sinus pathology such as a thrombosis can affect some or
all of the structures within the sinus.

45. 55-year-old woman visits the emergency department because of a 3-day history of fever, flank pain, pain with
urination, and nausea. Seven days ago, she was admitted to the hospital for similar symptoms and was diagnosed with
acute pyelonephritis. She was discharged with instructions to take oral ciprofloxacin after a 3-day course of intravenous
ciprofloxacin resulted in improvement. She also has hypertension, hyperlipidemia, and osteoporosis. Current medications
also include alendronate, calcium carbonate, ezetimibe, hydrochlorothiazide, and simvastatin. Her temperature is 39.2°C
(102.6°F), and blood pressure is 125/79 mm Hg. The most likely cause of this patient’s current condition is an interaction
between her current oral antibiotic and which of the following medications?
- Calcium carbonate


Why it’s the right answer: Here comes some organic chemistry. Honestly, it’s not completely necessary to know what I am about to
say as the explanation for the purposes of answering this question. In general, calcium we know becomes a cation (Ca++) when
ingested, and those positive charges mean it can bind things. Clearly this patient still has pyelonephritis and it’s like she never took
the ciprofloxacin, so something must have stolen it away. That’s the calcium! The calcium bound the ciprofloxacin and prevented its
absorption. In more scientific terms, ciprofloxacin has a carboxyl group that becomes ionized when ingested, and you guessed it, the
charge is negative! Positive and negative ions attract, so once the calcium binds to the ciprofloxacin molecule, absorption drops and
it’s as if the patient never even took the antibiotic.

Take home point: Calcium carbonate can decrease the effectiveness of medications including ciprofloxacin because of the positive
charge of the cation calcium (Ca++).

46. 47-year old man is brought to the ER semiconscious and combative. In addition to sedation, a short-acting
neuromuscular blocking agent is administered for intubation to prevent aspiration. Within a few seconds after admin of
the drug, he has transient muscle fasciculations in his face; he develops generalized paralysis within 1 minute. Two hours
after completion of the procedure, he is still paralyzed. A genetic abnormality of which of the following enzymes is the
most likely cause of his unusually slow recovery from paralysis?
- Pseudocholinesterase


Why it’s the right answer: Everything about this patient is normal (well, except for him being combative), up until the point he still
remains paralyzed after the anesthesiologist is ready to wake him up. He likely received a neuromuscular blocking agent such as
succinylcholine, a strong acetylcholine receptor agonist that causes muscle paralysis. Normally, when the succinylcholine is stopped

19 ©Test Pirates, LLC. All Rights Reserved.


Step 1 Practice Test #18 Explanations

after the procedure is completed, acetylcholinesterase and pseudocholinesterase (also an enzyme that breaks down acetylcholine,
but slower than acetylcholinesterase) break down the excess acetylcholine that has accumulated during paralysis. With a deficiency
in pseudocholinesterase (1 in 1500-2500 births, Males>Females), there is prolonged paralysis due to the slower breakdown of
acetylcholine. Interestingly, patients with this enzyme deficiency who use cocaine are more likely to suffer from cardiac arrest since
the cocaine acts more strongly in these patients.

Take home point: Prolonged paralysis during the use of a neuromuscular blocking agent for the purposes of anesthesia can be due
to a deficiency in the enzyme pseudocholinesterase.

47. Full-term newborn in respiratory distress. Imaging shows abdominal contents in left pleural cavity. Maldevelopment
of which structure led to diaphragm defect?
- Left pleuroperitoneal membrane


Why it’s the right answer: This is a congenital diaphragmatic hernia in which the stomach and intestine of the abdomen migrate
through a defect in the diaphragm into the pleural cavity and compress the lung. It should always be suspected in a newborn with
respiratory stress that may otherwise be healthy and full-term. This is also a favorite question for imaging, which shows pockets of
air and intestine in the left pleural cavity on a plain chest x-ray. There may even be air-fluid levels in the lung space further
confirming that intestine is in the pleural cavity. Typically, the left side will be affected and not the right because the diaphragm on
the right is protected by the liver. The structure that contains the defect is the pleuroperitoneal membrane, which forms the
diaphragm in utero. Finally, trauma can also cause diaphragmatic rupture, again of the left side more commonly, with a similar
presentation.

Take home point: Failure of the pleuroperitoneal membrane to completely develop in utero can lead to diaphragmatic herniation of
stomach and intestine into the pleural space and lung compression, commonly of the left side only.

48. 78-year-old with arteriosclerosis undergoes repair of infrarenal abdominal aortic aneurysm. Graft extends just below
the renal arteries to the bifurcation of the aorta. Which organ will lose primary blood supply and rely on collateral
circulation?
- Descending colon


Why it’s the right answer: This is an anatomy question about the blood supply to the gut and the branches of the descending aorta.
Both the celiac trunk and the superior mesenteric artery branch off of the descending aorta before the renal arteries. The graft
placed extends below the renal arteries. Below the renal arteries are the branches for the left and right testes (or ovaries in women)
and the inferior mesenteric. The inferior mesenteric artery supplies the descending colon and beyond via the left colic artery.
Therefore, the descending colon will experience compromised blood flow. The superior mesenteric artery has branches that supply
the transverse and ascending colon.

Take home point: The main branches of the descending aorta are the celiac trunk, the SMA, and the IMA. The renal arteries branch
off after the celiac and SMA but before the IMA. The SMA supplies the ascending and transverse colon, while the IMA supplies the
descending colon.

49. 70-year-old with angina pectoris is referred for cardiac catheterization. Patient worried about hospital-associated
infection. Studies show 30% of patients require admission after procedure, and 2 percent of admitted patients acquire
hospital infection. Patient's risk for infection overall?
- 6/1000


20 ©Test Pirates, LLC. All Rights Reserved.
Step 1 Practice Test #18 Explanations

Why it’s the right answer: Just like in Punnett squares in genetics, the chances of two events occurring when one event depends on
the other is the multiplication of the probability of each individual event occurring. So the chance that the patient will be admitted
(she has to be admitted to get a hospital-acquired infection) is 30% or 30/100, while the chances of her getting an infection while in
the hospital is 2% or 2/100. Therefore, we multiple: 30/100 x 2/100 = 6/1000

Take home point: The chances of two events occurring when one event depends on the other is the multiplication of the probability
of each individual event occurring.

50. 30-year-old woman with fatigue for 2 wks and intermittent fever for 5 days, had teeth cleaned a month ago, no abx
for prophylaxis, and had rheumatic fever as child and endocarditis 2 y ago. PE shows 3/6 murmur, ultrasound shows
abnormal mitral valve. Photo of growth from blood cultures shown (GP cocci in chains). Characteristic of causal
organism?
- Greening reaction on blood agar


Why it’s the right answer: The causal organism is streptococcus viridans, which is alpha-hemolytic. S. pneumo and S. viridans are
both alpha-hemolytic, meaning that when they are cultured on blood agar, they cause partial hemolysis, which looks green.
Complete hemolysis is clear (they cleared all the blood), and those are beta-hemolytic organisms, namely S. pyogenes and S.
agalactiae. S. viridans grows in chains (while S. pneumo grows in pairs). S. viridans is also part of the normal oral flora, so this
patient, with a history of rheumatic fever and endocarditis should have had antibiotic prophylaxis because her heart valves are
already damaged and at risk for repeat endocarditis – which she ended up contracting. S. viridans also does not have a capsule and
optochin resistant. Specifically, S. mutans (type of viridans) is the number one cause of subacute endocarditis in patients with dental
caries and gingivitis.

Take home point: In patients with preexisting heart valve damage who undergo dental procedures (or have dental caries, gingivitis)
are at risk for developing endocarditis due to Streptococcus viridans, an alpha-hemolytic (green) organism.





21 ©Test Pirates, LLC. All Rights Reserved.

You might also like